■ちょっとした疑問や質問はここに書いてね69■

このエントリーをはてなブックマークに追加
1ご冗談でしょう?名無しさん
前スレ
■ちょっとした疑問や質問はここに書いてね68■
http://science4.2ch.net/test/read.cgi/sci/1162751854/

質問する前に
教科書や参考書をよく読もう
http://www.google.com/ などの検索サイトを利用し、各自で調べること
宿題を聞くときは、どこまでやってみてどこが分からないのかを書くこと。
丸投げはダメだからね
(丸投げ君は完全無視。答えるだけ無駄。

質問に対する返答には、何かしらの返答ちょうだいね
★書き込む前に>>2の注意事項を読んでね
★数式の書き方(参考)はこちら>>3
(予備リンク:>>2-10
荒らし厳禁、煽りは黙殺、忘れないうちに定期age
単発質問スレを発見したらこのスレッドへの誘導をよろしくね。

定番FAQ
英語最新版
http://math.ucr.edu/home/baez/physics/
旧版日本語訳
http://research.kek.jp/people/morita/phys-faq/

前スレは既になされた質問に対するやり取りだけにとどめ、新規質問は新スレで行ってください。(レスが間に合わないと無駄なので)

他にも参考にあるサイトなどあればぜひ。
例)http://ja.wikipedia.org/
※wikipedia内の説明はすべてが正確なわけではありません。
このスレでの受け答えもそうですが。相互に補完しつつ精度を高めましょう
過去ログ
http://makimo.to/cgi-bin/search/search.cgi?q=%82%BF%82%E5%82%C1%82%C6%82%B5%82%BD&andor=AND&sf=0&H=&view=table&D=sci&shw=2000
2ご冗談でしょう?名無しさん:2006/11/19(日) 18:56:44 ID:???
書き込む際の注意

1.)
板の性格上、UNIX端末からの閲覧も多いと推察されます。
機種依存文字(ローマ数字、丸数字等)は避けて下さい。

2.)
以下のような質問に物理板住人は飽き飽きしているので、たぶん無視されます。
しないで下さい。
「相対性理論は間違っています」「量子力学は間違っています」
「宇宙論は間違っています」「シュレディンガーの猫は変です」
「永久機関を作りました」「タイムマシンについて教えて」
「どうして〜?」関連(→「どのようにして〜?」と質問すること)
「なぜ〜?」関連(たいてい、物理学の対象ではないため)

「どうして・なぜ」:http://academy.2ch.net/philo/
(哲学板・雑談板のほうがむいている場合が多いです。)
雑談は雑談スレで:http://science3.2ch.net/test/read.cgi/sci/973536997/

3.)
宿題を聞くときは、どこまでやってみてどこが分からないのかを書くこと。
丸投げはダメよ。丸投げに答えるのもダメよ。
せめてポインタを示す程度に留めましょう。

4.)
厨房を放置できない奴も厨房
3ご冗談でしょう?名無しさん:2006/11/19(日) 18:57:16 ID:???
数式の書き方(参考)
●スカラー:a,b,...,z, A,...,Z, α,β,...,ω, Α,Β,...,Ω,...(「ぎりしゃ」「あるふぁ〜おめが」で変換)
●ベクトル:V=[v1,v2,...], |V>,V↑,vector(V) (混同しないならスカラーの記号でいい。通常は縦ベクトル)
●テンソル:T^[i,j,k...]_[p,q,r,...], T[i,j,k,...;p,q,r,...]  (上下付き1成分表示)
●行列  M[i,j], I[i,j]=δ_[i,j]  M=[[M[1,1],M[2,1],...],[M[1,2],M[2,2],...],...], I=[[1,0,0,...],[0,1,0,...],...]
(右は全成分表示。行または列ごとに表示する。例:M=[[1,-1],[3,2]])
●転置行列・随伴行列:M ',tM, M†("†"は「きごう」で変換可) ●行列式・トレース:|A|=det(A), tr(A)
●複号:a±b("±"は「きごう」で変換可)
●内積・外積・3重積:a・b, a×b, a・(b×c)=(a×b)・c=det([a,b,c]), a×(b×c)
●関数・数列:f(x), f[x] a(n), a[n], a_n
●平方根:√(a+b)=(a+b)^(1/2)=sqrt(a+b) ("√"は「るーと」で変換可)
●指数関数・対数関数:exp(x+y)=e^(x+y) ln(x/2)=log[e](x/2)(exp(x)はeのx乗、lnは自然対数)
●三角比:sin(a), cos(x+y), tan(x/2)
●絶対値:|x|  ●共役複素数:z~ ●ガウス記号:[x] (関数の変数表示と混同しないよう注意)
●階乗:n!=n*(n-1)*(n-2)*...*2*1, n!!=n*(n-2)*(n-4)*...
●順列・組合せ:P[n,k]=nPk, C[n,k]=nCk, Π[n,k]=nΠk, H[n,k]=nHk ("Π"は「ぱい」で変換可)
4ご冗談でしょう?名無しさん:2006/11/19(日) 18:57:50 ID:???
数式の書き方続き(参考)
●微分・偏微分:dy/dx=y', ∂y/∂x=y_x ("∂"は「きごう」で変換可)
●ベクトル微分:∇f=grad(f), ∇・A=div(A),∇xA=rot(A), (∇^2)f=Δf ("∇"は「きごう」,"Δ"は「でるた」で変換可.)
●積分:∫[0,1]f(x)dx=F(x)|_[x=0,1], ∫[y=0,x]f(x,y)dy, ∬[D]f(x,y)dxdy, ∬[C]f(r)dl ("∫"は「いんてぐらる」,"∬"は「きごう」で変換可)
●数列和・数列積:Σ[k=1,n]a(k), Π[k=1,n]a(k) ("Σ"は「しぐま」,"Π"は「ぱい」で変換可)
●極限:lim[x→∞]f(x) ("∞"は「むげんだい」で変換可)
●図形:"△"は「さんかく」 "∠"は「かく」 "⊥"は「すいちょく」 "≡"は「ごうどう」 "∽"は「きごう」
●論理・集合:"⇔⇒∀∃∧∨¬∈∋⊆⊇⊂⊃∪∩"は「きごう」で変換
●等号・不等号:"≠≒<>≦≧≪≫"は「きごう」で変換

※この数学記号参考リスト、古いので一度物理向けに更新したほうがいいかも
5ご冗談でしょう?名無しさん:2006/11/19(日) 19:00:23 ID:udPanlaE

+。:.゚ヽ(*´∀`)ノ゚.:。+゚
6ご冗談でしょう?名無しさん:2006/11/19(日) 19:34:49 ID:lrMiSoaK
前スレの991です。下記の件お願い致します。

有機物とか高分子などの絶縁体には
よく静電気が溜まりやすいと言われますが
これはどこに電気が溜まるものなのでしょうか?
絶縁体表面に局所的に導電可能な領域がありコンデンサのように
何か溜まるような仕組みがあるのでしょうか?

>>992さんによると導電領域がなくても静電気を帯びることは
可能ではないかということでしたが、電子をが流れることが
出来ないのであれば、電気をためることも不可能だと思うのですが。
7ご冗談でしょう?名無しさん:2006/11/19(日) 19:54:51 ID:???
すみません、教えてください。

問題:
ヤング率がそれぞれE1,E2、密度ρ1,ρ2の2つの棒が接続されている。
そのときの波の反射率・透過率を求めよ。

とあるのですが、反射率・透過率の求め方がわかりません。
波の振幅によって求めるのかとは思うのですが・・・。
8駆け込み寺!w:2006/11/19(日) 20:36:25 ID:kcc7Tc5Z
素論のひといたら簡単に教えてくれませんか。
リサランドールの5次元って観測可能なんですか?
二つの4次元空間が共存するみたいだけど,
もう一方の4次元は,
我々の4次元と因果関係があるんですか?
9ご冗談でしょう?名無しさん:2006/11/19(日) 21:47:53 ID:2/XXHCkN
表面張力について質問です。
低い濃度の液体の表面張力が蒸留水の表面張力よりも極端に低くなる理由を教えて下さい 。液体の体積比が関係しているようなのですが…
10ご冗談でしょう?名無しさん:2006/11/19(日) 22:33:24 ID:???
11ご冗談でしょう?名無しさん:2006/11/19(日) 22:46:59 ID:???
一次元の拡散方程式を導け、という問題です。
比例関数がk(x)、温度がu(x,t)です。
まず、冲秒間の凅の領域への熱量の流入を考えました。
{k(x+凅)(∂u(x+凅,t)/∂x) - k(x)(∂u(x,t)/∂x)}冲
で、k(x+凅)とu(x+凅)を一次まで展開すると、
k(∂^2u/∂x^2)凅 + (∂k/∂x)(∂u/∂x)凅になりました。

しかし、答えは∂/∂x(k∂u/∂x)凅になっていました。何が間違っているのでしょうか?
よろしくおねがいします。
12ご冗談でしょう?名無しさん:2006/11/19(日) 22:51:06 ID:???
>>11
∂/∂x(k∂u/∂x)= k(∂^2u/∂x^2) + (∂k/∂x)(∂u/∂x)
13ご冗談でしょう?名無しさん:2006/11/19(日) 23:06:35 ID:???
>>12
あー、そうでした!
ありがとうございます!
14ご冗談でしょう?名無しさん:2006/11/19(日) 23:31:42 ID:???
静電気を防ぐためには空気をイオン化すれば
いいと本で読んだのですが
どういうことでしょうか?
15ご冗談でしょう?名無しさん:2006/11/20(月) 00:49:19 ID:???
電磁場とは電場と磁場が交互に振動しながら伝わる波の
ことですよね?
では磁場内や電場内では影響を受けて
例えば光が屈折したりなどしても良いと思うのですが
なぜそういう現象は起きないのでしょうか?
でも確かに電波などには影響するところをみると
波長によるということなのでしょうか?
想像ですが可視光と同じくらいの振動数の電場や磁場であれば
光にも影響が表れるということなのでしょうか?
16ご冗談でしょう?名無しさん:2006/11/20(月) 01:00:37 ID:???
前スレで回答を頂けなかったので、もう一度質問します。
1個の光子による波のコヒーレンス長はどのように計算するのか、ご存知の方いらっしゃい
ませんか。
17ご冗談でしょう?名無しさん:2006/11/20(月) 01:06:58 ID:???
186:2006/11/20(月) 01:07:06 ID:EZZJUbCs
どうかよろしくお願い致します。
19ご冗談でしょう?名無しさん:2006/11/20(月) 01:23:50 ID:???
> では磁場内や電場内では影響を受けて
> 例えば光が屈折したりなどしても良いと思うのですが
> なぜそういう現象は起きないのでしょうか?

真空中の電磁場は線形だから。

A という物体が電磁場 a を生じ、B という物体が電磁場 b を生じるとしたら、
AとBが両方あるときの電磁場は単純に a + b になる。

真空中でなく、媒質内の場合やは必ずしもそうとは限らない。「ファラデー効果」でぐぐるといい。
20ご冗談でしょう?名無しさん:2006/11/20(月) 02:05:12 ID:???
蛍光X線とは特性X線のことである。
と本には書いてあるのですが、それではなぜ同じ事を2つの言い方で表すのでしょうか?
また、これは発生の仕方の問題であって
普通のX線も蛍光X線も区別することは出来ないのですよね?
21ご冗談でしょう?名無しさん:2006/11/20(月) 02:42:15 ID:???
>>7
> とあるのですが、反射率・透過率の求め方がわかりません。
> 波の振幅によって求めるのかとは思うのですが・・・。

波の強度 (単位時間あたりに運ぶエネルギー) の比だ。同じ媒質、同じ振動数なら
振幅の二乗の比と考えてよい

なので、反射率については

  反射波の振幅^2 / 入射波の振幅^2

になる。透過波は別の媒質になるので透過率は振幅の2乗の比にはならないが、
この問題なら

  透過率 + 反射率 = 1

だから透過波の強度を計算する必要はない。きちんと計算した上で、足して1になる
ことを確認したほうが勉強になるけどな。
22ご冗談でしょう?名無しさん:2006/11/20(月) 04:22:04 ID:???
>>20
>それではなぜ同じ事を2つの言い方で表すのでしょうか?
お札も紙幣も同じものですな

>普通のX線も蛍光X線も区別することは出来ないのですよね?
スペクトルを見ればわかる
23ご冗談でしょう?名無しさん:2006/11/20(月) 05:38:29 ID:???
>前スレ997 ありがとうございます。μ秒オーダーとは、確かに長いですね。

>>15 電磁場と電磁波を素で混同してる? 場と言ったらふつうは静的な勾配の所。
場の振動が空間を伝わるのが波。 >可視光と同じくらいの振動数の電場や磁場
それは可視光そのものですがな。
24ご冗談でしょう?名無しさん:2006/11/20(月) 06:37:17 ID:???
>>6
イオンや電子ビームで清浄表面分析するときに帯電が問題になることがある

これが反例
25ご冗談でしょう?名無しさん:2006/11/20(月) 06:38:47 ID:???
反例ってのは
「導電可能な領域が必要だ」
っていう主張に対してね
267:2006/11/20(月) 08:13:14 ID:???
>>21
ありがとうございます。参考になりました。
27ご冗談でしょう?名無しさん:2006/11/20(月) 10:58:20 ID:0+nQ+O0N
領域x∈(-∞,∞),y∈(-t,0),z∈(-∞,∞)に電流密度J=[0,0,J]が分布している。
このとき、点(0,h,0)(h>0)におけるベクトルポテンシャルAを求めよ。

という問題で、A=(μ/4π)∫(J/r)dvという公式からAを求めようとして、
∫[-∞,∞]dx∫[-t,0]dy∫[-∞,∞]dz J*(x^2+(y-h)^2+z^2)^(-1/2)
の計算をしてみたのですが、発散してしまって求まりません。

どこが問題なのでしょうか?
公式を使うのに条件があるのでしょうか?
28ご冗談でしょう?名無しさん:2006/11/20(月) 11:32:18 ID:ksTvMjDG
個体(金属など)中を伝播する音速の値が2000m/sになることはありえますか
29ご冗談でしょう?名無しさん:2006/11/20(月) 11:36:42 ID:???
30ご冗談でしょう?名無しさん:2006/11/20(月) 11:40:11 ID:EZZJUbCs
モノクロメーターと回折格子の違いが
分かりません。
名前は違いますけど同一のものと考えて良いのでしょうか?
31ご冗談でしょう?名無しさん:2006/11/20(月) 11:54:00 ID:???
>>19
回答ありがとうございます。
「ファラデー効果」については知らなかったので検索して調べました。
しかしあなた様の仰る
>真空中の電磁場は線形だから。
>A という物体が電磁場 a を生じ、B という物体が電磁場 b を生じるとしたら、
>AとBが両方あるときの電磁場は単純に a + b になる。
ということがどうしても理解できません。

媒質内だと線形結合で表されず電場と磁場は直交しているという言いたいのですよね?
なぜ媒質中だとそういう現象が起こるのでしょうか?
自分でも考えてみましたが全く検討もつきません。
どういうことなのでしょうか?

>>23
回答ありがとうございます。
電磁場と電磁波は理解しております。
しかし上記の書き方は確かにおかしかったですね。
例えば磁場だけのパルスや電場だけのパルスであっても
電磁波に影響するのでしょうか?
或いは静的な勾配であっても電磁波の伝達のしやすさなどに
影響は与えられないものなのでしょうか?
32ご冗談でしょう?名無しさん:2006/11/20(月) 11:55:52 ID:???
>>27
計算間違えたんじゃない?
33ご冗談でしょう?名無しさん:2006/11/20(月) 12:02:57 ID:???
>>22
回答ありがとうございます。
スペクトルを見ればわかるということですが
www.sumibe.co.jp/.../technicalnote/img/2_01b.gif
ピークの部分はその原子に特有の値を持ちます
しかしハローを取り除いたりそのピーク位置を
何らかの方法でシフトさせたりなどしたら
もちろん蛍光X線なのかそうでないのかは判別不可能になりますよね?
34ご冗談でしょう?名無しさん:2006/11/20(月) 12:04:51 ID:???
>>31
>媒質内だと線形結合で表されず電場と磁場は直交しているという言いたいのですよね?

そんなこと言いたくないだろ。

>例えば磁場だけのパルスや電場だけのパルスであっても

水面に伝搬しないパルス作れる?
35ご冗談でしょう?名無しさん:2006/11/20(月) 12:20:09 ID:???
>>33
見えません。
つぅか何が言いたいのかさっぱりわかりません。
36ご冗談でしょう?名無しさん:2006/11/20(月) 12:43:40 ID:???
硬X線と軟X線の違いは単に波長領域が
異なるということですよね?
37ご冗談でしょう?名無しさん:2006/11/20(月) 12:48:29 ID:???
はい。
38ご冗談でしょう?名無しさん:2006/11/20(月) 13:19:21 ID:XYY6wZMs
金属棒にコイルを巻き、電流を流すと電磁石になるそうです

このとき分子はどのような挙動を示して磁性を持つようになるのでしょうか?

教えてください
39ご冗談でしょう?名無しさん:2006/11/20(月) 13:22:21 ID:???
宿題みたいな質問ですね
40ご冗談でしょう?名無しさん:2006/11/20(月) 13:42:14 ID:???
こんな妙な質問宿題にはでない
41ご冗談でしょう?名無しさん:2006/11/20(月) 13:48:03 ID:jnQj1y5j
1モルの水が1atm、100℃で気化したことによる体積増加の外部に対する仕事を計算せよ

これ意味わかりません
なんかヒントとかありますか?
4227:2006/11/20(月) 13:52:02 ID:0+nQ+O0N
>>32
∫[-∞,∞](x^2+k^2)^(-1/2)dx
が発散することと同値ですよね?
43ご冗談でしょう?名無しさん:2006/11/20(月) 13:53:29 ID:???
>>41
スレまで立てて答もらったのにまた同じこと聞くのか
http://science4.2ch.net/test/read.cgi/sci/1163010835/
44ご冗談でしょう?名無しさん:2006/11/20(月) 13:57:03 ID:jnQj1y5j
>>43
ありがと
まさか同じ学科のヤツが立てたのか・・・

とりあえずすまんかった
45ご冗談でしょう?名無しさん:2006/11/20(月) 14:24:51 ID:???
>>42
元の式が違う気がする。
4645:2006/11/20(月) 14:28:02 ID:???
いやあってるか
47ご冗談でしょう?名無しさん:2006/11/20(月) 16:44:18 ID:???
化学ではキラルと呼ぶものを
素粒子物理ではカイラルと呼ぶのはなぜですか?
48ご冗談でしょう?名無しさん:2006/11/20(月) 17:07:41 ID:???
>>47
原語のスペルを参照せよ

以上
49ご冗談でしょう?名無しさん:2006/11/20(月) 17:23:10 ID:???
どちらも同じChiral なんですが・・・

50ご冗談でしょう?名無しさん:2006/11/20(月) 17:44:04 ID:???
習慣じゃない?
たぶん、それぞれの分野の日本での創始者がそう使い始めちゃったってだけかと
51ご冗談でしょう?名無しさん:2006/11/20(月) 18:05:47 ID:Wc6sfIZX
もしかしたら散々ガイシュツかも知れませんが、質問させてください。

もし、音速で(もしくは超音速で)拳を突き出すことができたら、衝撃波みたいなのって出るんですか?
拳が焼けてしまうとかちぎれてしまうとかは無しで、「できたら」の話ですが。

また、その衝撃波を人が食らったとしたら、すぐ近くで食らうのと、例えば1メートルくらい
離れた所で食らうのではどれくらい衝撃が変わってくるでしょうか。
なんとなくでいいので教えていただければ幸いです。
52ご冗談でしょう?名無しさん:2006/11/20(月) 18:19:27 ID:p34zYEzy
オリジナル問題です!懸賞問題です。早いもの順で一問1000円です。早いもの順です。
大学三年レベルですのでそんなに難しくないと思います。

http://web.hc.keio.ac.jp/~fr054016/billsuzu-password.html

みなさまの挑戦をお待ちしています。
53ご冗談でしょう?名無しさん:2006/11/20(月) 18:21:05 ID:???
しね
54ご冗談でしょう?名無しさん:2006/11/20(月) 18:48:30 ID:???
>>51
音速では無理だけど超音速だとまさしく衝撃波が出る

>また、その衝撃波を人が食らったとしたら、すぐ近くで食らうのと、例えば1メートルくらい
>離れた所で食らうのではどれくらい衝撃が変わってくるでしょうか。

メチャメチャ違う。減りまくり。
つうか衝撃波の一番の被害者はそのこぶし。
55ご冗談でしょう?名無しさん:2006/11/20(月) 19:52:06 ID:bEe7WGkI
たとえばあるものをひっくり返したあと、もう一度ひっくり返すと、
ひっくり返さなかったときと同じ状態になるのはなぜなのでしょうか。
二回操作したのに、何もしなかったと同じ状態になるのはなぜなのでしょうか。
この世界がそのようにできているのは何故ですか。
56ご冗談でしょう?名無しさん:2006/11/20(月) 19:58:56 ID:???
二回操作しても元に戻らん操作なんてゴマンとあるが?
57ご冗談でしょう?名無しさん:2006/11/20(月) 20:10:27 ID:bEe7WGkI
はぐらかさないで。
58ご冗談でしょう?名無しさん:2006/11/20(月) 20:17:53 ID:???
>たとえばあるものをひっくり返したあと、もう一度ひっくり返すと、
>ひっくり返さなかったときと同じ状態になるのはなぜなのでしょうか

ならない場合が多々あります、よってそれ以降の質問には回答できません
コップに水入れてさかさまにして、コップを元に戻したら水も元に戻りますか?
59ご冗談でしょう?名無しさん:2006/11/20(月) 20:25:29 ID:bEe7WGkI
はぐらかすという意味がわかりませんか。
小学生に質問しているのではありません。
60ご冗談でしょう?名無しさん:2006/11/20(月) 20:28:04 ID:???
>>57
そんなつもりはなくて、二回操作しても元に戻らん操作なんてゴマンと
あるから、なぜもへったくれもない。単にゴマンとある操作の中から
二回操作で元に戻るものを考えただけであって、それが元に戻るのは
定義から当然。

61ご冗談でしょう?名無しさん:2006/11/20(月) 20:28:32 ID:bEe7WGkI
それともこのスレにはふさわしくありませんか。
物理学の範囲と思ったのですけれど・・
62ご冗談でしょう?名無しさん:2006/11/20(月) 20:29:13 ID:???
>>59
とりあえず>>55をもっと具体的に書いてくれ
抽象的すぎて意味分からん
63ご冗談でしょう?名無しさん:2006/11/20(月) 20:29:58 ID:bEe7WGkI
定義というのは物理学ではありません。
64ご冗談でしょう?名無しさん:2006/11/20(月) 20:30:16 ID:???
>>59
はぐらかしてるのはそっちだろと言いたい
何が疑問なのか全くわからん

65ご冗談でしょう?名無しさん:2006/11/20(月) 20:31:30 ID:???
ひっくり返しても元に戻らない「不可逆変化」
ttp://www.google.co.jp/search?hl=ja&q=%E4%B8%8D%E5%8F%AF%E9%80%86%E5%A4%89%E5%8C%96&lr=
66ご冗談でしょう?名無しさん:2006/11/20(月) 20:32:09 ID:bEe7WGkI
あなたはこの世界のあり方について考えることはないのですか。
67ご冗談でしょう?名無しさん:2006/11/20(月) 20:34:36 ID:???
>>66
考えるのは物理というか自然科学の究極目標だが
おまえの想定している世界はこの世界とは異なる

詳しくは>>65
68ご冗談でしょう?名無しさん:2006/11/20(月) 20:34:57 ID:bEe7WGkI
この世界の仕組みについての質問がふさわしくないのなら、
他の板に行きます。
どこがいいでしょうか。
69ご冗談でしょう?名無しさん:2006/11/20(月) 20:36:31 ID:???
70ご冗談でしょう?名無しさん:2006/11/20(月) 20:37:54 ID:???
つーか、アンタの質問が「この世界の仕組みについての質問」とは思えない
71ご冗談でしょう?名無しさん:2006/11/20(月) 20:38:09 ID:???
>>68
単に二回操作すれば元に戻るものを考えれば二回操作で元に戻るってだけ。
この世界のあり方とか仕組みとか、ミジンコも関係ないから
72ご冗談でしょう?名無しさん:2006/11/20(月) 20:39:02 ID:???
>>68
ふさわしくないことはないが
>>55で「世界はAであることは何故ですか?」
と聞いているAがすでに成り立たない場合多数なのでどうしようもない

「1+1が3になるのは何故ですか!!!」
と聞いているような物
「いや、3じゃないから」
としか言えない
73ご冗談でしょう?名無しさん:2006/11/20(月) 20:39:06 ID:bEe7WGkI
どうもずれているようです。
わたしが言いたいのは、
一枚の板をひっくり返したあともう一度ひっくり返すと、
もとの状態になるということです。
このような宇宙の振る舞いは何によって保障されているのかということです。
74ご冗談でしょう?名無しさん:2006/11/20(月) 20:41:48 ID:???
>>73
板の剛性
75ご冗談でしょう?名無しさん:2006/11/20(月) 20:42:02 ID:???
>>73
宇宙の振る舞いじゃなくて板のふるまいな
76ご冗談でしょう?名無しさん:2006/11/20(月) 20:42:28 ID:bEe7WGkI
はぐらかしです。
77ご冗談でしょう?名無しさん:2006/11/20(月) 20:44:02 ID:???
わかった。
板違いだ。
ここの住民には答えられない。
78ご冗談でしょう?名無しさん:2006/11/20(月) 20:44:40 ID:???
哲学板あたりで訊くと良いよ
79ご冗談でしょう?名無しさん:2006/11/20(月) 20:44:44 ID:???
>>76
気に入らなきゃはぐらかしか。

板の振る舞いを宇宙の振る舞いに無理矢理結び付けねば気が済まないおまいの
特異な思考回路の責任まで取れるかっての
80ご冗談でしょう?名無しさん:2006/11/20(月) 20:46:24 ID:???
ここにヒントがあると思う。
数学の問題だなw
http://www32.ocn.ne.jp/~gaido/fusigi/gun.htm
81ご冗談でしょう?名無しさん:2006/11/20(月) 20:47:43 ID:???
実際は戻ってない。ひっくり返す過程でエネルギーが使われ
それは周囲に広がり結局は熱になって周囲をちょっと変える。
板自身にも微妙に振動が残る。やはり熱に変るが。
これだけでエネルギー保存とエントロピー増大という宇宙の振舞が
現れてる。アンタの見方は大雑把すぎる。
82ご冗談でしょう?名無しさん:2006/11/20(月) 20:48:54 ID:???
>>79
おい、板の振る舞いは宇宙の振る舞いだと思うぞ。
それを否定しちゃ物理学の敗北だぜ。
83ご冗談でしょう?名無しさん:2006/11/20(月) 20:50:25 ID:???
板の振る舞い≠宇宙の振る舞い
板の振る舞い⊂宇宙の振る舞い
84ご冗談でしょう?名無しさん:2006/11/20(月) 20:51:08 ID:3fqMyUjJ
>>73 他の人が言っているようにそんな奇跡的現象見た事一度もないよ。
一枚の板を二回ひっくり返すと元の状態になる?きっと貴方の五覚が鈍い
だけだよ。で、敢えて鈍い状態に感覚を保って、「元の状態になる」と
するなら、それは「ひっくり返す」という操作かだ必然的に出てくる
結果であって、宇宙の振る舞いなどと大仰に構える必要の無い物だよ。で、
なんで世の中には2回ひっくり返すともとに戻るように見える操作が色々あるのだろう?
と思うのなら、元の疑問より数等高等な問題意識だな。とにかく今の貴方の言葉から
こちらが理解出来る範囲では貴方の問題意識が低すぎる、と言う事だけだ。
85ご冗談でしょう?名無しさん:2006/11/20(月) 20:52:34 ID:???
俺は問題意識が高いと見た。
86ご冗談でしょう?名無しさん:2006/11/20(月) 20:53:40 ID:???
じゃ答えてやれよ
87ご冗談でしょう?名無しさん:2006/11/20(月) 20:55:00 ID:???
物理の問題じゃないな。
それはそれで高度の疑問だと言えるだろう。
88ご冗談でしょう?名無しさん:2006/11/20(月) 20:57:28 ID:bEe7WGkI
みなさんありがとうございました。
>80が激しく参考になりそうです。
89ご冗談でしょう?名無しさん:2006/11/20(月) 20:58:48 ID:???
元に戻らない>>65とか>>69を参考にしないか?
自分の考えは全部正しくないと気が済まない人?
90ご冗談でしょう?名無しさん:2006/11/20(月) 21:03:50 ID:bEe7WGkI
>89さん、
そのようなことは定性的には理解しています。
私の疑問はもっと根本的(非物理学的)なものだったようです。
私は中学3年生です。失礼しました。
91ご冗談でしょう?名無しさん:2006/11/20(月) 21:06:13 ID:???
よくまあそう神経に障るものいいをするもんだ
92ご冗談でしょう?名無しさん:2006/11/20(月) 21:06:28 ID:???
まあ、たまにはこんなことでもなけりゃあ、世の中面白くないわな。
93ご冗談でしょう?名無しさん:2006/11/20(月) 21:07:53 ID:???
凡才が天才に嫉妬してる訳ね。
94ご冗談でしょう?名無しさん:2006/11/20(月) 21:09:51 ID:???
もうちょっと親切にしてやれば良かったんだよ。
95ご冗談でしょう?名無しさん:2006/11/20(月) 21:10:29 ID:???
>>90
物理板に来て非物理学的な質問をしても、まともな答えが得られないのは当たり前。
それで逆ギレして「はぐらかされた」じゃ、まさに「すぐキレる子供」の典型例だな。
そういう自分が一番悪い、という事実をよーく自覚するように > ID:bEe7WGkI
96ご冗談でしょう?名無しさん:2006/11/20(月) 21:10:31 ID:mr03o2vI
>>27
僕も前ベクトルポテンシャルの問題やったことあったが、
そのときは無限遠をベクトルポテンシャルの基準(A(∞)=0)とすると
任意の位置xのA(x)が発散してしまうから
無限遠じゃない別の場所を基準にする、って方法をとった。
君の使ってるその公式も無限遠が基準点になってるとかじゃないかな?
とりあえずそのこと調べてみてくれ。
97ご冗談でしょう?名無しさん:2006/11/20(月) 21:18:50 ID:???
お説教はやめろや。
公平に見て住民の敗北だぜ。
98ご冗談でしょう?名無しさん:2006/11/20(月) 21:20:23 ID:???
この板ID表示にすればいいのに
99ご冗談でしょう?名無しさん:2006/11/20(月) 21:24:20 ID:2DispE5b
>>33 亀れすだが、
電子によって励起されれば蛍光X線
X線によって励起されれば特性X線
学問的な名称 ものはおなじもの
100ご冗談でしょう?名無しさん:2006/11/20(月) 21:24:43 ID:???
>>90=93=97
みえすいたアホウ
101ご冗談でしょう?名無しさん:2006/11/20(月) 21:29:14 ID:2DispE5b
>>36
おれはエネルギー領域で考えるけど
どちらもおなじか
102ご冗談でしょう?名無しさん:2006/11/20(月) 21:41:35 ID:Wc6sfIZX
>>54
ありがとうございます。
103ご冗談でしょう?名無しさん:2006/11/20(月) 22:24:07 ID:???
回答者の質が低いな…。
任意の状態間を結ぶ可逆過程が存在することは
熱力学の基本的な要請だと思うんだが。
104ご冗談でしょう?名無しさん:2006/11/20(月) 22:41:04 ID:mYTCHdVY
熱力学の基本的な要請熱力学の基本的な要請熱力学の基本的な要請熱力学の基本的な要請
熱力学の基本的な要請熱力学の基本的な要請熱力学の基本的な要請熱力学の基本的な要請
熱力学の基本的な要請熱力学の基本的な要請熱力学の基本的な要請熱力学の基本的な要請
熱力学の基本的な要請熱力学の基本的な要請熱力学の基本的な要請熱力学の基本的な要請
熱力学の基本的な要請熱力学の基本的な要請熱力学の基本的な要請熱力学の基本的な要請
熱力学の基本的な要請熱力学の基本的な要請熱力学の基本的な要請熱力学の基本的な要請
105ご冗談でしょう?名無しさん:2006/11/20(月) 23:02:36 ID:???
↑質の低い回答者の一例
106ご冗談でしょう?名無しさん:2006/11/20(月) 23:59:01 ID:???
>>103
熱が低温から高温に移動するとでも?
107ご冗談でしょう?名無しさん:2006/11/21(火) 00:23:10 ID:???
>>106
ヒートポンプ知らないの?
10827:2006/11/21(火) 00:33:30 ID:GdA1aj96
>>96
あ、そうですね。基準点が臭いですね。
解答にはA0なる定数項が加えられていましたので、
もしかしたらココに無限大が丸め込まれているのでしょうか。
自分はベクトル解析も偏微分方程式も独学でしか知らない一年坊なので
また少し時間をかけて考えてみます。

ところで、模範解答では、電荷についての「アナロジー」から、
電流密度を電荷に置き換えた双対問題を解いているのですが、
ポアソン方程式の問題はこのようにアナロジーで解く手法が一般的なのでしょうか?

そもそもベクトルポテンシャルを求める問題自体が少数に思われますが
何冊かテキストを見てみても、方程式自体を解析的に解くという解法がみられないのですが。
109ロップマン ◆7DVSWG.5BE :2006/11/21(火) 00:58:03 ID:???
怪物理男爵はどこにいる?
11096:2006/11/21(火) 01:00:25 ID:si1c153I
>>108
お役に立てたようでうれしいです。

あ、正解が分かってるんですか?
となると正解のA(x)がゼロになるようなxを探せば、そこが基準点ですね。

ええっとね・・・無知なため「アナロジー」とかよく分かりません、
ごめんなさい。

ベクトルポテンシャルAって
スカラーポテンシャルφ(F=-gradφ、Fは保存力)
と同じような形で定義できるじゃん(B=rotA、Bはベクトル場)!
て理由で作られたみたいなものらしいから
(そこまでいいかげんかは不明)、
そもそもA自体の意味はあるにはあるけどあまり使われない
存在なのかもしれない。そういう意味で演習問題も少ないのかも。
111ロップマン ◆7DVSWG.5BE :2006/11/21(火) 01:01:13 ID:???
http://sports9.2ch.net/test/read.cgi/budou/1163995629/l50
怪物理男爵のために立てたぞ!
112ご冗談でしょう?名無しさん:2006/11/21(火) 01:01:55 ID:???
>>107
可逆過程か?
113ご冗談でしょう?名無しさん:2006/11/21(火) 01:31:37 ID:???
>>99
はじめて知りました。
114ご冗談でしょう?名無しさん:2006/11/21(火) 02:17:48 ID:???
>>107
熱力学習ったばかりでうれしいのは分かるけど
もう少ししたらエントロピーとか習うからね
115ご冗談でしょう?名無しさん:2006/11/21(火) 08:05:30 ID:???
>>107
カルノーサイクル知らないの?
>>114
出直しておいで。
116薬学部生:2006/11/21(火) 08:22:27 ID:???
すみません、物理化学の問題集に関して質問なんですが、
今大学一年生で物理化学やってます。物理化学の初学者が、授業と並行して
演習できる問題集をどなたか知っていらしたら教えていただきたいのですが。


117ご冗談でしょう?名無しさん:2006/11/21(火) 10:41:51 ID:???
なんで銀河とか惑星とかって回転するんですか?
118ご冗談でしょう?名無しさん:2006/11/21(火) 11:28:32 ID:???
>>116
聞くなら化学板のほうだろ
http://science4.2ch.net/test/read.cgi/bake/1107950798/
119ご冗談でしょう?名無しさん:2006/11/21(火) 11:30:49 ID:???
>>116
あえて物理板でその答えを出すなら田崎晴明さんの熱力学だろうな。
120ご冗談でしょう?名無しさん:2006/11/21(火) 11:36:09 ID:???
あれは計算練習というより、一通り習った後の色々な疑問に答えるのにいい本だと思う。
121ご冗談でしょう?名無しさん:2006/11/21(火) 11:37:13 ID:???
じゃあキャレン
122ご冗談でしょう?名無しさん:2006/11/21(火) 11:40:47 ID:???
定番だけど。。物理化学=熱力学なのか?
123ご冗談でしょう?名無しさん:2006/11/21(火) 11:41:39 ID:???
物理化学の殆どは熱力学だと認識してるが。。。
たまに量子力学も入ってくるようだが。
124ご冗談でしょう?名無しさん:2006/11/21(火) 11:44:37 ID:???
物理化学―分子論的アプローチ

量子論の夜明け
古典的波動方程式
シュレーディンガー方程式と箱の中の粒子
量子力学の仮説と一般原理
調和振動子と剛体回転子:二つの分光学モデル
水素原子
近似的方法
多電子原子
化学結合:二原子分子
多原子分子における結合〔ほか〕
気体の性質
ボルツマン因子と分配関数
分配関数と理想気体
熱力学第一法則
エントロピーと熱力学第二法則
エントロピーと熱力学第三法則
ヘルムホルツエネルギーとギブズエネルギー
相平衡
溶液(液‐液 溶液
固‐液 溶液)〔ほか〕

量子論と熱力学(及び統計力学)か?
125ご冗談でしょう?名無しさん:2006/11/21(火) 11:45:29 ID:???
この内容なら分子運動論30講とかかな。
でも演習無いな。
普通に物理化学の本を買うのが良いかと。
126ご冗談でしょう?名無しさん:2006/11/21(火) 12:12:56 ID:DMRuPlbo
どこで質問すれば良いのか分からないので、この板で質問します。

人間の体を構成するのに必要とされる元素の中で、鉄より重い元素はありますか?
127ご冗談でしょう?名無しさん:2006/11/21(火) 12:17:12 ID:???
亜鉛とかヨウ素とか
128ご冗談でしょう?名無しさん:2006/11/21(火) 13:37:32 ID:3q8jppjx
変圧器で電流と電圧の比を変えたら
オームの法則V=RIは成り立たないんじゃね?
129ご冗談でしょう?名無しさん:2006/11/21(火) 14:13:43 ID:VyvJRQJ3
鳥は何故とべるんですか?
130ご冗談でしょう?名無しさん:2006/11/21(火) 14:52:16 ID:???
>>127
ありがとーございました
131ご冗談でしょう?名無しさん:2006/11/21(火) 15:12:22 ID:???
>>124
すげーな。これだけの内容どうやって一年生でやるんだ?
詳しいとこは適当にはしょるんだろうけど、それでついていけるんかね。

と量子化学落としたおれは思うのであった。
132ご冗談でしょう?名無しさん:2006/11/21(火) 16:45:01 ID:I/AYn8fy
地球の重量>月の重量で
重力と引力は比例するから
地球の引力>月の引力になりますよね?
とゆうことは年々月は地球に引っ張られているはずですが、
月と地球はいつ、くっ付くのですか?
133ご冗談でしょう?名無しさん:2006/11/21(火) 16:50:14 ID:???
>>132
いちいち訂正するのもマンドクセーというほどいろいろ間違ってる
134ご冗談でしょう?名無しさん:2006/11/21(火) 18:12:09 ID:???
>>132
第一文から第四文まで全部間違えてる。
出なおせ。
135ご冗談でしょう?名無しさん:2006/11/21(火) 20:49:16 ID:MPT7gKJZ
>>108 発展問題。今厚さdの、無限に長い円筒中を電流が面密度Jで周回しているとする。
この時のベクトルポテンシャルを求めよ。
>模範解答では、電荷についての「アナロジー」から、
>電流密度を電荷に置き換えた双対問題を解いている
の意味が今ひとつ理解出来ない(双対?)のだが、もし
ポアッソン方程式(通常この言葉はスカラーに対する言葉に用います)に
帰着させる方法なら、上に挙げた問題は結構解くのが面倒になると思います。
それより素直にベクトルポテンシャルの定義に沿って解いた方が良い。
136ご冗談でしょう?名無しさん:2006/11/21(火) 22:18:56 ID:???
>>117
もともと回転していたから。
外部から回転を止めるような角運動量を与えられない限り、回転し続ける。

>>132
マンドクセーけど、いちいち訂正してやる。

>地球の重量>月の重量で
「重量」とは、その物体に働く重力の大きさのこと。
従って、重力場の状態によって、いくらでも変わる。
「質量」なら変わらないし、地球の質量>月の質量、だが。

>重力と引力は比例するから
「重力」とは「万有引力」のこと。比例するも何も、同じもの。

>地球の引力>月の引力になりますよね?
なんでやねん。
作用反作用の法則により、地球の引力=月の引力、となる。

>とゆうことは年々月は地球に引っ張られているはずですが、
地球を基準にして見れば、月が地球に引っ張られているように見えるが、
月を基準にして見れば、地球が月に引っ張られているように見えるぞ。

>月と地球はいつ、くっ付くのですか?
ちなみに、実際には、月と地球は離れつつある。
137ご冗談でしょう?名無しさん:2006/11/21(火) 23:55:57 ID:07XJXpNM
空はなぜ青いのだろうか?
13838:2006/11/21(火) 23:59:00 ID:HrrGM+Gi
38です
だれも答えてもらってないようですが、電気電子板という所
で質問した方がいい内容なのでしょうか?
139ご冗談でしょう?名無しさん:2006/11/22(水) 00:05:20 ID:???
>>138
ヒント
スピン
140ご冗談でしょう?名無しさん:2006/11/22(水) 01:16:42 ID:???
>>138

いや、分子関係ないから。
141ご冗談でしょう?名無しさん:2006/11/22(水) 02:02:39 ID:???
>>137
海の青が反射している

というのは冗談だ

「光の散乱 空の青」でググれ
14238:2006/11/22(水) 02:43:10 ID:wDsy/3j7
分子軌道上の電子のスピンが一様になるため磁性が変化し
その状態がしばらく続くのは軌道がバンドになっているから

という考え方でいいのでしょうか?
143ご冗談でしょう?名無しさん:2006/11/22(水) 02:56:13 ID:???
>>142
覚えたての単語をご披露したい気持ちは分からないでもないが、

あまりに支離滅裂すぎてコメント不可。
14438:2006/11/22(水) 08:26:35 ID:wDsy/3j7
結局どういうことのでしょうか?

教えて下さい
145ご冗談でしょう?名無しさん:2006/11/22(水) 09:41:02 ID:???
>>144

電流による磁場は、いわゆる磁性体での電子スピンや軌道角運動量によるものとは
まったく別。混ぜるな。
146ご冗談でしょう?名無しさん:2006/11/22(水) 09:47:00 ID:???
>>145
コイルの磁場で磁性体が磁化されることはある

でも
>金属棒にコイルを巻き、電流を流すと電磁石になるそうです
そうとは限らない
147145:2006/11/22(水) 11:27:01 ID:???
>>146

ああ、なるほど。その部分の記述に注目すれば確かに関係あるか。
そこまでは考えが回らんかった。

とするとそれを考慮して>>38への回答はこうか。

電磁石の心棒に強磁性体を使った場合に自発磁化が残るのは、単純に
磁区が揃って自発磁化の方向が揃うため。
電流を流している間だけある程度の磁化を持つ場合は、バンド構造に
基づくPauliパラとか内核電子の反磁性(逆向き)とか。
148ご冗談でしょう?名無しさん:2006/11/22(水) 14:21:37 ID:Llq/luAq
気体がマクセル分布しているとすると、速度ゼロの分子が最も多いということになるらしいのですが、
どうも納得できません。どう考えたら良いのでしょうか?
149ご冗談でしょう?名無しさん:2006/11/22(水) 14:23:37 ID:9e9vL9qF
ならん。統計力学でマクスウェル分布を計算してだしてみろ。
150ご冗談でしょう?名無しさん:2006/11/22(水) 14:43:10 ID:???
>速度ゼロの分子が最も多い

まあ1次元ならそうだろうな。
151ご冗談でしょう?名無しさん:2006/11/22(水) 14:49:04 ID:I/tamWtW
この板の住人にとっては既出かもしれませんが、答えを解説してください。
お願いします。

力のモーメント?
http://www.youtube.com/watch?v=hYdlE3TiVDY
152ご冗談でしょう?名無しさん:2006/11/22(水) 14:56:34 ID:E5Cg660S
ニュートンはどうやって重力と距離の関係を突き止めたの?
地球の大きさからしたら10mや100mなんて誤差じゃないの?
153ご冗談でしょう?名無しさん:2006/11/22(水) 15:10:29 ID:9e9vL9qF
ケプラーが、ケプラーが付いてた変わり者の金持ちが作った天体の観測結果を図に書き出したら、その動きが楕円で、
ガリレオの運動法則を発展させてr^(-2)型の力を発展させたらキレいにまとまった。
154ご冗談でしょう?名無しさん:2006/11/22(水) 18:33:07 ID:IBz8rlDd
どなたか、マイクロチャンネルプレートで検出したイオンをカウンティングする方法を教えてください。
155ご冗談でしょう?名無しさん:2006/11/22(水) 18:38:56 ID:???
スケーラーに突っ込め



以上
156ご冗談でしょう?名無しさん:2006/11/22(水) 19:38:20 ID:hNx/OSUy
立方メートルが「m3」とすると
「m4」は何て読むんですか?
157ご冗談でしょう?名無しさん:2006/11/22(水) 19:41:06 ID:???
えむ(めーとる)よんじょう
158ご冗談でしょう?名無しさん:2006/11/22(水) 19:46:31 ID:hNx/OSUy
>>157
立方メートルのような呼び方は無いんでしょうか?
159ご冗談でしょう?名無しさん:2006/11/22(水) 21:18:32 ID:???
>>108の代わりに補足しておきますが、
108さんのいうアナロジーというのは、例えば、円柱表面の電流が流れている時、
円柱表面の電荷が作るポテンシャルを求めて、(ガウスの法則で電場を求め、線積分)
それをρ→i↑に置き換える。という方法で、初歩的な問題集なんかによく載ってる方法です。

定義から求めるやり方は計算が面倒なので、あんまり問題集にのっていませんね。
僕もまだ2年なので、将来的にベクトルポテンシャルが重要になるというのは話でしか聞いたことがありませんが
2年生向けの電磁気の教科書では、ベクポに関する問題は、電荷と電流のアナロジーを使う問題が少し載ってる程度ですね。
両方できるようにしたらいいんじゃないんですか?
160110:2006/11/22(水) 22:22:19 ID:z3Yp/40H
>>159
なるほど、アナロジーの意味がなんとなく見えてきました。
あとベクポってちゃんと重要な物理量だったんだ、知らなかった。
ありがとうございました。
161ご冗談でしょう?名無しさん:2006/11/23(木) 00:09:44 ID:oY8Ygztl
アインシュタイン方程式に出てくる接続記号ですが、
Γ{d,ab}=1/2g{cd}(∂g/∂x+・・・)

このcは何を表してるんでしょうか?分かりづらくてすみません。
162ご冗談でしょう?名無しさん:2006/11/23(木) 00:22:32 ID:???
>>161
 テンソルの足ぐらい省略せずに書け。
 たぶん、アインシュタインの規約により足し上げられているダミーインデックスがcなんだろうな。
頭の中でΣ_{c=0,1,2,3}を補完して嫁。
163ご冗談でしょう?名無しさん:2006/11/23(木) 01:16:32 ID:B14P+3Ux
一般的な霧吹きで作った水滴1つあたりの体積って平均でどれくらいですか?
164ご冗談でしょう?名無しさん:2006/11/23(木) 01:35:07 ID:???
>>158
木が4つでジャングルとかいってたやつ思い出した。
165ご冗談でしょう?名無しさん:2006/11/23(木) 03:12:46 ID:???
>>163
一般的な霧吹きなら

ばらつくから答えようがない
166ご冗談でしょう?名無しさん:2006/11/23(木) 03:38:49 ID:???
http://www.youtube.com/watch?v=hYdlE3TiVDY

YouTubeで発見したもの
力のモーメント・・?
習った記憶ナッシング。
回答お願いします。
167166:2006/11/23(木) 03:39:19 ID:nnBMuOHM
一回あげます。
168ご冗談でしょう?名無しさん:2006/11/23(木) 05:41:17 ID:???
またそれか。
ググるなり過去ログ読むなりご自由に。
169ご冗談でしょう?名無しさん:2006/11/23(木) 07:54:20 ID:???
物理学やってんのにモーメントもわかんねえの?(プ
170ご冗談でしょう?名無しさん:2006/11/23(木) 07:58:08 ID:???
>>165
平均の意味も知らない奴がでてくるな
171ご冗談でしょう?名無しさん:2006/11/23(木) 10:39:45 ID:???
>>170
人をあざわらう前に、まずお答え頂けますか?
172ご冗談でしょう?名無しさん:2006/11/23(木) 10:44:19 ID:???
>>171
平均くらい辞書引けよww
173ご冗談でしょう?名無しさん:2006/11/23(木) 11:06:30 ID:???
>>170
あざわらってるのは >>165 。そんなこともわかんねーのか
174ご冗談でしょう?名無しさん:2006/11/23(木) 11:17:32 ID:???
>>173
アンカーくらいまともにうてないのか??www
175ご冗談でしょう?名無しさん:2006/11/23(木) 11:18:48 ID:???
低レベルな争いだな
176ご冗談でしょう?名無しさん:2006/11/23(木) 11:19:46 ID:???
>>163 が低(ry
177ご冗談でしょう?名無しさん:2006/11/23(木) 11:33:21 ID:???
>>163の質問自体は普通だろ。
それに答えられない物理板の人々(特に>>165)。
低脳172に釣られる>>173=171(テンパリながらアンカー誤爆する。)
自分のミスを認めない>>175=173=171。

まぁ物理板の人達ってこんなもんだろうな。
178ご冗談でしょう?名無しさん:2006/11/23(木) 11:35:13 ID:S/o75dYV
>15>23
のことでちょっとお聞きしたいのですが
電場・磁場単体では電磁波に影響を及ぼさないというなら
なぜ電磁波は電磁波に影響を及ぼさないのでしょうか?
電磁波と電磁波を直行してやることで電磁波同士で
散乱を起こしたり進行方向が変わってもおかしくないと思うのですが。
179ご冗談でしょう?名無しさん:2006/11/23(木) 11:37:55 ID:???
いやおっしゃるように光子-光子散乱もありますよ
180ご冗談でしょう?名無しさん:2006/11/23(木) 11:49:27 ID:???
>>167=177が答えもらえなくて暴れてるって結論でいい?
そうやって妄想イコールをするような人なら同じことやられても文句ないよね?
181ご冗談でしょう?名無しさん:2006/11/23(木) 11:52:36 ID:HNmq77ec
円電流から生じる磁場ですが,円に軸上の任意の点での磁場については様々な
参考書に出ていますし,理解も簡単なのですが,軸上を含めた一般的な場合
(任意の点)の磁場をきちんと式で計算して表している本がありましたら教えて
いただきたいです.お願いします.

本についての質問はスレ違いかな?
182ご冗談でしょう?名無しさん:2006/11/23(木) 11:55:23 ID:???
>>181
共立の詳解電磁気学演習
183ご冗談でしょう?名無しさん:2006/11/23(木) 12:01:00 ID:HNmq77ec
>>182
ありがとうございます.当たってみます.
184ご冗談でしょう?名無しさん:2006/11/23(木) 12:08:27 ID:???
>>180がアンカーの使い方覚えましたw

おめでとうw

で何方か
>>163の答えわかりますでしょうか?
185ご冗談でしょう?名無しさん:2006/11/23(木) 12:19:54 ID:???
検索しろよ
186ご冗談でしょう?名無しさん:2006/11/23(木) 12:31:29 ID:???
煽れば答えてもらえると思ってる典型的な厨房だな
187ご冗談でしょう?名無しさん:2006/11/23(木) 13:29:07 ID:???
紐の重さも考慮して、実体振り子の慣性モーメントを求めるときって
紐の慣性モーメントと球の慣性モーメントをそれぞれ足せばOKですか?
188ご冗談でしょう?名無しさん:2006/11/23(木) 13:56:24 ID:???
積分は足し算です
189ご冗談でしょう?名無しさん:2006/11/23(木) 14:06:24 ID:???
すみません。
波動で、モードを決定するn,m,lなどの整数って、なんて呼ぶんですか?
量子力学だと、量子数って呼びますが、数学的っていうか、一般的な呼び名はありますか?
190ご冗談でしょう?名無しさん:2006/11/23(木) 14:12:39 ID:???
>>189
次数かなぁ
191ご冗談でしょう?名無しさん:2006/11/23(木) 14:35:47 ID:???
>>190
固有値っていうのは違いますか?
192ご冗談でしょう?名無しさん:2006/11/23(木) 14:45:30 ID:???
それはちょっと許せないかな
193ご冗談でしょう?名無しさん:2006/11/23(木) 14:54:48 ID:???
というか全然違う
194ご冗談でしょう?名無しさん:2006/11/23(木) 15:51:40 ID:S/o75dYV
夕方になると空が赤くなるのは
昼間と比べて太陽光の通過する空気層の量が多く
青色の光が散乱しつくされて赤色しか残っていないためと
説明されますよね?
では地球の大気層がもうすこし大きかったらどうなるのでしょうか?
大きくなるにつれて赤色になり、次に赤外線のみになってきて
可視光がなくなり段々暗くなってくるのでしょうか?
反対に薄くなった場合にはどうなるのでしょうか?
紫外線の量が増えるのは当然でしょうけど
空の色も変化するのでしょうか?青色から段々白色に近い色になっていくのでしょうか?
195ご冗談でしょう?名無しさん:2006/11/23(木) 16:31:00 ID:???
>>194
方向性としては大体有ってる
薄くなった場合は月の映像を思い浮かべればわかりやすいと思う
散乱されないから真っ黒
196189:2006/11/23(木) 16:38:04 ID:???
わかりました。ありがとうございました。
197ご冗談でしょう?名無しさん:2006/11/23(木) 18:14:05 ID:ZBHzr79k
R.ペンローズの言う非計算論的な理論っていうのは具体的にどういうものなんだろうか?
198ご冗談でしょう?名無しさん:2006/11/23(木) 18:24:00 ID:???
Two-Resistanse-Theoryって日本語でいうと
どうなりますか??
199ご冗談でしょう?名無しさん:2006/11/23(木) 18:35:38 ID:???
自己解決しました
200ご冗談でしょう?名無しさん:2006/11/23(木) 19:44:55 ID:9TKXRxC9
すいません、光電効果について、どうかよろしくお願いします。

ぐぐったところ、光電効果も色々あるようですが、高校でやってる
 ttp://www12.plala.or.jp/ksp/quantum/photoelectric1/tomo-photoelectric-fig3.png
これについて教えてください。
金属板に光を当て、そこからエネルギーを得た電子が金属板を飛び出し、
それが電極に到達すると、電流として検知される↑コレです。

これについての考察を行う宿題で、二点ほど判りません。
どうか、どうか教えてください、お願いします。
2つ質問するのがダメなら、
そのうちの2つめ、後者のほうだけでも、どうかお願いします。

  1つめ 
ttp://www12.plala.or.jp/ksp/quantum/photoelectric1/tomo-photoelectric-fig6.png
光の強度を変えると、要するに、当る光子が増えるので、
飛び出る光電子の量が増える。(ただし光電子のエネルギーは変わらない)
よって、↑のように、単純に電流…タテ軸方向にグラフが大きくなります。

これに基づき、「光電管として望ましい特性について考察せよ」だそうです。
望ましいと言われても、光電管の一般的な用途が判らないので(ぐぐっても色々な物が…)
どうにも答えられません。

光強度に応じて、電流が変わるので、
仮に光電管の用途を「光を電流へと変換する物」に限定すれば、
少ない光(光強度:弱)で、大きな電流を取り出せれば、「望ましい」のかな、
と考えましたが…こういうことでしょうか?

つづきます
201ご冗談でしょう?名無しさん:2006/11/23(木) 19:48:29 ID:9TKXRxC9
 
 2つめ
こちらがいよいよ判りません。お願いします。
光電効果は、学校では、(歴史的には逆?ですが)
光の粒子としての性質を確認する実験として行っています。

しかし、それについて、
粒子としての光子は、(波動ではないので)「不連続に放出」されている。
しかし、光電流は、「連続的な電流」として測定される。
これは何故か、説明せよ。
例えば、光強度をギリギリまで下げた場合、不連続となるのか。考察せよ。
この時測定機の応答速度に望まれるものはなにか、考察せよ。

↑↑が設問です。

色々考えたんですが、どうにも判りません。
いちおうの推測だと、
 ・不連続であるが、非常に小さく・細かく・大量にあるので、
  電流計では連続的に「見える」。
 ・だから、光子1つ1つを検知できるほど、敏感で、かつ応答速度が早い
  電流計であれば、理論上は、あるいは「不連続な電流」を測定出来るかも?

と、考えたんですが、どうでしょうか。


どうか、どうかよろしくお願いします。
202ご冗談でしょう?名無しさん:2006/11/23(木) 20:11:48 ID:???
つ【高校スレ】
203ご冗談でしょう?名無しさん:2006/11/23(木) 20:31:37 ID:7U/sOVRe
>>202
あれ、このスレでは良くなかったですか…??
すいませんでした(´・ω・`)

一応、そちらにも、訊いてみます…ありがとうございました。
204ご冗談でしょう?名無しさん:2006/11/23(木) 22:07:01 ID:???
>>203
それだけ考えられれば十分
その考えを書いて提出すればいいと思うよ
205ご冗談でしょう?名無しさん:2006/11/23(木) 22:32:25 ID:+PpOtB9a
>>204
ID変わってしまいましたが、>>200-201です。

これで十分でしょうか。
よかったです。
本当にどうもありがとうございました。
206オオノ:2006/11/23(木) 22:44:41 ID:wUIS7kYr
はじめまして。
オオノと申します。
大学の課題で以下のような、エントロピーに関する問題が出されたのですが、
普段物理とは縁遠い授業を受けているため
内部エネルギーとエントロピーの関係をうまく理解できておらず
解くのに苦戦しています。

(問題1)
10℃と80℃の水1 グラムのエントロピーはどれだけ違うか。ただし,こ
の温度範囲で,1℃温度上昇させるのに必要な熱が4.2J であるとせよ。

(問題2)
圧力P で外部と釣り合っている気体の体積が(釣り合いをわずかにずら
して)dV だけ増加すると,気体はP dV の仕事を外部にして自らはその量の
エネルギーを失う。また同じ気体が温度T で外部と熱平衡にあるとき(温度
をわずかに変えて)熱dQ を吸収すると,気体のエネルギーが増える。
(可逆過程となっていることに注意せよ。)
仕事と熱をあわせ,気体の内部エネルギーに関する第一法則を書くと
dE=dQ - PdV
である。熱の出入りの部分をエントロピーの変化dS を用いて書き直せ。


どなたかもし宜しければアドバイスを頂けないでしょうか。
よろしくお願いします。
(この文章は他の掲示板にも書き込んでいるものです。
マルチポストになって申し訳ないのですが
なにぶん提出期限が近いため少し焦っています。
すみません。)
207ご冗談でしょう?名無しさん:2006/11/23(木) 22:52:07 ID:???
分かりませんと大きく書いて提出しろ
208ご冗談でしょう?名無しさん:2006/11/23(木) 23:20:58 ID:???
マルチだって明記するのはいいが、どこにマルチしたか書かないと、
黙ってマルチしたのと結果的に変わんないよなあ。
209ご冗談でしょう?名無しさん:2006/11/23(木) 23:21:22 ID:737qzVMt
709 ゆとり世代 2006/11/23(木) 23:00:20 ID:baHKFT810
おまえら考えたことないか?
絶対に曲がらないダイヤモンドでできた長さ1光年の棒があるとしよう
その棒の先に時計をくくりつけ、棒の付け根は地球にぶっ指す。
では先っちょにある時計はどれだけのスピードで動くか計算してみよう
地球を中心とした時計の起動距離=2光年x3.14=6.28光年
地球は1日に1回転するので、1日で6.28光年進むのである
なんと、時計は光速の365x6.28倍の速度で進むことになる
ではそのような速さで動く物体の時間の流れはどうなるか
波動係数を使って求めてみると、驚くことにその時計は過去へと進むのである
さてここで問題だ。その時計をくくりつける前の時間まで戻った場合
その時計はどうなるのか考えてみてくれ。
異論反論珍回答どうぞ。
210ご冗談でしょう?名無しさん:2006/11/23(木) 23:40:58 ID:???
211ご冗談でしょう?名無しさん:2006/11/24(金) 00:10:51 ID:1j+Xoz2N
>>209 とりあえず君がアインシュタインよりバカだってことは十分理解出来た。
そしてアインシュタインの偉い所は似たような考えから(勿論君の挙げた例
よりはましだが)自分自身の作った理論を批判的に検討し、更なる高みに達した
ことだ。もう少し先人の行跡をたどり、自省、って言葉を覚えよう。
212ご冗談でしょう?名無しさん:2006/11/24(金) 00:14:54 ID:???
>>209
こういう人って、原始がボールみたいなものだと思ってるんだろうな。
213ご冗談でしょう?名無しさん:2006/11/24(金) 01:19:25 ID:cOy8Ol8w
>>195
真っ黒になるのですか?
ふと気づいたのですが可視光以下の波長を全て吸収出来るということは
ダークマターになり得るということなのでしょうか?
214ご冗談でしょう?名無しさん:2006/11/24(金) 01:22:09 ID:???
>>213
ただの板おいても光を遮ることは出来るが
板はダークマターか?
215ご冗談でしょう?名無しさん:2006/11/24(金) 01:34:17 ID:???
「正孔が移動する」って
電子の足りない物体が空間を移動するのですか?
それとも
電子の足りない状態の場所が変わるだけですか?

216ご冗談でしょう?名無しさん:2006/11/24(金) 01:46:02 ID:???
普通の金属なら後者
217ご冗談でしょう?名無しさん:2006/11/24(金) 01:46:14 ID:XTAxQv/0
>>215
後者
218ご冗談でしょう?名無しさん:2006/11/24(金) 01:53:02 ID:XTAxQv/0
>>201
電流A(C/s)と検出器の不感時間の問題ということか
219ご冗談でしょう?名無しさん:2006/11/24(金) 02:29:25 ID:cOy8Ol8w
>>214
板は全ての波長の光を吸収出来るわけではないでしょう?
もし全ての波長の光を吸収できるほど
大気層が厚ければ観測することは不可能になると思うのですが。

220ご冗談でしょう?名無しさん:2006/11/24(金) 02:34:34 ID:???
もし全ての波長の光を吸収できるほど
板が厚ければ観測することは不可能になると思うのですが。

ともいえないか?

どっちも「反対側から光が通過してこない」
ってだけで全然ダークマターじゃないがな
221ご冗談でしょう?名無しさん:2006/11/24(金) 02:36:07 ID:???
>>219

大丈夫か?
「真っ黒になる」というのは「大気層が厚いと光を吸収するから真っ黒」という話だったか?

それに「ダークマター」は「全ての光を吸収」するわけではない。単に見えないだけだ
222ご冗談でしょう?名無しさん:2006/11/24(金) 03:08:47 ID:rBqC4G94
コウモリが出す周波数は、どんくらいですか? 誰か教えて下さい。
223ご冗談でしょう?名無しさん:2006/11/24(金) 03:11:38 ID:???
224ご冗談でしょう?名無しさん:2006/11/24(金) 03:16:54 ID:???
http://www.amazon.co.jp/gp/product/4480090061/sr=1-1/qid=1164305208/ref=sr_1_1/250-8289172-6173045?ie=UTF8&s=books
ペンローズの“量子脳”理論―心と意識の科学的基礎をもとめて

この本の原著の題名って分かりますか?
http://www.amazon.co.jp/gp/product/0679776311/sr=1-1/qid=1164305457/ref=sr_1_1/250-8289172-6173045?ie=UTF8&s=english-books
これのことなんでしょうか?

225ご冗談でしょう?名無しさん:2006/11/24(金) 03:22:31 ID:???
分からんけどタイトルが全然違うから別物じゃないか?
226224:2006/11/24(金) 03:31:30 ID:???
でもこれ以外に該当するほんがAmazonで
見あたらないんですよ。
227ご冗談でしょう?名無しさん:2006/11/24(金) 03:48:44 ID:???
Amazonに有るとは限らないだろ
228ご冗談でしょう?名無しさん:2006/11/24(金) 03:56:36 ID:???
229ご冗談でしょう?名無しさん:2006/11/24(金) 05:09:59 ID:TLcZzB4j
すみません
F=maってファックはマンコとアナルでやれって意味じゃないですか
でもマンコとアナルをかけるって所がよく分からないんです
誰か教えて頂けませんか?
230ご冗談でしょう?名無しさん:2006/11/24(金) 06:37:06 ID:???
ID:TLcZzB4j
231ご冗談でしょう?名無しさん:2006/11/24(金) 07:59:25 ID:???
>>224
これ日本独自企画で,一冊の原著ってのはなかったような。
232ご冗談でしょう?名無しさん:2006/11/24(金) 08:08:48 ID:M7sOXnHE
物理的に彼氏はどうやってできますか?方程式を教えてください。
233ご冗談でしょう?名無しさん:2006/11/24(金) 10:12:04 ID:???
>>229,232
 おまえの話はつまらん。
234ご冗談でしょう?名無しさん:2006/11/24(金) 12:39:19 ID:cOy8Ol8w
火花の正体って何なのでしょうか?
このまえ青色の火花の出るライターを見たのですが
もしかして炎色反応のように物質の種類によって
色が異なるのでしょうか?
235ご冗談でしょう?名無しさん:2006/11/24(金) 15:21:52 ID:XTAxQv/0
>>234
おそらく炎色反応
236ご冗談でしょう?名無しさん:2006/11/24(金) 16:25:26 ID:DXl4m5aQ
電子の軌道ってどうして
1p→2s→2p→3s→3p→4s→3d→4p→5s→4d
ってなるんですか?
1p軌道とか1d軌道ってないのですか?
237ご冗談でしょう?名無しさん:2006/11/24(金) 17:49:04 ID:???
>>236
ない

正確に知りたければ、
量子力学を学べ
238ご冗談でしょう?名無しさん:2006/11/24(金) 18:46:51 ID:???
>>236
お前が1pから始めてるじゃねえかw
239ご冗談でしょう?名無しさん:2006/11/24(金) 18:58:11 ID:???
>>236
俺はもっぱら3Pだな
240ご冗談でしょう?名無しさん:2006/11/24(金) 19:28:41 ID:???
俺はもっぱら1Pだな
241ご冗談でしょう?名無しさん:2006/11/24(金) 19:52:16 ID:???
>>235
本当に炎色反応なのでしょうか?
例えば電気火花は絶対に黄色のような気がしませんか?
銅の炎色反応の色は淡青色ですけど、どう解釈すれば良いのでしょうか?
それとあれは高電圧になった電極或いは配線の一部が燃焼しながら
飛散する現象ということなのでしょうか?
とすると火花を散らし続けるとその物質の重量は小さくなっていくということなのでしょうか?
242ご冗談でしょう?名無しさん:2006/11/24(金) 20:23:32 ID:???
電気なら炎色反応じゃないな
ちなみに絶対に黄色ではない

夏に外出て雷でも見てこい
243ご冗談でしょう?名無しさん:2006/11/24(金) 20:27:40 ID:???
炎色反応もあるだろうし、熱輻射もあるだろう
244ご冗談でしょう?名無しさん:2006/11/24(金) 20:27:49 ID:???
酸素色?
245ご冗談でしょう?名無しさん:2006/11/24(金) 21:16:52 ID:1npG9Jiy
>>235だけど

>>234
ごめん花火とまちがえた
火花ね

炎色反応もあるとおもうし
励起される外殻電子のエネルギー順位幅
により可視光領域の波長が微妙に違うのかも
細かくはしりません あくまで推測
246ご冗談でしょう?名無しさん:2006/11/24(金) 21:21:12 ID:1j+Xoz2N
>>236 pとかdっていうのは自由度が沢山有る。前者は3、後者は5。で、
不確定性原理によると、あまり小さい空間領域(主量子数nが小さい程、波動関数の
空間的拡がりが小さい)に沢山の独立な波動関数は存在し得ない。というわけで、
角運動量の大きな波動関数はある程度拡がった、主量子数の大きなものになって初めて
現れる。
 ちなみに、人工的だが、半径がそれなりの大きさの、ある厚さの球殻において
-Vとなるような類いのポテンシャルを考えれば、1p、1d何てのもあり得る。
247ご冗談でしょう?名無しさん:2006/11/24(金) 23:22:00 ID:vbOOcOD6
質問です。
地上1000mから10kgの物体が落下した時の衝撃ってどのくらいなのか計算して下さい。

http://www.ntv.co.jp/FERC/research/20000618/r054.html
248ご冗談でしょう?名無しさん:2006/11/24(金) 23:25:48 ID:vbOOcOD6
計算したら、以下のスレに来てどれくらい凄い衝撃力かを書き込んで下さい


http://science4.2ch.net/test/read.cgi/wild/1154693524/
249ご冗談でしょう?名無しさん:2006/11/24(金) 23:28:09 ID:???
250ご冗談でしょう?名無しさん:2006/11/25(土) 02:36:50 ID:???
見にいったら、熊パンチの強さの話じゃん(w 物理板まで応援求めに来るかよ
べつに10Kgじゃなくても、熊パンチで地上1000mまで物を飛ばせるか考えてみた?
251ご冗談でしょう?名無しさん:2006/11/25(土) 05:53:18 ID:???
キラーン、と☆になるお
252ご冗談でしょう?名無しさん:2006/11/25(土) 07:23:55 ID:ijk/5obU
>>250
シャチの尾ビレによるキックは、体重100kg以上のアシカを
海面上10m吹き飛ばすそうです。
どっちが凄いのですか?
253ご冗談でしょう?名無しさん:2006/11/25(土) 07:42:31 ID:???
ID:ijk/5obU
ID:vbOOcOD6
254ご冗談でしょう?名無しさん:2006/11/25(土) 12:22:48 ID:s5yD9y38
なんで蛍すぐ死んでしまうん?
255ご冗談でしょう?名無しさん:2006/11/25(土) 12:32:58 ID:cL2JcS/3
つ生物板
256ご冗談でしょう?名無しさん:2006/11/25(土) 18:43:35 ID:???
エロゲやってたら可視光のレーザーを物(鏡ではない)に当てて反射してきた光を観測することで距離を測る機械が出てきたんだけど
そんなもん無いよね?
257ご冗談でしょう?名無しさん:2006/11/25(土) 18:46:37 ID:???
>>256
道端で使ってるの見たことないの?
258ご冗談でしょう?名無しさん:2006/11/25(土) 18:53:29 ID:???
259ご冗談でしょう?名無しさん:2006/11/25(土) 18:56:29 ID:???
>>257
それは可視光じゃない
>>258
それは可視光なの?
260ご冗談でしょう?名無しさん:2006/11/25(土) 19:00:23 ID:???
>>259
これは可視光らしいが。
ttp://www.kouguya.com/a2.html
261ご冗談でしょう?名無しさん:2006/11/25(土) 20:00:50 ID:???
なあ、物理学に明るい人ちょっと来て一言書き込んで行ってくれないか?
野生生物板はバカばっかりで困ってる

http://science4.2ch.net/test/read.cgi/wild/1154693524/
262ご冗談でしょう?名無しさん:2006/11/25(土) 20:54:42 ID:???
>>259

ライカの可視レーザー測距計なんぞは道端(っつうか工事関連というか)
で使ってたりもするぞ。
263ご冗談でしょう?名無しさん:2006/11/25(土) 20:58:51 ID:???
>>256
>>1

>http://www.google.com/ などの検索サイトを利用し、各自で調べること
264ご冗談でしょう?名無しさん:2006/11/25(土) 21:44:02 ID:ijk/5obU
この板から変な奴が野生生物板に来たぞ
どうにかしろ
265ご冗談でしょう?名無しさん:2006/11/25(土) 22:27:29 ID:???
ほんとだ。いきなり代表を名乗っててキモい。南無。
266ご冗談でしょう?名無しさん:2006/11/25(土) 22:45:05 ID:???
本当に申し訳ないけどそっちで始末してくれ
267ご冗談でしょう?名無しさん:2006/11/25(土) 23:08:34 ID:SsnrRLS7
物理実験で球面の曲率半径を求める実験をしました
実験書に載っている導出過程でR=a^2/6h+h/2≒a^2/6hってなってるのですが
これはh/2がa^2/6hに比べて非常に小さな値になるから無視できると考えていいからですか?
実験だから正確なほうがいいような気もするのですが・・・
ちなみにa=65.25,h=3.0799です
268ご冗談でしょう?名無しさん:2006/11/25(土) 23:11:12 ID:/pLkOS9n
無視できる
269ご冗談でしょう?名無しさん:2006/11/25(土) 23:26:09 ID:???
>>267
無視できるかどうかは君が決めることだ。いや、ホントに。
270ご冗談でしょう?名無しさん:2006/11/25(土) 23:31:00 ID:???
>>267
 実験なんだから出てくる数値には有効数字というもんがある。
 第2項を足すことによる変化が、第1項の有効数字から外れた部分なら、
足す意味はない。

 「日本の人口、だいたい1億人」と言った人がいたとして、それに「いや、昨日
うちに息子が生まれたからだいたい1億1人になった」と言ってもナンセンス
だということだ。
271ご冗談でしょう?名無しさん:2006/11/25(土) 23:57:53 ID:ijk/5obU
今晩は。野生生物板から来ました。
今、野生生物板の某スレでは熊の腕力についての意見が割れています。
「熊のパンチは1000m上空から10kgの鉄球が落下したに等しいエネルギー」とか
「熊のパンチ一撃で陸上の動物殆どが即死する」とか
「熊の咬合筋は虎の三倍」とか
「ホッキョクグマは体重7tのアフリカゾウに勝る」
などと馬鹿げた議論がかわされています。
物理的に説得力のあるとどめをどなたかお願いします
272ご冗談でしょう?名無しさん:2006/11/26(日) 00:00:31 ID:wfSxdNjb
わろた
273ご冗談でしょう?名無しさん:2006/11/26(日) 00:00:43 ID:???
今晩は。
274ご冗談でしょう?名無しさん:2006/11/26(日) 00:07:03 ID:???
見聞を広めるためにも
協力してやっていいんじゃね?
275ご冗談でしょう?名無しさん:2006/11/26(日) 00:08:59 ID:sDf55PMS
>>274
同意。
今見てきたけど、かなり低レベルな事で議論してるみたいw
276ご冗談でしょう?名無しさん:2006/11/26(日) 00:14:10 ID:???
起電力法を用いて、相互インピーダンスを導出する過程で行き詰っています。
解に指数積分が出てくるのですが、どうも式変形がうまくいきません。
どなたかご教授or向いてる板に誘導お願いしますorz
277ご冗談でしょう?名無しさん:2006/11/26(日) 00:16:09 ID:???
数学板いけ
あそこは丸投げOKだから
278ご冗談でしょう?名無しさん:2006/11/26(日) 00:20:16 ID:???
数学板で、物理板にいけと言われましたw
279ご冗談でしょう?名無しさん:2006/11/26(日) 00:28:04 ID:???
>>278

んじゃ、マルチは死んどけ。
280ご冗談でしょう?名無しさん:2006/11/26(日) 00:43:18 ID:???
>>276
それで完了なんじゃない?
Green関数の6重積分になってるんだと思うが。
普通、モーメント法ではそれをガウス積分して解くよ。
281ご冗談でしょう?名無しさん:2006/11/26(日) 01:14:53 ID:k4jz3lBb
教えて下さい よく花火とかで一万発とか打ち上げやりますがその花火の発する音
が聞こえなくなるには花火の打ち上げ場所からおよそ何キロ離れる必要がありますか?
まあ花火が何メートル上空で破裂するかという条件とかで変わってきますがおおよそ
の事が知りたいです よろしくお願いします
282ご冗談でしょう?名無しさん:2006/11/26(日) 01:22:56 ID:k4jz3lBb
訂正します 上空の破裂音じゃなくて花火の打ち上げ時のあのドンという重低音
が聞こえなくなるまで離れるには何キロ距離が必要ですか?
ご教授お願いします
283ご冗談でしょう?名無しさん:2006/11/26(日) 01:23:31 ID:???
>>281
そもそも人や静けさによるわな。
284ご冗談でしょう?名無しさん:2006/11/26(日) 01:33:20 ID:???
> おおよその事が知りたいです

ここが

「おおよその考え方をお聞きしたいです」
「参考になる数式や参考文献があったら教えてください」

となっていたら答える用意があった。
285ご冗談でしょう?名無しさん:2006/11/26(日) 01:45:29 ID:???
「3次元における仕事」を理科の嫌いな中学生でもわかるように説明出来る人いないかな?
高3の頭あまりよくない(俺もよくないんだけど・・・)奴のカテキョしててなんとか理解させてやりたいんだけど
286ご冗談でしょう?名無しさん:2006/11/26(日) 01:47:40 ID:???
∫F*ds
(*はスカラー積を表す。)
これが「仕事」というものの定義。後は計算すれば良い。
287ご冗談でしょう?名無しさん:2006/11/26(日) 01:54:20 ID:???
とりあえず1次元と2次元をしっかり理解させることから始めよう
3次元で急に分からなくなると言うことはそれ以前からちゃんと分かってない
288ご冗談でしょう?名無しさん:2006/11/26(日) 01:58:59 ID:???
>>286
レスサンクス
3次元における仕事と運動エネルギーを理解させる簡単な実験ってなんか思いつく?
できれば日常生活との関連して・・・

289ご冗談でしょう?名無しさん:2006/11/26(日) 02:02:02 ID:???
>>287
まあ普通に考えてそうなんだが受験前らしくて手っ取り早く教えてやれねーかなと思ってさ
290ご冗談でしょう?名無しさん:2006/11/26(日) 02:09:08 ID:???
つ 急がば回れ
291ご冗談でしょう?名無しさん:2006/11/26(日) 02:22:26 ID:???
いきなり3次元は
公式丸暗記させて定形問題解かせる
と言うこと以外無理

頭の良い奴なら1次元聞いたら3次元も何となくわかるかもしれんが
292ご冗談でしょう?名無しさん:2006/11/26(日) 03:10:01 ID:iVbz2o78
静電気についてなんですが、
なぜ物質には正に耐電しやすいものと負に帯電しやすいものが
あるのですか?
自分なりに調べたり、考えてみたのですがどうしても分かりません。
どなたか教えてもらえませんか?
293ご冗談でしょう?名無しさん:2006/11/26(日) 03:35:37 ID:???
>>292
とりあえず学年と物理の履修状況を教えてくれないか?
294ご冗談でしょう?名無しさん:2006/11/26(日) 03:40:23 ID:???
>>293
高校2年の女子です。物理は初めてなのでやさしくおねがいします
295ご冗談でしょう?名無しさん:2006/11/26(日) 09:55:53 ID:???
ちなみに女です
296ご冗談でしょう?名無しさん:2006/11/26(日) 10:27:44 ID:???
ネットコミュニティにおいて必要もなく女と明かした場合は乳を晒さなければならないんだよ。
297ご冗談でしょう?名無しさん:2006/11/26(日) 10:57:24 ID:zShGr5wO
sinθ≒θを満たす振り子の最大揺れ角θをもとめ
振り子の長さを1mとしてそのときの振れ幅を計算せよ

この問題はどうやって考えればいいのですか。
できればθの値も教えてください
298ご冗談でしょう?名無しさん:2006/11/26(日) 11:00:54 ID:???
299ご冗談でしょう?名無しさん:2006/11/26(日) 11:04:06 ID:LR949/F7
300ご冗談でしょう?名無しさん:2006/11/26(日) 11:21:12 ID:zShGr5wO
sinθ≒θを満たす振り子の最大揺れ角θをもとめ
振り子の長さを1mとしてそのときの振れ幅を計算せよ

この問題はどうやって考えればいいのですか。
できればθの値も教えてください


301ご冗談でしょう?名無しさん:2006/11/26(日) 11:23:56 ID:???
>>300
有効数字によるだろ。
302ご冗談でしょう?名無しさん:2006/11/26(日) 11:49:11 ID:dJmsyW6N
レスありがとうございます
有効数字4桁です
303ご冗談でしょう?名無しさん:2006/11/26(日) 11:52:19 ID:???
何で条件後出しなんだよ。死ね。
質量などの他の数値が十分精度があるとすると、
θとsinθの値の表から有効数字4桁で計算して結果が合うところはどこか分かる。
もっと厳密にやりたいなら楕円積分とかなんとかも利用したり。
304292:2006/11/26(日) 12:54:15 ID:iVbz2o78
>>293
化学科の院生です。
ふつうに専門用語を使って説明してくれた方が助かります。
分からない単語などは後で自分で調べますので
ちゃんとしたことを教えてもらいたいのですが。
或いはどの本を見れば詳しく載っているなどということを
教えて下さっても構いません。
どうかよろしくお願い致します。

なお>>294-295は私ではありませんので。悪しからず。
305ご冗談でしょう?名無しさん:2006/11/26(日) 12:59:15 ID:???
>>304
えー化学の問題なんじゃないの?

あと問題にしてるサイズによって回答変わるよね
分子の集合体なのか1分子レベルかで
306ご冗談でしょう?名無しさん:2006/11/26(日) 13:08:26 ID:iVbz2o78
>>305
もしかすると化学の分野なのかも知れませんが
私は習ったこともありませんし、周りにこれに詳しい方もいらっしゃいません。
分からないのですが1分子レベルのことを議論することで
バルクとしての議論も可能になるのではないのでしょうか?
307ご冗談でしょう?名無しさん:2006/11/26(日) 13:20:22 ID:???
院生を名乗るにしては質問が大雑把過ぎて
こっちも答えにくい
308ご冗談でしょう?名無しさん:2006/11/26(日) 13:22:18 ID:???
309ご冗談でしょう?名無しさん:2006/11/26(日) 13:28:01 ID:iVbz2o78
>>307
すいません、わざと大雑把な質問の仕方をしております。
具体的なことを聞き方も可能ですがたぶん回答者側は
余計に混乱すると思ってこういう質問の仕方をしております。

もう少し詳しく書きますと
http://www.watanabe-electric.co.jp/seidenki/index.asp
のページのようになぜ、アスベスト、人毛はプラスの帯電しやすくて
テフロン、塩化ビニールなどは負に帯電しやすいのかということが知りたいのです。

310ご冗談でしょう?名無しさん:2006/11/26(日) 15:31:00 ID:???
>>309
そのページの下のほうに説明があるみたいだが・・・
311ご冗談でしょう?名無しさん:2006/11/26(日) 17:16:20 ID:cAmnfBLB
相対性理論で単純な速度の合成則が使えないて聞いたんだけど、じゃどんな合成測が使えるの?
詳しい人おしえて。
312ご冗談でしょう?名無しさん:2006/11/26(日) 17:21:11 ID:???
(u+v)/(1+(uv/c^2))
とかそんな感じ
313ご冗談でしょう?名無しさん:2006/11/26(日) 17:24:16 ID:cAmnfBLB

なるほど。それと光速一定てのはどんなかんけいが?
314ご冗談でしょう?名無しさん:2006/11/26(日) 17:28:00 ID:???
u か v のどちらかを c と置いて計算してみれば?
315ご冗談でしょう?名無しさん:2006/11/26(日) 17:51:27 ID:???
>>311
とりあえず特殊で良いから相対論の本読んでこい
式が載ってる奴で簡単めの物ならわかりやすいはず
316ご冗談でしょう?名無しさん:2006/11/26(日) 18:05:35 ID:wPYp4XC1
少し質問があるのですが、以前見た中学生用の問題集に次のような問題がありました。

なめらかな斜辺をすべり落ちそのままなめらかな平面上を運動する物体について
(1)平面上での運動をなんというか。
(2)このとき力は働いているか。

確かこんな感じだったと思うのですが、(2)で答えは働いてないと書いてありました。
実際には重力と垂直抗力が働いて、それが釣り合ってる為合力が0になっているだけで、この答えではまずいかと思うのですが。。

知り合いに問題をだしてみたところ(物理科の人にも)、皆力は働いてないと答えたのですがこの答えは問題ないのでしょうか?
317ご冗談でしょう?名無しさん:2006/11/26(日) 18:07:37 ID:???
力ってのは一般に合力のことだよ
318292:2006/11/26(日) 19:00:21 ID:iVbz2o78
どうかよろしくお願い致します。
319ご冗談でしょう?名無しさん:2006/11/26(日) 19:07:24 ID:???
>>318
俺の推論だと
分子が1価になるために必要なエネルギーと
−1価になるために必要なエネルギーとがあって

より1価になりやすい奴が1価になって、もう一方は−1価になる
何故そういう区分が出来るかというと軌道を考えれば分かるんじゃないかな?
イメージとしては高校で習うイオン化傾向と同じ気がする

勘だけど
320ご冗談でしょう?名無しさん:2006/11/26(日) 19:23:23 ID:???
専門は電子ですが、物性なのでこちらの方が適切っぽいので、
電気電子板から移動してきました。

私今、研究で薄膜半導体を作製しています。
そこで、基板に石英ガラスを用いているのですが、
20度〜30度付近に大きな回折ピークが見られるのです。
これが石英ガラスによるバックグラウンドのピークだと言うのは解るのですが、
何故このようなピークが観測されるのでしょうか。
ここでよいのか解りませんが、解る方いましたらよろしくお願いします。
321ご冗談でしょう?名無しさん:2006/11/26(日) 19:24:59 ID:???
>>320
角度までは良く分からんけど、
ボソンピークとか石英ガラスの中の三角錐構造の部分からの反射とか、
局所ブラッグ平面とか
そういうのが関係してるのかな。
322ご冗談でしょう?名無しさん:2006/11/26(日) 19:26:49 ID:???
石英は単結晶とか多結晶じゃないから鋭い回折ピークは出ないけど
有る程度サイズがきまってて適当に並んでるもんだから
その辺にぼんやりとピークが出来る
323ご冗談でしょう?名無しさん:2006/11/26(日) 19:29:41 ID:???
>>321
即レスありがとうございますー。
しかし専門ではなく、勉強しはじめ段階なのであまり専門的な事はわかりません・・
ちとその辺りの用語を調べて見ます。

>>322
えっと、石英ガラスはアモルファスだから強い回折ピークはでないけども、
アモルファスのわりにはある程度配向性があるから、
鋭くはないが、うっすらとしたピークが観測される。

って感じですかね? 
324322:2006/11/26(日) 19:32:38 ID:???
>>323
別に専門じゃなくてこれからそんな関係ないなら
そのぐらいの理解で良いかと

または教授とか回折やってる研究室とかの友人に聞きくのが良いと思う
325320:2006/11/26(日) 19:38:43 ID:???
なるほど。
その辺りで納得しておきますー。
もう少し詳しくなってきたらまた考えて見ます。
ありがとうございましたー。
326ご冗談でしょう?名無しさん:2006/11/26(日) 23:32:17 ID:wPYp4XC1
>>317
レスどうもです。
力が働いてないと釣り合ってるってのは(少なくとも物理においては)同義ってことでいいんですね。
327ご冗談でしょう?名無しさん:2006/11/26(日) 23:32:39 ID:???
3原子分子の内部エネルギーはどのように考えればいいんでしょうか?
328ご冗談でしょう?名無しさん:2006/11/26(日) 23:34:28 ID:???
>>326
ていうかむしろ、あんまそんな事区別して考えてない。
どうでも良い話だし。力がかかってると考えようが実質力がかかってないと考えようが。
出る結果は同じ。だから普通物理に進んでいる人間は、
運動の記述がしやすい「力がかかっていない」という表現を用いるというだけ。
なので、物理の人間の感覚と、正式な用語の定義は違う可能性が有るから、
自分で調べて。
329ご冗談でしょう?名無しさん:2006/11/27(月) 01:16:47 ID:???
外部からエネルギーをまったく与えずに
物体が突然二つに分裂して反対方向にふっとんでいくようなことってありますか?
330ご冗談でしょう?名無しさん:2006/11/27(月) 01:17:52 ID:???
時限式爆薬とか。
331ご冗談でしょう?名無しさん:2006/11/27(月) 02:46:01 ID:???
>>329
ブラックホールの潮汐力による長い物体の分離が頭に浮かんだ
332ご冗談でしょう?名無しさん:2006/11/27(月) 04:16:04 ID:???
>>329
核分裂
333ご冗談でしょう?名無しさん:2006/11/27(月) 04:28:43 ID:MIm4XsNQ
透過率・吸収度を測定したので検索してたら、
光学的特性って言葉が出てきたんですけど
これって何ですか???
どこを探しても説明が見当たらないんです。。。
334ご冗談でしょう?名無しさん:2006/11/27(月) 05:51:52 ID:???
光に対する特性。
例えば、「この物質の光学的特性は、透過率は○○で表面反射率は〜」
みたいに使う。
335ご冗談でしょう?名無しさん:2006/11/27(月) 07:50:15 ID:???
>>326
俺は「重力と垂直抗力が加わってるやんけ」と答えるけどな。
336ご冗談でしょう?名無しさん:2006/11/27(月) 09:34:33 ID:???
>>335
合力がゼロを示せば、それも正解
337ご冗談でしょう?名無しさん:2006/11/27(月) 10:05:37 ID:???
>>316
単に力というと普通は外力のこと
内力は別の問題
(物体を任意に分割して考えれば
一般に内力が働いてないわけがないし)
338ご冗談でしょう?名無しさん:2006/11/27(月) 12:08:08 ID:v2Tf4VZX
エントロピーを捨てると廃熱が発生する。
その廃熱を利用する術は無いのでしょうか?
無理なら何故なのでしょうか?

もし可能ならラプラスの悪魔辺りが召喚出来そうな気がします。
観測→データ入力→プログラミング(分子の選別)に従って入力されたデータから動作→データ削除→廃熱と高エネルギーの気体が持つ熱を使って観測・動作・削除・・・

だめ?
339ご冗談でしょう?名無しさん:2006/11/27(月) 12:42:39 ID:OpXZEN5W
>>337
重力と垂直抗力は外力じゃないかな?
地球と物体を一つとして考えてるわけではないので。。
340ご冗談でしょう?名無しさん:2006/11/27(月) 17:15:09 ID:KtAtn6at
ちょっとスレ違いかもしれませんが、
ネルソンの確率力学について易しくかつ詳しく書いてある参考書・教科書知りませんか?
具体的には、ネルソンが確率力学を導入しようと考え、実際の物理法則に適用させようとする、
その直前までの数学的な解説が学部の人間でもわかりやすく書いてある本が知りたいです。
(フォッカー・プランク方程式や、平均前方微分・平均後方微分などの考え方・またその数学的性質や、
 それが数学的に他の量子力学の考え方と同等である、という証明など)

保江邦夫先生の「Excelで学ぶ量子力学」、「数理物理学方法序説3 量子力学」は読んでいますが、
イマイチよくわかりません。特に後者はまるでわかりません。
こういう場合は、確率力学などの物理学の本よりもむしろ、数学の確率論を独自に勉強する方がいいのでしょうか…
341ご冗談でしょう?名無しさん:2006/11/27(月) 20:12:52 ID:???
>>338
発電するなり、暖房として利用するなり、ご自由に。
342ご冗談でしょう?名無しさん:2006/11/27(月) 22:01:48 ID:U+yJ6xu9
質問です
アルミニウムのプラズモンのエネルギーを求めたいんですけど,
アルミニウムの具体的な電荷密度が分かりません
金属はだいたいが10^29[m-3]ってことは知ってるんですが...
どなたか教えてくださいm(_ _)m
343ご冗談でしょう?名無しさん:2006/11/27(月) 22:02:46 ID:???
体積分かってるんだから電荷の数分かれば良いじゃん
344ご冗談でしょう?名無しさん:2006/11/27(月) 22:48:59 ID:???
LC回路を発振させると電磁波が放射されますが、LC回路の電流の、電磁波の放射に伴う
減衰の項はどういう形で微分方程式に含まれるのでしょう。
345342:2006/11/27(月) 22:55:01 ID:U+yJ6xu9
すみませんまだよく分からないんですけど,
アルミニウムは
モル体積 : 10.00 ×10-3 m3/mol
電子殻 : 2,8,3
なので,単位体積あたりの自由電子の数は,
(3×6.02×10^23)/(10.00 ×10-3)=1.8×10^26
でいいんでしょうか?
346ご冗談でしょう?名無しさん:2006/11/27(月) 23:49:25 ID:H9rHtHEn
物理学科ってやはり旧帝でないと入学する意味ないですか?
347ご冗談でしょう?名無しさん:2006/11/27(月) 23:51:40 ID:???
>>346
お前の言う“意味”とはなんだ?
348ご冗談でしょう?名無しさん:2006/11/27(月) 23:51:59 ID:???
>>346
やる気さえあれば院から東大とかいけるし問題ないよ
349ご冗談でしょう?名無しさん:2006/11/28(火) 00:24:24 ID:???
>>344
等価回路だけで解く場合には、放射抵抗ってのを
加味しないといけない。
波動方程式から解く場合には、減衰項を含む形に
すればいいんじゃないかな。
350ご冗談でしょう?名無しさん:2006/11/28(火) 00:59:01 ID:???
>>340
なんか危なっかしいな

普通の確率過程の知識 + 普通の量子力学の知識
があればネルソンは追えると思うが
351ご冗談でしょう?名無しさん:2006/11/28(火) 02:12:56 ID:xMbylxts
音と距離に関する関係式知ってる人は教えて下さい 
例えばある音源に対して距離が二倍になると音の大きさは何分の一になるとか
352ご冗談でしょう?名無しさん:2006/11/28(火) 04:26:47 ID:???
>>351
点音源なら球面波になるはずだから、音波の強度は距離の2乗に反比例すると思いますけど。
353ご冗談でしょう?名無しさん:2006/11/28(火) 04:33:38 ID:???
>>349
なるほど。また時間の取れるときに詳しい文献を漁ってみます。
現象としては当然起こることなのに、普通の教科書の類にはほとんど触れられていなくて。
ご回答ありがとうございます。
354ご冗談でしょう?名無しさん:2006/11/28(火) 05:58:45 ID:???
ローレンツ力F↑って
F↑=q(E↑+v↑×B↑)
なのか、それとも
F↑=q(v↑×B↑)
なのかどちらなのでしょう。
本によって違ったりするから、一体どちらのことをさしているのか・・・。
355ご冗談でしょう?名無しさん:2006/11/28(火) 05:59:31 ID:???
>>346
ないよ。全然ない。だからやめとけ。お前には無理だ
356ご冗談でしょう?名無しさん:2006/11/28(火) 06:03:44 ID:GDkHQiMc
皆さん難しそうな話してるとこ悪いんですが、射精した時に4M近い天井まで届くということはありえますか?
さきほど何日かぶりにオナニーしたら飛んだと思うのですが、その後すぐうたた寝してしまったので
ホントだったのか自信がありません。
357ご冗談でしょう?名無しさん:2006/11/28(火) 06:05:07 ID:???
ありえない
358ご冗談でしょう?名無しさん:2006/11/28(火) 07:08:00 ID:???
4mって結構高い天井のとこに住んでるんだな
359ご冗談でしょう?名無しさん:2006/11/28(火) 12:15:14 ID:1UAoxWe7
4mは無理だな。三日くらい溜めてると上向きでピュシュって1mくらい飛んだりするけど
360ご冗談でしょう?名無しさん:2006/11/28(火) 12:27:16 ID:???
>>354
どっちの意味でも使われる。漏れの印象では、高校まではqv×B、
大学以降はq(E+v×B)で定義したものが多い。いずれにしても、
本によって違うとわかっているなら、本ごとにどっちの意味で
使われているのか確かめてから読み進めればいいのでは?
361ご冗談でしょう?名無しさん:2006/11/28(火) 13:24:40 ID:???
>>360
なるほど。どちらもローレンツ力と呼ぶわけですね。
以前はq(v↑×B↑)のことだと思っていて、とある本にq(E↑+v↑×B↑)がローレンツ力だと
定義してあるのを見て、「あれ?」と思ったことがありました。

ちゃんと定義してあれば混同しないけど、統一してほしい・・・。
362ご冗談でしょう?名無しさん:2006/11/28(火) 13:53:05 ID:???
これしきの事でそんなことを言ってたら磁場の定義とか磁化の定義とかでも頭を悩ますぞ。
363ご冗談でしょう?名無しさん:2006/11/28(火) 13:55:19 ID:???
直径Lmmmのボルトを一定のトルクで回した時のボルトの
進む(押し出す)力を計算してください。
364ご冗談でしょう?名無しさん:2006/11/28(火) 14:42:30 ID:???
窒素やネオン、リンの基底状態のスペクトル項を求めると、最外殻が2pや3pなのでL=Pになると思うのですが、
表を見てそれぞれのスペクトル項を確認してみるとなぜかSとなっていて悩んでいます。なぜこうなるのでしょうか?よろしくお願いします。
365ご冗談でしょう?名無しさん:2006/11/28(火) 17:56:32 ID:???
>>362
単位系の違いもなー
366ご冗談でしょう?名無しさん:2006/11/28(火) 18:21:01 ID:???
>>285
おまえ日大だろ
367ご冗談でしょう?名無しさん:2006/11/28(火) 18:44:40 ID:???
>>362
磁場の定義って
F↑=qv↑×μH↑
と力と関連付けて定義されるもの?
それともマクスウェルの方程式から定義されるべきもの?

そういえば、電磁気学の講義で先生がマクスウェルの方程式はゲージ変換で不変であるような
方程式の最も簡単なものだろうとかなんとか言ってたな。

>>365
単位系はいまのところなんとかなってる。いざとなれば理科年表を見るし。
368ご冗談でしょう?名無しさん:2006/11/28(火) 19:55:26 ID:???
ローレンツ力が電場と磁場の定義式、ってなんかで見た気がする。
369ご冗談でしょう?名無しさん:2006/11/28(火) 20:24:21 ID:???
やっぱり電場も磁場も力と関連付けて定義するのかな。
370ご冗談でしょう?名無しさん:2006/11/28(火) 22:13:09 ID:yOV+duNC
どうしても腑に落ちないのですが、
β崩壊で中性子は電子と陽子と電子ニュートリノになりますが
一体ニュートリノって何なのでしょうか?
反対に陽子と電子をひっつけただけだと中性子にならずに
何になるのですか?

それと弱い力はβ崩壊のときに放出されるエネルギーだと
言われますがこれはニュートリノと電子、陽子を結びつける力ということなのでしょうか?

ここらへんのことが本を読んでさっぱり分かりません。
どなたか分かりやすく説明してくださいませんでしょうか?
371ご冗談でしょう?名無しさん:2006/11/28(火) 22:17:30 ID:???
>>370
ttp://ja.wikipedia.org/wiki/%E3%83%99%E3%83%BC%E3%82%BF%E5%B4%A9%E5%A3%8A#.CE.B2-.E5.B4.A9.E5.A3.8A
とりあえずWIKI

あと、学年と物理の履修状況晒してくれない?
372ご冗談でしょう?名無しさん:2006/11/28(火) 22:20:20 ID:sCr3MT8p
物理学科以外でも、工学部とかだと量子力学やりますか?
373ご冗談でしょう?名無しさん:2006/11/28(火) 22:25:44 ID:kZMZG28c
>>372
やるよ
半導体とか材料系も
でも公式使えればあと導出なんてどうでもいいやってかんじ
374ご冗談でしょう?名無しさん:2006/11/28(火) 22:33:06 ID:yOV+duNC
>>371
Wikiはすでに読んでいます。
自分は化学を専攻している社会人です。
独学で勉強しているんですがやっぱり難しいです。
どうかご教授ねがえませんでしょうか?
375ご冗談でしょう?名無しさん:2006/11/28(火) 22:51:33 ID:???
>>370
中性子の崩壊で出てくるのは電子ニュートリノの反物質で、
陽子と電子をぶつけると中性子の他に電子ニュートリノが出てくる、
とwikipediaには書いてあった。
自分はまだちゃんと学んだことが無いから以下話半分で。

量子論でいう“力”は複数の場の間の相互作用のことを指していて、
巨視的に力として見えてくる運動量のやり取りだけでなく
異なる粒子同士の変換も含まれているんじゃないかな。
強い力や弱い力や電磁気力とかの区分は
相互作用がどんな形で起こっているかで分類しているとか。
376ご冗談でしょう?名無しさん:2006/11/28(火) 23:11:59 ID:???
フェムト秒レーザーの発生原理について知りたいのですけど
どうやってこんな短いパルスを発生させることが出来るのでしょうか?
ネットでは検索しましたがレーザーの発生原理は書いてあっても
フェムト秒パルスをいかにして作るのかについては
どこのページも述べてくれていません。
本で調べたいのですがどの本に載っているのでしょうか?
学校の図書室でも調べましたがどうしてもうまく見つけることが
出来ませんでした。どなたか教えもらえないでしょうか?
377ご冗談でしょう?名無しさん:2006/11/28(火) 23:16:45 ID:yOV+duNC
>>375
丁寧な回答ありがとうございます。
W/Zボソンが電子、陽子、反電子ニュートリノを結びつけ合っているような
イメージなのでしょうか?
378Σ(´D`;の):2006/11/29(水) 00:27:22 ID:???
http://drudgery.s83.xrea.com/g.htm
あたまのいーおまいらにへるぷキボンヌ!
379ご冗談でしょう?名無しさん:2006/11/29(水) 00:28:28 ID:???
しね
380ご冗談でしょう?名無しさん:2006/11/29(水) 01:51:33 ID:???
>>377
電子と陽子とニュートリノから中性子が作られてると考えてたらそれは間違い。
せめてクォークレベルまでは見るべきで、陽子uud→中性子uddがベータ崩壊。
uがdに変わる時にボソンやニュートリノが関わってくる。←俺もようわからんけど多分この辺が弱い力
381ご冗談でしょう?名無しさん:2006/11/29(水) 02:22:41 ID:???
調和振動子の問題で、
(-d^2/dξ^2 + ξ^2)ψ=εψは、ξ→±∞のとき、εの項は無視できる。
というのがよくわかりません。
どの教科書や参考書みても、書いてあるのですが、どういうことなのでしょうか?
382ご冗談でしょう?名無しさん:2006/11/29(水) 02:23:28 ID:fkeE7flD
レナード現象ってご存知の方いますか?
レナード博士という人がノーベル賞受賞するきっかけになったらしいんですが
383376:2006/11/29(水) 03:22:14 ID:M4rKtUhd
どうかよろしくお願い致します。
本当に困っています。
384ご冗談でしょう?名無しさん:2006/11/29(水) 04:18:00 ID:???
こんな時間じゃ人がいないのも当たり前だろう・・・
とりあえず調べたらこんなん見つかったが
ttp://www.aist.go.jp/aist_j/press_release/pr2003/pr20030731/pr20030731.html
ttp://jstore.jst.go.jp/cgi-bin/patent/ipc/detail.cgi?id=3509&parent=B23%20%20%20%20%20%20%20%20%20%20%20%20%20&fabrication=1
チタンサファイアレーザーが云々とか言ってるけど何のことやら俺は知らない
385ご冗談でしょう?名無しさん:2006/11/29(水) 04:58:26 ID:???
>>377
中性子の崩壊と言っているのは、微視的に見ると>>380の言うとおり
dクォークのuクォークへの変身。
この変身を司るのがいわゆる弱い力で、それを媒介する粒子がW/Zボソン。
だからこの時Wボソンも放出されているわけだが、
このWボソンも極めて短い時間で崩壊する。
だからβ崩壊というのは、実は二段階に変身が起こっている。

d  ->u+W-
W- ->電子+反電子ニュートリノ

で、観測者から見ると、
中性子->陽子+電子+反電子ニュートリノ
に崩壊したように見える。
386ご冗談でしょう?名無しさん:2006/11/29(水) 10:34:05 ID:R7JSn4jv
互いに距離sだけ離れた2つの点電荷qと-qによる電気双極子がある
電気双極子モーメントp=qsの中点を原点、軸pの方向をx軸、また原点と距離r離れている点とx軸のなす角をθとする
r>>sの条件で問いに答えよ
(1)マクローリン展開の公式を用いてA点の電位を求めよ
(2)A点の電場をE=(Er,Eθ)のように二次元極座標での成分表示で表すとき、Er,Eθを求めよ
(3)A点の電場をE=(Ex,Ey)のように直角座標での成分表示で表すとき、Ex=0となる角度θを求めよ.またこの角度θのときのEyの大きさを求めよ


(2)のErは(1)で求めたVをrで偏微分して出てきたので単位の次元があってるのですが
EθはVをθで偏微分して1/rをかけてるのでまるで単位の次元が合いません
Eθの単位は[degree]になるんじゃないのですか?
それと(3)の解き方もお願いします
387ご冗談でしょう?名無しさん:2006/11/29(水) 10:37:23 ID:???
>>386
Eθの単位は[degree]じゃありません
(3)に関しては丸投げは駄目
388ご冗談でしょう?名無しさん:2006/11/29(水) 11:50:01 ID:???
http://f57.aaa.livedoor.jp/~monositu/imagebord/img/Monositu_20061129114602-130.JPG
Eθはこんなんでいいのですか?
電場を極座標で表すとErが電場の強さ、Eθが電場の向きのx軸との角度を示すと思っているのですが違いますか?
389ご冗談でしょう?名無しさん:2006/11/29(水) 11:52:40 ID:???
>>388
>電場を極座標で表すとErが電場の強さ、Eθが電場の向きのx軸との角度を示すと思っているのですが違いますか?
違う
390ご冗談でしょう?名無しさん:2006/11/29(水) 14:25:32 ID:???
>>377
ファインマンダイアグラムについて調べて
それと一緒に考えろ
391381:2006/11/29(水) 19:57:15 ID:???
別のスレで質問することにします。
392ご冗談でしょう?名無しさん:2006/11/29(水) 20:16:35 ID:R7JSn4jv
Ex、Eyを求めて(3)は解けたのですがEθは何を表す値なのですか?疑問です
393ご冗談でしょう?名無しさん:2006/11/29(水) 20:31:35 ID:???
光速度不変の原理って間違ってるの?

wikiで全否定されてたけど、高校時代にさも
正論のように教えられた…

猿の惑星みたいな時間の錯誤は
起こるのか否か教えてください。
394ご冗談でしょう?名無しさん:2006/11/29(水) 20:37:21 ID:???
何のwikiを見たのか知らんが、んなもん信用すんなw
395ご冗談でしょう?名無しさん:2006/11/29(水) 20:38:36 ID:???
>>393
まだそれに反する観測事実はないという意味で正しい。

>wikiで全否定されてた
どこ?
396ご冗談でしょう?名無しさん:2006/11/30(木) 02:19:13 ID:hD4M5JIg
DMM(デジタルマルチメータ)を用いずに、電流計と電圧計のみを用いて、
電流計と電圧計の内部抵抗を知る方法ってありますか?
教えて下さい!
397393:2006/11/30(木) 02:30:24 ID:???
398ご冗談でしょう?名無しさん:2006/11/30(木) 02:32:24 ID:???
>>397
そのページが全部間違いだと思うのが普通
399ご冗談でしょう?名無しさん:2006/11/30(木) 02:36:21 ID:???
>>397
インターネットはとても便利な道具だが、その一方で入手した情報の正確性・信頼性に
問題がある場合が少なくないという弱点は知っておかなければならない。
誰でも適当に発信できるメディアだからね。ここみたいな掲示板もそれは同じだけど。

で、「現代の物理学界でこのような批判がされないのはアインシュタインを神聖視しているから」的な
発言はほぼすべて、物理学を正しく理解せずに批判しようとする人の妄言と思ったほうがいい。
400ご冗談でしょう?名無しさん:2006/11/30(木) 04:12:44 ID:6TQbQjgC
角をL M Tで表すと何?
401ご冗談でしょう?名無しさん:2006/11/30(木) 04:54:43 ID:???
>>397
物理学は実験事実に沿う事が全てであって、詭弁を述べれば成り立つものではない。
数学的に無矛盾な系を考える事は幾らでもできるが、
そのような世界が物理学的に存在するとは限らない。

数学的に間違っている場合は言うまでも無く議論の埓外だけどな
402ご冗談でしょう?名無しさん:2006/11/30(木) 07:18:55 ID:???
>>395
反するどころか実測されてるだろ
403ご冗談でしょう?名無しさん:2006/11/30(木) 10:45:46 ID:???
>>402
君が>>395に噛みつく意味がわからない。
404397:2006/11/30(木) 11:53:29 ID:ID18P2Ia
>>398-401
明確な御返答ありがとうございました。
勉強になりました。
405ご冗談でしょう?名無しさん:2006/11/30(木) 11:56:54 ID:???
>>403
別に噛み付いてない
406ご冗談でしょう?名無しさん:2006/11/30(木) 12:24:22 ID:???
>>402
実測には必ず誤差が伴うから、慎重な言い方をすれば>>395のようになるだけ
407ご冗談でしょう?名無しさん:2006/11/30(木) 14:05:36 ID:???
>>400 各は無次元だよ。
408ご冗談でしょう?名無しさん:2006/11/30(木) 16:17:56 ID:???
誤差って。
409ご冗談でしょう?名無しさん:2006/11/30(木) 16:36:55 ID:???
本屋で「マンガでわかる物理(力学編)」ての見つけたんですが
これやたら萌えるんですが大丈夫ですか?
410ご冗談でしょう?名無しさん:2006/11/30(木) 18:27:10 ID:0Y7j6/Y5
ちょい質問なんだが、

たとえば、部屋の温度が一定なのにそこに
あるモノの温度が違うってことはありえる?
ちなみに発熱するものではなく、
十分な時間経っていたとして。

金属のテーブルの温度がひんやりするのは熱伝導率が大きいからなんかな?
411ご冗談でしょう?名無しさん:2006/11/30(木) 18:31:37 ID:3jTVZV/x
ここで質問していいのかどうか微妙なのですが…。
波長板に関する質問です。
直線偏光の2つの異なる波長のレーザを同軸伝播させて、
一方の偏光はそのままで、もう一方の偏光を90度回転させるような
2波波長板(デュアル波長板)の構造もしくは原理を教えて下さい。
412ご冗談でしょう?名無しさん:2006/11/30(木) 18:34:17 ID:???
>>410
室温が一定で、十分な時間が経過していれば、中の物の温度も室温と同じとなる。
但し、初期の室温が、中の物の初期温度で暖められたり、冷やされたりするから
一定時間後の室温は変化している。
ひんやり感じるのは熱伝導率が大きいで正解。
413ご冗談でしょう?名無しさん:2006/11/30(木) 18:41:49 ID:???
結局、ペンタクォークってあるのでしょうか?

素粒子実験で発見されたと報告されたものが、追試で見つからない場合
扱いはどうなるのでしょう? 単独グループの発見だけだと評価は
保留扱いになるんでしょうか? 

詳しい人教えてくださいです。
414ご冗談でしょう?名無しさん:2006/11/30(木) 18:51:52 ID:???
常温核融合と同じ運命をたどるでしょう。
415ご冗談でしょう?名無しさん:2006/11/30(木) 20:03:55 ID:???
Particle Data Groupが2年毎に出してるReview of Particle Physics、
2004年版ではΘ+のStatusは星3つでBaryon Summary Tableにも載ってたけど
2006年版では星1つに減らされ、Summary Tableからは除かれ、
Particle Listingsでの記述だけになっちゃった
416ご冗談でしょう?名無しさん:2006/11/30(木) 20:15:23 ID:???
ちなみにStatusの星の数と存在の確からしさの関係は

4つ:certain
3つ:very likely〜certain
2つ:only fair
1つ:poor
417ご冗談でしょう?名無しさん:2006/11/30(木) 20:29:05 ID:???
Higgsも一瞬騒がれたなw
バックグラウンドに埋もれたシグナルフィットなんて、如何様にでもふらつくものだからな。
3σ程度で「ほぼ確実!」とか騒いでいるとこういう目に会う。
418ご冗談でしょう?名無しさん :2006/11/30(木) 20:37:05 ID:???
>>415 お返事ありがとうございました。
Review of Particle Physics 早速見てみました。
確かに 2004年版では Exotic Baryons として載ってたのが
(文章には、6つ実験の報告があるけど留保つきって書いてますね)
2006年版ではなくなってます・・びっくりしました
419ご冗談でしょう?名無しさん:2006/11/30(木) 20:46:51 ID:???
物理学者で、雷鳴ってる日に凧を揚げて雷の電圧計ろうとした奴いたよな?そいつの名前何だっけ?ボルトだっけ?
420ご冗談でしょう?名無しさん:2006/11/30(木) 20:51:30 ID:???
MOTHERで雷を吸収する「フランクリンバッジ」ってあったろ
421ご冗談でしょう?名無しさん:2006/11/30(木) 20:51:46 ID:???
422ご冗談でしょう?名無しさん:2006/11/30(木) 20:52:23 ID:???
>>419
マルチ
423ご冗談でしょう?名無しさん:2006/11/30(木) 20:59:32 ID:uzI0jyZX
フランクリンさんか・・・ありがとm(__)m
424ご冗談でしょう?名無しさん:2006/11/30(木) 21:27:26 ID:G5//YI/D
SETI(地球外知的生命体探査)で名高いプエルトリコのアレシボ天文台がこのほど
うお座のη(イータ)星の方角から、断続的で規則性のある
電磁波が発せられているのを観測したと発表し、話題となっている。

アレシボ天文台によると、この電磁波が初めて捉えられたのは日本時間の27日午前3時頃。
周波数1420MHzの電磁波が24.840秒続き、65.242秒の空白をおくというパターンで
繰り返し発せられるのを、およそ6時間に渡って観測した。
その後、一旦は電磁波は途絶えたが、およそ27時間後の28日午前6時頃(日本時間)
今度は同じ方角から同じ周波数の電磁波が65.242秒続き、24.840秒の空白を空けるという
全く逆のパターンで発せられるのが観測され
現在も同周波数、同パターンの電磁波が受信されているという。

アレシボ天文台では、現在うお座のη(イータ)星方向に太陽に似た恒星や地球型惑星が
存在するという確証は無く、恒星などから似たような電磁波が発せられることも多いので
この電磁波が直ちに地球外知的生命体の発したものと断定するには早計過ぎるとしながらも
他の研究機関とも連携しながら慎重に観測を続け、データの解析を急ぎたいとしている。

http://naturenews.web.fc2.com/news/20061129.html
425ご冗談でしょう?名無しさん:2006/11/30(木) 21:40:12 ID:???
地雷だったなんて。。。久しぶりに踏んじまったよ。naturenewsというサイト名に騙された OTZ
426ご冗談でしょう?名無しさん:2006/12/01(金) 00:37:37 ID:f8VTZdJN
スレ違いだったのでこちらで聞きます

内容がマニアックなんですけど
半導体中のholeと、optical phononとのカップリング
特に、散乱現象について論じている物性の本、もしくは論文ってありませんかね?

古典電磁気とかじゃなくて、場の量子論とかちゃんと考慮してる
多少古めでお願いします。
427ご冗談でしょう?名無しさん:2006/12/01(金) 00:49:20 ID:m2DAMiNV
どうして、宇宙が拡大していくなんて分かるんですか?(
僕は縮小して言っているように考えますけど。
宇宙論強い人教えてください。
428ご冗談でしょう?名無しさん:2006/12/01(金) 00:51:30 ID:FSFpFM+j
>>427
星の速度を観測すると遠くにいる星ほど大きい速度で地球からはなれていっているから
429ご冗談でしょう?名無しさん:2006/12/01(金) 00:55:40 ID:???
遠くの銀河を見ると波長が伸びて観測されるから遠ざかってるのが判るのさ
430ご冗談でしょう?名無しさん:2006/12/01(金) 02:26:15 ID:???
第2宇宙速度とかなんとかいうけれど、そんなすごいスピードを出さなくても、地上から上空に向かって、
1G以上の加速度で進めば、地球の重力圏を振り切って宇宙に出ることはできそうな気がするのですが、
それではダメなのでしょうか?
431ご冗談でしょう?名無しさん:2006/12/01(金) 02:33:57 ID:???
1G以上じゃなくてもまっすぐ上に進めば宇宙には出られる
まあそれでもいいっちゃいいんだが

例えば衛生、あいつなんかは周回軌道に乗っているから打ち上げた後に燃料使わなくても良い
もしこれがまっすぐうちあがっただけなら、ずっと下向きに燃料噴出してないと落ちてくる
432ご冗談でしょう?名無しさん:2006/12/01(金) 02:34:05 ID:???
>>430
それができればそれでもいいけど、ロケットでそれをやると物凄く効率が悪い。
433ご冗談でしょう?名無しさん:2006/12/01(金) 02:44:40 ID:???
レスありがとうございます。

それだけ確認出来れば、納得です<(_ _)>
434ご冗談でしょう?名無しさん:2006/12/01(金) 05:00:24 ID:???
振動のモードとはいったいなんですか。
435ご冗談でしょう?名無しさん:2006/12/01(金) 09:18:34 ID:???
436ご冗談でしょう?名無しさん:2006/12/01(金) 11:58:53 ID:???
>>427
>僕は縮小して言っているように考えますけど
そう考える根拠は?
437ご冗談でしょう?名無しさん:2006/12/01(金) 12:22:55 ID:???
私、量子力学と熱力学を始めたばかりの初心者ですが、疑問があります。
運動方程式はp,qが与えられたときの時間発展
マクスウェル方程式は、電荷や電流が与えられたときの電磁場の形とその時間発展
シュレーディンガー方程式は、ハミルトニアンがあたえられたときの波動関数の形とその時間発展
をそれぞれ記述している、と理解しています。
つまり、物理にはそれぞれの系の状態の時間発展を記述する方程式があるのではないか?
と思いついたのですが、熱力学にはそういうものがないような気がします。
それはなぜなのでしょうか?見当違いなことをいってたらすみません。
438ご冗談でしょう?名無しさん:2006/12/01(金) 12:33:57 ID:???
そもそもが熱平衡状態に有る、つまり、時間無限大で物質がどの様な挙動を示すかを考える学問だから。
時間発展と言う考え方は無い。
439ご冗談でしょう?名無しさん:2006/12/01(金) 12:39:09 ID:???
>>438
なるほど。熱平衡という限定された状態のみについて考えるのですね。
限定されてない状態のマクロな挙動は流体力学とかになるんですかね?
440ご冗談でしょう?名無しさん:2006/12/01(金) 13:03:54 ID:???
>>439

非平衡系の熱力学ってのも昔からあるよ。まともに扱えるようになってきたのは
比較的最近だとは思うけど。
非平衡定常系の方がやりやすいけど、非平衡非定常系をやっている人もいたと思う。
ただ、まだそんなに完成された学問じゃない。
441437:2006/12/01(金) 14:20:04 ID:???
勉強になりました。
お二方、ありがとうございました。
442376:2006/12/01(金) 14:23:30 ID:s6crZlYj
どうかよろしくお願い致します。
どなたか詳しい方はおられないのでしょうか?
図書室で調べてもやっぱり分かりません。
どうかよろしくお願い致します。
443ご冗談でしょう?名無しさん:2006/12/01(金) 14:31:50 ID:s6crZlYj
蛍光灯には白色だけでなく、黄色や橙色などのものも
ありますがこれは普通の蛍光灯とは発光原理が異なるのでしょうか?
単に蛍光灯の周りにカラーフィルムを貼り付けたものなのでしょうか?
444ご冗談でしょう?名無しさん:2006/12/01(金) 14:39:43 ID:???
>>443
昼白色とか電球色とか微妙に色のついたのはよく見るけど
はっきり黄色や橙色のは見たことないな。ナトリウムランプや
ネオンサインじゃないんだよね。

いずれにしても蛍光灯というからにはガラス管の内側に
そういう色を出す蛍光体を塗布してるんだと思うが。
これは普通の蛍光灯も同じ
445444:2006/12/01(金) 14:48:07 ID:???
>>444
>ネオンサインじゃないんだよね
ちょっとぐぐったらネオンサインも本来の赤色以外のは
蛍光体で発色を調整してて、それは蛍光灯そのものだった。
通常の蛍光灯が水銀ガスを使ってる点以外は原理は同じ。
446ご冗談でしょう?名無しさん:2006/12/01(金) 15:41:54 ID:???
>>442
知識レベルをおしえて
447ご冗談でしょう?名無しさん:2006/12/01(金) 15:51:23 ID:s6crZlYj
>>446
普通のレーザーは扱ったことがあります。
フェムト秒など、時間分解分光などについては論文などで
読んだくらいで実際に扱ったことはありません。
レーザーに関する基礎的な知識はありますが
如何にしてこんなに短い時間のパルスレーザーを可能になるのかが知りたいのです。
どうかよろしくお願い致します。
448ご冗談でしょう?名無しさん:2006/12/01(金) 16:00:16 ID:???
>>447
マルチモードで発振するような共振器作っておいて発振させる
(ここまではモード間の位相がばらばらのただのマルチモードレーザー)

その中に強い光だけ透過させるような非線形媒質を入れる
例えば過飽和吸収体やKerrレンズ+アパーチャなど
するとモード間の位相がそろったときに短パルスになり光強度が上がるため
短パルス光が共振器中を往復できるようになり発振に至る

っていうのがモードロックの大体の様子です
449ご冗談でしょう?名無しさん:2006/12/01(金) 16:15:12 ID:???
>>411
高速軸と低速軸との位相差が
片方の色ではnπ、もう片方の色では(n+1/2)πになるように
結晶の厚さを選ぶ
450ご冗談でしょう?名無しさん:2006/12/01(金) 16:15:45 ID:???
トンネルの橙色のランプはナトリウムランプだね。
451ご冗談でしょう?名無しさん:2006/12/01(金) 17:03:38 ID:???
>>431-432
第2宇宙速度の意味わかってるか?
452ご冗談でしょう?名無しさん:2006/12/01(金) 18:37:43 ID:tPYDqLg8
磁束密度Bの磁界に直交して半径rの一巻きの円形コイルがおかれていて、この磁界が増加した時コイルに沿って生じる電界の大きさEがV/2πrになるのはなぜですか?
453ご冗談でしょう?名無しさん:2006/12/01(金) 18:55:03 ID:???
>>452
つ[電界と電位の関係]
454ご冗談でしょう?名無しさん:2006/12/01(金) 19:13:49 ID:???
>>451
分かってないのは君だと思うよ
455ご冗談でしょう?名無しさん:2006/12/01(金) 19:26:07 ID:???
いや、分かってないのは俺だ!
456ご冗談でしょう?名無しさん:2006/12/01(金) 20:14:13 ID:???
いや、俺だってば
457ご冗談でしょう?名無しさん:2006/12/01(金) 20:16:23 ID:dWdKRqIm
実験で得られた時系列データをフーリエ級数展開で近似して、係数anとbnを求めたいんですが、そのためのC言語かFortranのライブラリってご存知無いでしょうか?
教えてくれたら私のとっておきエロ画像アップします。
458ご冗談でしょう?名無しさん:2006/12/01(金) 20:20:27 ID:???
「FFT ライブラリ」でググるといくつか見つかるよ。
459ご冗談でしょう?名無しさん:2006/12/01(金) 20:57:16 ID:???
>>454
初速1m/s、加速度0でも地球の引力圏を脱出してしまいますが
第2宇宙速度というのは1m/sよりも小さいのでしょうか?
460ご冗談でしょう?名無しさん:2006/12/01(金) 21:01:41 ID:???
459取り消し
461ご冗談でしょう?名無しさん:2006/12/01(金) 21:13:11 ID:???
認めません
462ご冗談でしょう?名無しさん:2006/12/01(金) 22:18:12 ID:???
「人類は別の惑星に移住を」 ホーキング博士

ロンドン(ロイター)
 英ケンブリッジ大のスティーブン・ホーキング博士(64)は30日、英BBCラジオとのインタビューで、
人類の未来について語り、「滅亡を避けるためにはどこか別の惑星に移住しなければならない」と話した。
博士はまた、自ら宇宙旅行に参加してみたいと述べ、新たな挑戦への意欲を示した。

ホーキング博士は世界的な理論物理学者で、一般向けの近著としては「ホーキング、宇宙のすべてを語る」
がある。20代で筋萎縮性側索硬化症と診断され、車いすの生活に。
40代で気管切開の手術を受けた後は、コンピューターによる合成音声で会話をしている。

このほど、英王立協会が優れた科学者に贈るコプリー賞の受賞を機に、インタビューに応じた。
博士はこの中で、「人類は遅かれ早かれ、いん石の衝突や核戦争によって滅亡する可能性がある」と指摘。
そのうえで「別の太陽系の惑星へ移動し、自立した生活を営むことができるようになれば、将来は安泰だ」
と強調した。

そのためには高速の移動手段が必要になる。人類が生存可能な惑星に到達するのに、
従来のロケット技術では5万年もかかってしまう。

「SF映画には、一瞬のうちに目的地へ移動できる『ワープ』という方法が登場する。だが残念ながら、
光を超える速さで移動することは、理論上不可能だ」と、博士は話す。
「しかし、光速に近い速さを実現することなら可能だ。これが達成できれば、目的とする惑星に
6年間で到着できる。移動する本人たちにとって、それほど長い時間ではないはずだ」という。

博士はまた、自身の将来について、「死ぬことを恐れてはいないが、死を急ぐつもりもない。
次なる目標は宇宙へ行くことだ」と言明。
「(民間宇宙旅行会社を設立した)リチャード・ブランソン(バージングループ会長)が、
力を貸してくれるかもしれない」と語った。

http://www.cnn.co.jp/science/CNN200612010014.html
463ご冗談でしょう?名無しさん:2006/12/01(金) 22:27:22 ID:???
で、質問は?
464ご冗談でしょう?名無しさん:2006/12/01(金) 22:28:56 ID:1fgPNV9O
遅レスですが340ですが
>>350さんありがとうございます。
多分「普通の確率過程の知識」が抜けてるんだと思います。
確率論の勉強してみようと思います。

ttp://www.aa.alpha-net.ne.jp/t2366/ネルソンの確率量子化htm.htm
ここに書いてあることとかをもっと1つ1つ掘り下げれば理解できますかね?
465ご冗談でしょう?名無しさん:2006/12/01(金) 23:10:41 ID:???
>>457
EXCELでもできる
466ご冗談でしょう?名無しさん:2006/12/02(土) 00:22:29 ID:0e/Rgq2e
金属と半導体の電気抵抗のそれぞれの特性を利用してどんな製品に利用されているか?
467ご冗談でしょう?名無しさん:2006/12/02(土) 00:26:15 ID:???
468ご冗談でしょう?名無しさん:2006/12/02(土) 00:33:03 ID:0e/Rgq2e
>>467
半導体は温度測定?
金属は?
469ご冗談でしょう?名無しさん:2006/12/02(土) 00:44:01 ID:D7v/NDtF
>>468 俺思うに、真面目に考えないで>>466の答えには、例えば
「電線」って答えれば良い、と思うんだが、どうだろう?
470ご冗談でしょう?名無しさん:2006/12/02(土) 00:55:23 ID:???
>>466
熊ん子とかいいと思うよ
471ご冗談でしょう?名無しさん:2006/12/02(土) 01:09:41 ID:???
確かに両方使われてるしなぁ。
472ご冗談でしょう?名無しさん:2006/12/02(土) 11:42:40 ID:pcOdQv1N
反射式ストーブで発生させた輻射熱は真空中でも
進むことが出来ると本に書かれているのですがどういうことなのでしょうか?
反射式ストーブとは言え最初に発するのは燃焼体の熱ですよね?
それが反射板に当たることで熱線に変わるということなのでしょうか?
それと熱線は一体何を媒介として真空中を伝わっていくものなのでしょうか?
473ご冗談でしょう?名無しさん:2006/12/02(土) 11:51:43 ID:???
ポンドの単位がlbなのは何か意味があるのでしょうか?
pdとかpnにならないのはなぜなのでしょうか?
474ご冗談でしょう?名無しさん:2006/12/02(土) 12:29:46 ID:???
>>472
電磁波
475ご冗談でしょう?名無しさん:2006/12/02(土) 12:46:46 ID:???
>>473

Libraより
476ご冗談でしょう?名無しさん:2006/12/02(土) 14:39:39 ID:???
>>472
太陽だってあったかいだろ
477ご冗談でしょう?名無しさん:2006/12/02(土) 15:03:52 ID:???
熱力学第1法則の中で、電磁波の運ぶエネルギーは
仕事と熱のどちらの方に入れるのが普通?
478ご冗談でしょう?名無しさん:2006/12/02(土) 15:08:07 ID:???
dU=TdS-PdV+μdN+α←ここ
479ご冗談でしょう?名無しさん:2006/12/02(土) 15:22:47 ID:???
>>477 マクロに見て体積変化がなければ熱と見ていいんじゃないの?
480472:2006/12/02(土) 16:27:22 ID:pcOdQv1N
熱線によるエネルギーとhνで表される光のエネルギーとは
同じものだということなのでしょうか?
ということは電子レンジによる物質を暖める原理と反射式ストーブによる
物質を暖める原理は光の波長が違うだけで同じだということなのでしょうか?
481ご冗談でしょう?名無しさん:2006/12/02(土) 17:24:54 ID:???
まことに相済みませんが
「なんで質量を持った物質同士は引き寄せ合うの?」か教えてください。
http://news20.2ch.net/test/read.cgi/news/1165046205/
482ご冗談でしょう?名無しさん:2006/12/02(土) 17:49:01 ID:???
ポテンシャルをつくるから。静電気と同じようなもの。
電荷が存在すればクーロン力が働く。質量が存在すれば万有引力が働く。
質量を持つということで、空間に影響をするんだと考えておけばいいんじゃない。
そして、その空間の影響を受けて、力が働くと。
483ご冗談でしょう?名無しさん:2006/12/02(土) 18:16:17 ID:???
>>477
場合によるんじゃない。輻射は熱、アンテナでキャッチするなら仕事とか。
>>479
体積は関係ないだろう…。
484ご冗談でしょう?名無しさん:2006/12/02(土) 18:17:38 ID:???
>>480
似たようなもん。
485ご冗談でしょう?名無しさん:2006/12/02(土) 19:32:01 ID:???
>>480

励起する対象が違う(電子レンジは分子間振動・回転,ストーブは
分子内振動)が、吸収された電磁波のエネルギーが熱に転化している
という意味では同じ。
486ご冗談でしょう?名無しさん:2006/12/02(土) 20:36:57 ID:???
>>483
体積が関係ないって???
仕事=力×変位
だから、体積変化しなければ仕事も0って意味だら?
487ご冗談でしょう?名無しさん:2006/12/02(土) 20:39:54 ID:???
>>486
静岡県民か?
488ご冗談でしょう?名無しさん:2006/12/02(土) 20:48:28 ID:???
えっ? そうだけどなんでわかるの?
静岡以外では仕事の定義が違うのか?
489ご冗談でしょう?名無しさん:2006/12/02(土) 20:49:07 ID:???
>意味だら?

方言丸出しw
490ご冗談でしょう?名無しさん:2006/12/02(土) 21:00:34 ID:???
もう帰る
491ご冗談でしょう?名無しさん:2006/12/02(土) 22:36:09 ID:???
ワロタww
492ご冗談でしょう?名無しさん:2006/12/03(日) 00:11:30 ID:XEv64Vum
だらって・・・



 知り合い思い出した
493ご冗談でしょう?名無しさん:2006/12/03(日) 00:26:05 ID:???
かわいいな
494ご冗談でしょう?名無しさん:2006/12/03(日) 01:08:23 ID:P/hHV0fn
ERPペアとなってる量子って、
なんで片っぽ観測して状態を特定すると
もう片っぽまで特定されるわけ?

なんでそげな性質を持つのか、
それが知りたいわけよ

遠く離れていたとしても同時に反応するあたりもおかしい
無限大の速度で伝達される、その媒体も怪しい

なんなんだこれは
495ご冗談でしょう?名無しさん:2006/12/03(日) 01:10:34 ID:???
EPRパラドックスでぐぐれ
496ご冗談でしょう?名無しさん:2006/12/03(日) 01:24:05 ID:???
光学って、やっぱりヘクト光学とかできっちり勉強しておくべき?
それとも、結局大部分は波動や電磁気学や量子力学に含まれるのだから、特に必要ない?

光学の試験は、レーザーを使ったフラウンホーファー回折とか、波長が違うと屈折率が異なるのは
なぜかとかいった問題だった。もちろん単位は取ったんだけど・・・。
497ご冗談でしょう?名無しさん:2006/12/03(日) 02:22:26 ID:???
大学生なんだから
必要か必要でないかぐらいじぶんで判断しろ
498ご冗談でしょう?名無しさん:2006/12/03(日) 02:29:14 ID:???
>>497
大学生はそんなに偉い存在じゃない。
だいたいの大学生ってのは思い上がってる。
これから先なにが本当に必要かなどひとりの大学生風情にそう簡単にわかりやしない。
自分で判断できないから質問してるんじゃないか。頭の固い奴だな。
499ご冗談でしょう?名無しさん:2006/12/03(日) 02:33:50 ID:???
んじゃ言ってやろう

大学生ったって、千差万別これから先にやりたいことで
必要なこと、そうでないことは全く変わってくる

だから自分で考えろと言うんだ
500ご冗談でしょう?名無しさん:2006/12/03(日) 02:35:57 ID:XEv64Vum
直感も大事
501ご冗談でしょう?名無しさん:2006/12/03(日) 02:39:33 ID:???
もちろん考えているさ。あなたの言う通りね。
でも、大学生がこれから先自分はいったいなにをするかはっきり決めるのは難しいと思うんだ。
もちろん、やりたいことというのは漠然とあるのだが。

ただ、光学に関しては微妙。興味はあるが、まともに取り組む時間がとれないというのが現実。
502ご冗談でしょう?名無しさん:2006/12/03(日) 02:43:44 ID:???
>>501
とりあえず、俺のやっていた研究(物性実験)では全く光学は使わなかった

どの分野をやるかどのレベルまでやるか
好きにしろとしか言えないなw

とりあえず優がとれるレベルまではやっておいたら?
503ご冗談でしょう?名無しさん:2006/12/03(日) 02:50:04 ID:???
>>502
まぁそうだよね。
俺としては、光学をやっている暇があれば量子力学を深く勉強したい。
あと、数学が好きなんだよね。下手の横好きなんだけど。
数学科並みに数学の科目を受講して、もちろん物理の科目も履修していたら
ある程度切り捨てざるをえない部分がどうしてもでてきてしまう。

光学は基礎的な事柄は理解したから、これぐらいで我慢しておこうか・・・。
504ご冗談でしょう?名無しさん:2006/12/03(日) 03:04:20 ID:???
必要になったらやればいいさ。
505ご冗談でしょう?名無しさん:2006/12/03(日) 03:07:21 ID:???
>>504
そうだね。ありがとう。
506ご冗談でしょう?名無しさん:2006/12/03(日) 04:36:07 ID:???
大学の物理なのですが、教えてもらえると助かります・・・

両端を固定した長さπ、線密度σの弦が張力Tで張ってある。
この弦の微小振動の一般解を書き下せ。
507ご冗談でしょう?名無しさん:2006/12/03(日) 04:36:35 ID:???
508ご冗談でしょう?名無しさん:2006/12/03(日) 04:54:11 ID:Z+3soHH+
http://www.youtube.com/watch?v=hYdlE3TiVDY

すいません。これどうやって解くか教えてください。
509ご冗談でしょう?名無しさん:2006/12/03(日) 08:09:01 ID:???
510ご冗談でしょう?名無しさん:2006/12/03(日) 09:17:33 ID:Z+3soHH+
過去ログどこらへんですか?
511ご冗談でしょう?名無しさん:2006/12/03(日) 09:34:41 ID:???
hYdlE3TiVDY でググれ
512ご冗談でしょう?名無しさん:2006/12/03(日) 09:47:29 ID:Z+3soHH+
調べましたけど回転させて外すのはともかく
また確実にボルトをつけるのが無理でしょ?
513ご冗談でしょう?名無しさん:2006/12/03(日) 09:51:39 ID:???
>>486
関係ないよ。コンデンサにエネルギーを加えるのに体積変化があるとでも?
514ご冗談でしょう?名無しさん:2006/12/03(日) 14:48:18 ID:???
すいません、中学校の理科で質問させて下さい。

図はttp://imepita.jp/20061203/526030です

右の図のような装置をつくり、スイッチを入れたら、磁針XのN極がウの向きを指して止まった。
これについて、次の問いに答えなさい。

(1)このとき、コイルの中の磁界の向きはA,Bのどちらか。

(2)このとき、電流の向きはa,bのどちらか。

(3)このとき、磁針YのN極はカ〜ケのどの向きを指しているか。

(4)乾電池の向きを変えて電流の向きを逆の向きに流すと、磁針YのN極はカ〜ケのどの向きを指すか。

(1)は右ねじの法則でA、(2)もこの法則でbだと思うのですが、
(3)からはどのような法則で考えればいいのか解りません。
どうか、ヒントをお願いします。
515ご冗談でしょう?名無しさん:2006/12/03(日) 15:06:45 ID:???
>>514
>(1)は右ねじの法則でA

違う。単に磁石の向きから磁界の向きを推測するだけ。
(2)はそれあってるか?
(3)も右ねじの法則。
516ご冗談でしょう?名無しさん:2006/12/03(日) 18:33:30 ID:???
>>512
自分で作ってやってみろ
実物があると案外簡単にできることがわかる
実際に作った俺が言うんだから間違いない
517質問:2006/12/03(日) 18:51:49 ID:qmf3XdqM
文章がうまく書けないんだけど、ある物体に対して何かの力を加えたとき、その何かに応じて各々必ず決まった反応をする運動の名前があったんだが…例えば冷やすと必ず縮む、水をかけると必ず溶けるみたいな…思い出せない…てか、うまく書けない…
518ご冗談でしょう?名無しさん:2006/12/03(日) 18:53:54 ID:???
>>517
質問内容をまとめてから出直してくれ
超能力者でもなければ質問内容までは分かってくれないw
519ご冗談でしょう?名無しさん:2006/12/03(日) 18:55:01 ID:???
応答?
520514:2006/12/03(日) 18:59:09 ID:N4MIoWNn
>>515
thx!
(3)も右ねじを使うんだな。
(4)は頑張って解いてみるよ。ありがとう。
521ご冗談でしょう?名無しさん:2006/12/03(日) 19:02:52 ID:???
>>517
因果関係?
522ご冗談でしょう?名無しさん:2006/12/03(日) 20:06:13 ID:???
電磁波はエネルギーがE=hνの光子の集まりですが
静電場、静磁場をつくっている光子のエネルギーはいくらなのでしょうか?
523ご冗談でしょう?名無しさん:2006/12/03(日) 20:36:15 ID:???
>>522
ローレンツ変換
524ご冗談でしょう?名無しさん:2006/12/03(日) 20:59:28 ID:???
>>1の問題の解は

∂^2 * u(x,t) / ∂t^2 = c^2*∂*u(x,t) / ∂x^2 でいいのかな・・・?
525ご冗談でしょう?名無しさん:2006/12/03(日) 21:12:39 ID:???
ものすごく変な質問なのですが、前々から気になっていたので質問させてください。

時間の最小構成単位って何分の何秒なのでしょうか?

たとえば、テレビなら1/60秒単位で動いているようで、それは人間の認識できる最短時間で
あると聞きました。
それなら、この現実世界における、時間の最短時間とはどのくらいなのでしょうか。

物理では、原子や電子といったかたちで、モノをもっとも小さな要素に分解できています。
とすれば、物理で考える所の、もっとも小さな時間の要素とは何なのでしょう。


変な質問ですみません。
でも、ずーっと疑問に思っていたことなんです。
お手数ですが、御教授頂ければと思います。 宜しくお願いします。
526ご冗談でしょう?名無しさん:2006/12/03(日) 21:14:35 ID:???
今の所クォークとか
527ご冗談でしょう?名無しさん:2006/12/03(日) 21:15:06 ID:???
あぁ、時間か。時間は今の所連続と考えられてルはず(;´Д`)ハァハァ
まだ量子化はされてない(;´Д`)ハァハァ
528ご冗談でしょう?名無しさん:2006/12/03(日) 22:26:06 ID:???
>>523
ハア?
529ご冗談でしょう?名無しさん:2006/12/03(日) 22:57:31 ID:???
たぶん>>522の質問にまともに答えられる奴はこの板にいないと思う。

>>525
プランク時間(Gh/2π/c^5)=5.4×10^(-44)秒
530ご冗談でしょう?名無しさん:2006/12/03(日) 23:00:18 ID:???
訂正:(Gh/2π/c^5)^(1/2)=5.4×10^(-44)秒
531ご冗談でしょう?名無しさん:2006/12/03(日) 23:19:17 ID:???
最小観測であって
最小構成では無い気がするが
532ご冗談でしょう?名無しさん:2006/12/03(日) 23:20:39 ID:BPxtYc3q
コイルとコンデンサーについてですが
それぞれリアクタンスをRで表すのはおかしいでしょうか
やはりRでは抵抗という意味合いになってしまう?
533ご冗談でしょう?名無しさん:2006/12/03(日) 23:23:24 ID:???
なってしまうなあ
534ご冗談でしょう?名無しさん:2006/12/03(日) 23:24:45 ID:???
>>531
現在の物理で曲がりなりにもなんか言える最小の時間という意味。
535ご冗談でしょう?名無しさん:2006/12/03(日) 23:30:32 ID:???
V=ωLI

V=1/ωc*I
を見ると納得いかない・・・どう言えばいいんでしょう
536ご冗談でしょう?名無しさん:2006/12/04(月) 01:11:43 ID:???
そのままで良いじゃん
周波数特定しないと回路としての特性を表現できない

逆に言えば周波数の項を残しておかないと別物の回路になってしまう
537ご冗談でしょう?名無しさん:2006/12/04(月) 01:18:43 ID:???
>>535
もうちょい具体的にお願い。
リアクタンスの意味が理解できないってこと?
538質問:2006/12/04(月) 02:02:12 ID:CglkHPGs
517ですが、えーと全ての生物や物体に言える事ですが、与えられた力や行為によって対象の物が起こす反応の理系的な用語です。なんとかって習ったんですが思い出せない…
539ご冗談でしょう?名無しさん:2006/12/04(月) 02:04:39 ID:???
>>538
ぜんっぜんわからん
物理かどうかすら定かじゃないしな
教えてくれた先生に訊いてくれ
540ご冗談でしょう?名無しさん:2006/12/04(月) 02:43:37 ID:oGYbXP9l
書籍等の直方体を、立てる場合と、寝かす場合は
寝かす方が安定していると思いますが、
この現象はを物理で説明するにはどうすればよいでしょうか?
541ご冗談でしょう?名無しさん:2006/12/04(月) 02:49:29 ID:???
>>540
慣性モーメント

時間についてだけど、俺は連続体じゃなくて離散体だと考えている。
ついでに空間も、ごく微細なグリッド状だと考えている。
根拠はないけどね。
542540:2006/12/04(月) 02:59:01 ID:oGYbXP9l
初期状態でA面が下向きの直方体を、ある辺を中心に回転してB面を下向きにするために
必要なエネルギーを計算は可能でしょうか?
543ご冗談でしょう?名無しさん:2006/12/04(月) 03:03:43 ID:???
可能
544質問:2006/12/04(月) 08:36:59 ID:CglkHPGs
>>539 その先生は三年前に死にました…
545ご冗談でしょう?名無しさん:2006/12/04(月) 09:00:38 ID:???
とりあえず、>>519とか>>521が合ってるか違ってるかぐらい言え
546質問:2006/12/04(月) 11:00:32 ID:CglkHPGs
>>519>>521 返答ありがとう☆でも違うんだよ…  えーと、>>545みたいな傲慢な勘違いヤローのキモい童貞にもある反応の呼び名…あー!なんだっけ!たしか漢字二文字??
547ご冗談でしょう?名無しさん:2006/12/04(月) 11:40:19 ID:???
うわ、香ばしい
548ご冗談でしょう?名無しさん:2006/12/04(月) 11:55:36 ID:???
以後スルーで
549ご冗談でしょう?名無しさん:2006/12/04(月) 12:33:45 ID:???
>>537
うん、そもそもも分かってないのだと思う
それぞれV=RIにあてはめるとRの部分にあたる
だからその部分=Rでいいのかなでも抵抗って訳でもないしな・・・
・・・んで結局これはなんなんだ?です。

今日その範囲が入ってるテストだった。
点数だけで見れば80は固いけど理解はできてない。
550ご冗談でしょう?名無しさん:2006/12/04(月) 14:00:38 ID:IdJBGqa+
ベンゼンにある不揮発性物質を溶かしたら0.45K上昇した
その時の不揮発性の分子量を求めなさい
って問題があるんですけど
ヒントください
551ご冗談でしょう?名無しさん:2006/12/04(月) 14:15:06 ID:???
>>550

うむ、問題文がそれだけだとしたらどうやっても解けないな。
552ご冗談でしょう?名無しさん:2006/12/04(月) 14:16:23 ID:???
溶解熱
553ご冗談でしょう?名無しさん:2006/12/04(月) 14:27:26 ID:IdJBGqa+
>>551
すいません
ベンゼン10p^3
密度は0.879g/p^3
モル沸点上昇は2.70

不揮発性物質は0.0879gです
>>552
妖怪熱は違うはずです
554ご冗談でしょう?名無しさん:2006/12/04(月) 15:19:45 ID:???
(* 々`ノ(* 々`ノ(* 々`ノ(* 々`ノ(* 々`ノ(* 々`ノ(* 々`ノ(* 々`ノ(* 々`ノ(* 々`ノ(* 々`ノ(* 々`ノ(* 々`ノ(* 々`ノ(* 々`ノ(* 々`ノ
(* 々`ノ(* 々`ノ(* 々`ノ(* 々`ノ(* 々`ノ(* 々`ノ(* 々`ノ(* 々`ノ(* 々`ノ(* 々`ノ(* 々`ノ(* 々`ノ(* 々`ノ(* 々`ノ(* 々`ノ(* 々`ノ
(* 々`ノ(* 々`ノ(* 々`ノ(* 々`ノ(* 々`ノ(* 々`ノ(* 々`ノ(* 々`ノ(* 々`ノ(* 々`ノ(* 々`ノ(* 々`ノ(* 々`ノ(* 々`ノ(* 々`ノ(* 々`ノ
(* 々`ノ(* 々`ノ(* 々`ノ(* 々`ノ(* 々`ノ(* 々`ノ(* 々`ノ(* 々`ノ(* 々`ノ(* 々`ノ(* 々`ノ(* 々`ノ(* 々`ノ(* 々`ノ(* 々`ノ(* 々`ノ
(* 々`ノ(* 々`ノ(* 々`ノ(* 々`ノ(* 々`ノ(* 々`ノ(* 々`ノ(* 々`ノ(* 々`ノ(* 々`ノ(* 々`ノ(* 々`ノ(* 々`ノ(* 々`ノ(* 々`ノ(* 々`ノ
(* 々`ノ(* 々`ノ(* 々`ノ(* 々`ノ(* 々`ノ(* 々`ノ(* 々`ノ(* 々`ノ(* 々`ノ(* 々`ノ(* 々`ノ(* 々`ノ(* 々`ノ(* 々`ノ(* 々`ノ(* 々`ノ
(* 々`ノ(* 々`ノ(* 々`ノ(* 々`ノ(* 々`ノ(* 々`ノ(* 々`ノ(* 々`ノ(* 々`ノ(* 々`ノ(* 々`ノ(* 々`ノ(* 々`ノ(* 々`ノ(* 々`ノ(* 々`ノ
(* 々`ノ(* 々`ノ(* 々`ノ(* 々`ノ(* 々`ノ(* 々`ノ(* 々`ノ(* 々`ノ(* 々`ノ(* 々`ノ(* 々`ノ(* 々`ノ(* 々`ノ(* 々`ノ(* 々`ノ(* 々`ノ
(* 々`ノ(* 々`ノ(* 々`ノ(* 々`ノ(* 々`ノ(* 々`ノ(* 々`ノ(* 々`ノ(* 々`ノ(* 々`ノ(* 々`ノ(* 々`ノ(* 々`ノ(* 々`ノ(* 々`ノ(* 々`ノ
(* 々`ノ(* 々`ノ(* 々`ノ(* 々`ノ(* 々`ノ(* 々`ノ(* 々`ノ(* 々`ノ(* 々`ノ(* 々`ノ(* 々`ノ(* 々`ノ(* 々`ノ(* 々`ノ(* 々`ノ(* 々`ノ
(* 々`ノ(* 々`ノ(* 々`ノ(* 々`ノ(* 々`ノ(* 々`ノ(* 々`ノ(* 々`ノ(* 々`ノ(* 々`ノ(* 々`ノ(* 々`ノ(* 々`ノ(* 々`ノ(* 々`ノ(* 々`ノ
(* 々`ノ(* 々`ノ(* 々`ノ(* 々`ノ(* 々`ノ(* 々`ノ(* 々`ノ(* 々`ノ(* 々`ノ(* 々`ノ(* 々`ノ(* 々`ノ(* 々`ノ(* 々`ノ(* 々`ノ(* 々`ノ
(* 々`ノ(* 々`ノ(* 々`ノ(* 々`ノ(* 々`ノ(* 々`ノ(* 々`ノ(* 々`ノ(* 々`ノ(* 々`ノ(* 々`ノ(* 々`ノ(* 々`ノ(* 々`ノ(* 々`ノ(* 々`ノ
(* 々`ノ(* 々`ノ(* 々`ノ(* 々`ノ(* 々`ノ(* 々`ノ(* 々`ノ(* 々`ノ(* 々`ノ(* 々`ノ(* 々`ノ(* 々`ノ(* 々`ノ(* 々`ノ(* 々`ノ(* 々`ノ
(* 々`ノ(* 々`ノ(* 々`ノ(* 々`ノ(* 々`ノ(* 々`ノ(* 々`ノ(* 々`ノ(* 々`ノ(* 々`ノ(* 々`ノ(* 々`ノ(* 々`ノ(* 々`ノ(* 々`ノ(* 々`ノ
(* 々`ノ(* 々`ノ(* 々`ノ(* 々`ノ(* 々`ノ(* 々`ノ(* 々`ノ(* 々`ノ(* 々`ノ(* 々`ノ(* 々`ノ(* 々`ノ(* 々`ノ(* 々`ノ(* 々`ノ(* 々`ノ
(* 々`ノ(* 々`ノ(* 々`ノ(* 々`ノ(* 々`ノ(* 々`ノ(* 々`ノ(* 々`ノ(* 々`ノ(* 々`ノ(* 々`ノ(* 々`ノ(* 々`ノ(* 々`ノ(* 々`ノ(* 々`ノ
555トシボー:2006/12/04(月) 16:35:52 ID:???
あうー
556ご冗談でしょう?名無しさん:2006/12/04(月) 18:39:08 ID:rJDRT/pi
超伝導コイルの巻き方に『ダブルパンケーキ型に巻く』という記述がありますが、
どういう巻き方なのですか?
http://www.lhd.nifs.ac.jp/result/nifs_news/1992/92mar.html#label1
↑ここが、巻き方について詳しく書かれている数少ないサイトなのですが、
意味がよくわかりません。
もっとわかりやすく説明できる方、回答お願いします!
557ご冗談でしょう?名無しさん:2006/12/04(月) 21:34:50 ID:???
>>550
「沸点」が抜けているので解けない問題になった訳だ。
558ご冗談でしょう?名無しさん:2006/12/04(月) 22:12:21 ID:???
>>556
難しく考えすぎてると思う
蚊取り線香を2枚用意して、一枚を180°回転させるとピタッとあうでしょ?
あの形状だと思うよ
559ご冗談でしょう?名無しさん:2006/12/04(月) 22:17:12 ID:6DAYMtTJ
質問です。
ガリウムリンやヒ素などのバンドギャップは、純度に関わらず一定なのですか?
560ご冗談でしょう?名無しさん:2006/12/04(月) 22:19:10 ID:???
ガリウムリンとかヒ素って言った場合通常100%の物を想定していると思うんだが・・・
合金みたいになれば変わるのは知ってるよな?
561ご冗談でしょう?名無しさん:2006/12/04(月) 22:22:44 ID:6DAYMtTJ
559です。
実は実験でツェナーダイオードとLEDとの立ち上がり電圧の違いを述べる問題がありまして、
「ツェナーダイオードはツェナー降伏を起こすから通常のシリコンよりも純度が低い。
だから、バンドギャップに差が生じる」
って書いたんですよ。
これはどうなのでしょうか?
562ご冗談でしょう?名無しさん:2006/12/04(月) 22:24:42 ID:???
563ご冗談でしょう?名無しさん:2006/12/04(月) 23:15:31 ID:???
>>529
なるほど
プランク時間でもう少し調べてみますね。
ありがとうございました。
564556:2006/12/05(火) 01:12:49 ID:???
>558
蚊取り線香の話で、やっと理解できました!
長い導線の中間を巻枠に固定して、その両側を巻いていく
ということですね?!ありがとうございました!
565ご冗談でしょう?名無しさん:2006/12/05(火) 10:52:45 ID:/N8gsmUK
確率微分方程式
 dX(t)=b(X(t),t)dt+√(h/2m)dW(t)
から、前方フォッカー・プランク方程式
 ∂p(X(t),t)/∂t=−∇・b(X(t),t)p(x(t),t)+(h/2m)(∇^2)p(x(t),t)
を導け。
というのが、数式的にどうしてもつながりません。
多分なんらかのテクニックが必要なんだと思いますがどなたか助けていただけませんか…

ちなみにhは「エイチ・バー」です。mは質量
X(t)は確率過程、bは平均前方速度、W(t)はウィーナー過程、
p(X(t),t)は確率分布密度関数
です。
566ご冗談でしょう?名無しさん:2006/12/05(火) 11:32:20 ID:???
半径aの無限長円柱状導体に電流Iが流れているときの単位長あたりの内部インダクタンスは?
567ご冗談でしょう?名無しさん:2006/12/05(火) 12:25:43 ID:???
中2です。
なんでアルファ崩壊では
中性子や陽子一個づつではなくて
アルファ粒子として飛び出すんですか?
先生に聞いてもなんかはぐらかされます。
568ご冗談でしょう?名無しさん:2006/12/05(火) 12:32:55 ID:???
>>567
中2病の間違えでは
569ご冗談でしょう?名無しさん:2006/12/05(火) 13:04:04 ID:???
>>555 特に必要なテクニックはない。分布関数の発展方程式を微分展開して
その係数をウィーナー過程を表すノイズで平均しろ。3次以上の係数が消える
ことがすぐ出る。係数は微小時間で積分すればよい。
570ご冗談でしょう?名無しさん:2006/12/05(火) 13:24:33 ID:sQgjM+Wp
>>567
中2でα崩壊なんて単語を知ってるだけですごいと思うけどな。
だがしかし、α粒子ってなんなのか知ってるのか?
571ご冗談でしょう?名無しさん:2006/12/05(火) 14:37:42 ID:???
量子力学の調和振動子について質問です。
エルミートの微分方程式を、級数解の方法で解いたところ、漸化式が得られて
そこからE=hω(n+(1/2))というのは得られたのですが、漸化式からエルミート関数を導く方法が分かりません。
教科書だとエルミート関数が与えられているのですが、漸化式から求めるのは不可能なのですか?
572ご冗談でしょう?名無しさん:2006/12/05(火) 15:17:06 ID:???
>>571
漸化式を高校でやったようにan=なになにって
一般項にすれば良いんじゃない?
573ご冗談でしょう?名無しさん:2006/12/05(火) 16:42:52 ID:???
>>567

中性子1個や陽子1個の状態はかなりエネルギーが高いから
574ご冗談でしょう?名無しさん:2006/12/05(火) 17:21:03 ID:???
>>573
んなこたない

第0近似として、陽子の質量をmとすれば
中性子1個or陽子一個のエネルギー=mc^2
アルファ粒子のエネルギー=4mc^2
575ご冗談でしょう?名無しさん:2006/12/05(火) 17:22:35 ID:n7QViLTD
>>571 本当に教科書ではいきなりエルミート関数が出てきたの?
固有関数をf(x)として定番だけどxが無限大にいくときf(x)が0に近付くべき、
って条件なかった?そうすると漸化式にでてくるエネルギー固有値の様子から、
量子数が半整数でないと、そこからの寄与がいつまでも残ってしまって、問題の
級数は無限大まで定義出来るどころか収束半径1になってしまう。勿論実軸上では
収束半径越えた部分で0に収束するかもしれないので、普通はこのスレの数日
前に出てきた疑問にあったように、問題の方程式がxの大きい時、固有値を無視出来る、
ということから近似解としてガウス関数を持ってきて、厳密な解をそれと何か
別の関数の積という形に問題を焼直して解きますが十分に納得いく論法と思いませんか?
ある圧倒的な領域において答えが簡単な関数で近似出来そうな時、正しい答えを
その関数とべつの関数の積、もしくは和の形に仮定して真の答えを求めようとする、
のは常套手段です。
576ご冗談でしょう?名無しさん:2006/12/05(火) 17:49:47 ID:???
>>570 >>573 >>574
ありがとうございます。
いろいろ検索してみたんですが、
まだわかってません。

アルファ崩壊で中性子や陽子の結合を切るのに
真っ二つでなく、また1つずつでもなく
2個と2個のかたまり(1個と1個でもなく)の粒子で飛び出すのが不思議です

陽子と中性子をくっつけてる電子みたいのがあるからまとまっている方が
安定しているということもあると思うんですが

身の回りには水素原子の方がたくさんあるみたいなので
それなら水素の原子核と同じ陽子が飛び出てもいいんじゃないかとも思って・・・
577ご冗談でしょう?名無しさん:2006/12/05(火) 18:20:33 ID:???
>>572
関数に微分の形が入っているので、一般項を求められそうにありません。

>>575
>問題の方程式がxの大きい時、固有値を無視出来る、
>ということから近似解としてガウス関数を持ってきて、厳密な解をそれと何か
>別の関数の積という形に問題を焼直して解きますが
僕が疑問なのは、それ以降の部分です。
φ=H(ξ)e^(-ξ^2/2)という形にまでは、仰る論法で求めたのですが、
Hの具体的な形が求められないのです。
ベッセル関数なんかの場合は、漸化式から一般項を求めて、係数が求まるのですが
エルミートの場合は上に書いたように、n階微分の形が入ってとても複雑なので
書き下したくらいでは推測できないような気がするのですが・・・。
578ご冗談でしょう?名無しさん:2006/12/05(火) 18:31:06 ID:???
>>574

おいおい、なぜ核子あたりで考えない?
579ご冗談でしょう?名無しさん:2006/12/05(火) 18:37:39 ID:???
>>576
ヘリウムの原子核(アルファ粒子)の質量は、>>574の書き方で言うと4mよりちょっと小さい。
理科年表見てみるとわかるが、4Heは軽い原子核の中では例外的なほどに、核子一個あたり
の質量欠損がでかい核だ。陽子2個と中性子2個出すより、4He1個出す方が、克服すべき
エネルギー障壁が低い。

580ご冗談でしょう?名無しさん:2006/12/05(火) 18:50:22 ID:???
理科年表面白いよなあ・・・。ずっと読んでいられる。
581ご冗談でしょう?名無しさん:2006/12/05(火) 20:00:11 ID:???
>>576は質量欠損はわかるのだろうか…
わかってなくても、すぐ理解できそうな雰囲気だけど
582ご冗談でしょう?名無しさん:2006/12/05(火) 20:01:36 ID:???
グランドカノニカルを大分配って訳すのおかしくね?
完全分配が正しいんじゃね?
583ご冗談でしょう?名無しさん:2006/12/05(火) 20:26:39 ID:???
>>582
じゃあ、ミクロカノニカルは不完全分配?

>>579 >>581
質量欠損なんて言わずに、1核子結合エネルギーが大きいと言えば
いいんじゃない。

>>576
要するに、陽子2個中性子2個の組み合わせは結合が堅いと言うこと。

>陽子と中性子をくっつけてる電子みたいのがあるからまとまっている方が
>安定しているということもあると思うんですが
くっつけているのは電子じゃなくて中間子だが、まとまっている方が安定
というのは正しい。ただあんまりまとまりすぎると、陽子同士の斥力が
大きくなってしまうので、陽子2個中性子2個がちょうど良い組み合わせ
と言うこと。

>身の回りには水素原子の方がたくさんあるみたいなので

宇宙誕生のときに水素が大量に作られたからね。
584ご冗談でしょう?名無しさん:2006/12/05(火) 20:38:28 ID:cmJUXfkG
http://ime.nu/www.youtube.com/watch?v=hYdlE3TiVDY

この問題の答えを教えてください。
585ご冗談でしょう?名無しさん:2006/12/05(火) 20:43:15 ID:???
見たけどどこが問題なんだ??

で、とりあえず、>>1をよんどけ>>584
586584:2006/12/05(火) 21:20:11 ID:cmJUXfkG
>>585以外の人で、分かる人教えて下さい。
587ご冗談でしょう?名無しさん:2006/12/05(火) 21:21:10 ID:???
自分で作って見ればわかるよ。簡単にはずれるから。
588ご冗談でしょう?名無しさん :2006/12/05(火) 21:24:29 ID:n3IopFRG
私の身体を作ってるカルシウムとかって、いったいいつ出来たんでしょうか?
地球ができるずっと前、既に死んじゃった恒星の核の中で出来たことは確実
だと思うんですが、
それ以上の情報って、わかる方法ってあるんでしょうか?
589ご冗談でしょう?名無しさん:2006/12/05(火) 21:25:17 ID:???
590ご冗談でしょう?名無しさん:2006/12/05(火) 21:26:00 ID:???
>それ以上の情報

具体的に何が知りたいのかさっぱりだ
591ご冗談でしょう?名無しさん:2006/12/05(火) 21:27:55 ID:???
>>589
よくもまぁ短い時間に同じ話がループできるもんだな。
592ご冗談でしょう?名無しさん :2006/12/05(火) 21:28:11 ID:n3IopFRG
>>590
「私の身体を作ってるカルシウムはいつ出来たのか?」
では質問になりません?
593ご冗談でしょう?名無しさん:2006/12/05(火) 21:29:57 ID:???
>>592
お前の体の中にあるカルシウム原子1個が
いつカルシウム原子になったかか?

誰にも分からんw
594ご冗談でしょう?名無しさん :2006/12/05(火) 21:34:15 ID:???
>>593
恒星の核内での反応がある程度わかっているのなら、
1)特定の原子番号の原子に対して、同時に生成される同位体の比率が
あるレンジで予想できないのか?
2)その場合、崩壊の半減期等から現在の地球上に存在する、原子の生成
時期の推定は可能ではないのか?

に変えても意味不明でしょうか?
(ちなみに専門ではないので、1)、2)に関する情報は全然知りません)
595ご冗談でしょう?名無しさん:2006/12/05(火) 21:40:08 ID:???
同時期に出来たカルシウムを持ってくれば分かるかもね

で、カルシウムは全部同時に出来るのか??
596ご冗談でしょう?名無しさん :2006/12/05(火) 21:43:59 ID:n3IopFRG
>>595
そう、それも大きな疑問なのです。
私の身体の原子は一つの恒星産の原子で全部できてるのか?
それとも色んな星の中味が混じってるのか? 
597ご冗談でしょう?名無しさん:2006/12/05(火) 21:45:44 ID:???
そもそも一つの恒星でも何十億という時間を掛けて核融合とかしてて、
その生成量も一定かどうかわからないから半減期を考えるのにもノイズが多くて、
さらにいえば、いつできたかなんて絶対に分からんわな。
598ご冗談でしょう?名無しさん:2006/12/05(火) 21:46:18 ID:???
>>596
その辺は分からないと言うほか無いな
可能性を考えればいろいろ考えられるが
特定する手段がない
599ご冗談でしょう?名無しさん:2006/12/05(火) 21:49:27 ID:???
宇宙に棒が浮いています。その棒の端に棒に垂直に力をかけると棒の重心のを中心とした回転運動をしながら並進運動をしますか?
それとも回転せずに並進運動をしますか?
もし回転するなら、なぜですか?
600ご冗談でしょう?名無しさん:2006/12/05(火) 21:51:18 ID:???
>>599
運動方程式って知ってる?
601599:2006/12/05(火) 21:58:00 ID:???
N=Iβですか?
602ご冗談でしょう?名無しさん:2006/12/05(火) 21:59:01 ID:???
慣性モーメント知ってるなら自分で計算できるよね?
どういう運動するか。
603ご冗談でしょう?名無しさん:2006/12/05(火) 22:04:28 ID:qCHUroGQ
純度の低いガリウムヒ素は、
100%のそれに比べてバンドギャップは
あがるんでしょうか?
あがるとしたら、それはどうしてでしょうか?
こういう単純な質問は、どこにも書いていないので困っています。。。
604ご冗談でしょう?名無しさん:2006/12/05(火) 22:06:16 ID:???
混ぜるものによるかな。
不純物半導体なんかはバンドギャップの間に一段電子の入れる場所をつくって電子が流れ易くなるんじゃなかったっけ。
605ご冗談でしょう?名無しさん:2006/12/05(火) 22:08:32 ID:???
606ご冗談でしょう?名無しさん:2006/12/05(火) 22:16:15 ID:KCL/ydgq
もとの白色光のスペクトル、すなわち単色光の中に黄色はあるが、マゼンタはない
この理由について考察せよ

という課題なのですが
「黄色は色でもあり光でもあるが、マゼンタは青と赤の光の混色による色であり、マゼンタという単色光はない」
こういう答え方で合ってるでしょうか_
607599:2006/12/05(火) 22:16:17 ID:???
>>600
分かりません・・・
この例の場合、偶力はないけれど中心などの回りを考えると力のモーメントがあるけど、偶力じゃなくてただの力のモーメントでも回転運動は起こるのでしょうか?
608ご冗談でしょう?名無しさん:2006/12/05(火) 22:18:22 ID:???
609ご冗談でしょう?名無しさん:2006/12/05(火) 22:19:23 ID:???
>>606
マゼンタの定義って何だろう
それによるよね
610ご冗談でしょう?名無しさん:2006/12/05(火) 22:35:27 ID:KCL/ydgq
マゼンタって赤紫色ですよね
赤紫の波長って赤よりさらに長い波長域の750-800あたりにありますよね
この課題の意図がなんなのかイマイチわかりません
611ご冗談でしょう?名無しさん:2006/12/05(火) 22:38:17 ID:???
光の三原色が足されて白のはずなのに
マゼンタがないのは何故?

と言う問いだと思われ
612ご冗談でしょう?名無しさん:2006/12/05(火) 22:48:10 ID:???
赤と青を混ぜるとなぜ紫になるのですか?
光の周波数で説明つくのかな?
613ご冗談でしょう?名無しさん:2006/12/05(火) 22:49:11 ID:???
つかないよ
614ご冗談でしょう?名無しさん:2006/12/05(火) 22:56:38 ID:???
>>577おねがいします。
615ご冗談でしょう?名無しさん:2006/12/05(火) 22:57:23 ID:???
マルチにも催促馬鹿にも答えるつもりは無いのだ。
616ご冗談でしょう?名無しさん:2006/12/05(火) 22:59:03 ID:???
エルミート多項式なんてy=Σx^nっておいてΣ使って書いておけば良いんじゃね。
617ご冗談でしょう?名無しさん:2006/12/05(火) 23:02:51 ID:???
>>606
物理と言うより生物の問題のような。
618ご冗談でしょう?名無しさん:2006/12/05(火) 23:03:53 ID:???
そもそもマゼンダって色が分からない。
白色光のスペクトルの中に無いのはある色とある色の混合だからじゃね。
619ご冗談でしょう?名無しさん:2006/12/05(火) 23:17:32 ID:qCHUroGQ
>>605
調べたんだから丸投げじゃないじゃんか
620ご冗談でしょう?名無しさん:2006/12/05(火) 23:20:23 ID:???
>>619
その足りない脳みそで足りないなりに考えた予想などを述べたら
答えてやらないでもない。
621ご冗談でしょう?名無しさん:2006/12/05(火) 23:21:01 ID:???
ついでに言わせてもらえば

開き直りって人として如何なものかと。
622ご冗談でしょう?名無しさん:2006/12/05(火) 23:22:25 ID:???
私の友達が
「外界からの影響を完全に遮断した安定した物質に、重力による圧力という
運動エネルギーをかけ続けたら、重力があり続けることを条件に半永久的に
熱エネルギーが得られる。」と言ってるんですが、検証よろ。

「重力の運動エネルギーを圧力により熱エネルギーに変換しているから
物質の消費も全くなく熱エネルギーだけを得られる」という事らしいっす。
(熱エネルギーの大小は考えないものとする。)

私は「外界からの影響を完全に遮断した段階で熱エネルギーの放出が不可能」
または、「熱エネルギーを放出できるとしたら電磁波などの消失により
[安定した物質]の質量が減少するので半永久的は不可能」だと
思っているのですが。

すみません。別スレにも書きました‥ごめんなさい。
623ご冗談でしょう?名無しさん:2006/12/05(火) 23:23:53 ID:???
>>622
マルチ死ねボケ。
624ご冗談でしょう?名無しさん:2006/12/05(火) 23:25:06 ID:???
>>623
だから謝ってンじゃん!
625ご冗談でしょう?名無しさん:2006/12/05(火) 23:25:07 ID:???
>>622
物理の前に文体の勉強して来い
626ご冗談でしょう?名無しさん:2006/12/05(火) 23:26:49 ID:???
>>624

んじゃ、あとから謝るとして>>624の両親頃してくるわ。
627ご冗談でしょう?名無しさん:2006/12/05(火) 23:31:11 ID:???
>>626
そのレスは人間としてどうよ。
628ご冗談でしょう?名無しさん:2006/12/05(火) 23:33:14 ID:jKqHJU2t
>>567
これは君が本当のことを理解するには
段階がいるんだよ
もしかすると中学の先生は物理の専門じゃないから
知らないかもしれないことなんだよ

簡単に言うとα粒子ってヘリウム原子核のことなんだけど
このこのヘリウム原子核の状態が他の粒子より飛び出しやすい
状態になるからなんだ
なぜ飛び出しやすいかって?それはね、トンネル効果でぐぐってみてね
629ご冗談でしょう?名無しさん:2006/12/05(火) 23:40:28 ID:???
>>628
的をはずした回答だなぁ
567が知りたいのは出てくるのが核子一個やもっとでかい原子核じゃなくてα粒子である事の理由だろ
トンネル効果で説明できる事じゃねぇぞ
630ご冗談でしょう?名無しさん:2006/12/05(火) 23:50:08 ID:uR68oHWO
N が  [kg・m/s^2]  なのは何でですか??
631ご冗談でしょう?名無しさん:2006/12/05(火) 23:50:20 ID:???
>>624
マルチを謝るなら、どこにマルチしたか晒さないと、
黙ってマルチしたのと結果的に変わらない。
632ご冗談でしょう?名無しさん:2006/12/05(火) 23:50:49 ID:jKqHJU2t
>>629
じゃあ格子あたりの結合エネルギーが他に比べて安定で
なおかつパリティ的にも安定してるから
て答えればいいのか
中学生相手にナンセンスだな
633ご冗談でしょう?名無しさん:2006/12/05(火) 23:52:19 ID:???
>>630
定義
634ご冗談でしょう?名無しさん:2006/12/05(火) 23:53:17 ID:FABz9d3K
電球の電圧と電流を測っていたのですが、電流計の最大表示を100mAから1000mA
にするといっきにいままでより電圧があがったのですが、どうしてでしょうか?
635ご冗談でしょう?名無しさん:2006/12/05(火) 23:53:24 ID:???
>>631
やだねっ♪
636ご冗談でしょう?名無しさん:2006/12/05(火) 23:54:07 ID:jKqHJU2t
あとは
α粒子は電価小さいからクーロン力や表面張力からの脱出がしやすい
とかか
637ご冗談でしょう?名無しさん:2006/12/05(火) 23:56:24 ID:???
>>632
トンネル効果はナンセンス以前に的外れだから。
638ご冗談でしょう?名無しさん:2006/12/05(火) 23:58:37 ID:jKqHJU2t
>>637
たしかにトンネル効果はなぜアルファ粒子なのかという問いに対しては
答えてないな
639ご冗談でしょう?名無しさん:2006/12/06(水) 00:01:33 ID:???
指数の肩のエネルギー積分が全然大きさ違うから
ああなるほどα粒子だけなんだって納得して読んでたのは俺だけかw
640ご冗談でしょう?名無しさん:2006/12/06(水) 00:05:38 ID:zGgMVfNk
>>639
分かりやすく説明をお願いします。
641ご冗談でしょう?名無しさん:2006/12/06(水) 00:10:10 ID:???
>>629
 説明の途上でトンネル効果は出てくるがな。
 ま、>>628はえらそうなわりには外しまくっているという点には同意。
642ご冗談でしょう?名無しさん:2006/12/06(水) 00:13:05 ID:zGgMVfNk
粛々と反省します







という具合に先生もなるかも
643ご冗談でしょう?名無しさん:2006/12/06(水) 00:17:04 ID:???
「停留させる」って英語でなんてゆうの?例えば「作用積分を停留させる」みたいな
644ご冗談でしょう?名無しさん:2006/12/06(水) 00:17:54 ID:???
645ご冗談でしょう?名無しさん:2006/12/06(水) 00:18:26 ID:???
停留点stationary point・停留値stationary value
646ご冗談でしょう?名無しさん:2006/12/06(水) 00:22:18 ID:???
「極値にする」という意味ではextremizeかな。
647ご冗談でしょう?名無しさん:2006/12/06(水) 00:30:00 ID:???
>>644
すいません。携帯からなのでpcサイトみれないでつ

>>645
ありがとうございます。動詞はわかりますか?

>>646
極値と同義語にしちゃってもいいんですかね?
648ご冗談でしょう?名無しさん:2006/12/06(水) 00:51:53 ID:???
はーい、次の方のご質問をどうぞ。
649ご冗談でしょう?名無しさん:2006/12/06(水) 01:07:15 ID:WOV73vTr
高校物理からの質問です

光波の分野で薄膜(シャボン玉の膜)による光の干渉実験が教科書で紹介されています
いきなり質問ですが
なぜ”薄”膜でないといけないのですか??
650ご冗談でしょう?名無しさん:2006/12/06(水) 01:09:23 ID:???
別に薄い必要はないが
膜の厚さの幅が少なくないといけない
651ご冗談でしょう?名無しさん:2006/12/06(水) 01:09:26 ID:???
薄膜じゃない条件で考えてごらん。
652ご冗談でしょう?名無しさん:2006/12/06(水) 01:09:46 ID:???
厚い膜より薄い膜のほうが厚みが安定してるから
653ご冗談でしょう?名無しさん:2006/12/06(水) 01:12:42 ID:WOV73vTr
かなり知識が乏しいので
もう少し詳しい説明をつけてもらえると助かります
654ご冗談でしょう?名無しさん:2006/12/06(水) 01:13:21 ID:???
655ご冗談でしょう?名無しさん:2006/12/06(水) 01:26:16 ID:???
>>649
>>653

マルチ死ねボケ。
656ご冗談でしょう?名無しさん:2006/12/06(水) 01:26:38 ID:???
>>653
まず薄膜の実験を理解しろ
話はそれからだ
657ご冗談でしょう?名無しさん:2006/12/06(水) 02:25:02 ID:SAwQUpDR
質問です。
密度が一様で半径r、中心角2θの
円弧の重心をもとめる問題がわかりません。

アフォな私に教えてください。
658ご冗談でしょう?名無しさん:2006/12/06(水) 02:28:13 ID:???
659ご冗談でしょう?名無しさん:2006/12/06(水) 02:28:29 ID:???
>>657
 重心の定義に従って積分せよ。
660ご冗談でしょう?名無しさん:2006/12/06(水) 02:28:46 ID:b+NPeho7
どうせ重心は対称な線上だ。てか積分しろ。
661ご冗談でしょう?名無しさん:2006/12/06(水) 02:33:52 ID:SAwQUpDR
教科書に載っている公式の通りやろうとしているんですが。。。

積分する前のdsをどうおけばいいかわからないんです。
662ご冗談でしょう?名無しさん:2006/12/06(水) 02:35:54 ID:???
とりあえず円と3角形でやってみろ
663ご冗談でしょう?名無しさん:2006/12/06(水) 02:41:35 ID:SAwQUpDR
よくわかりません。
ds=rdθですか?

664ご冗談でしょう?名無しさん:2006/12/06(水) 07:38:02 ID:???
dsってなんだ・・・・平面図形なんだから、積分も2次元のはずだぞ。>>661
665ご冗談でしょう?名無しさん:2006/12/06(水) 10:24:25 ID:???
アンサンブル平均というものがよく分かりません。
具体例をあげて説明していただければ幸いです。
666ご冗談でしょう?名無しさん:2006/12/06(水) 11:27:56 ID:Ju9yh8lB
検索しても見つからなかったのですが、おすすめの本スレみたいなものはありますか?
なければ工学向けの電磁気学の参考書でおすすめがあったら教えてください。
667ご冗談でしょう?名無しさん:2006/12/06(水) 11:29:48 ID:???
>>666
大学生のための参考書・教科書Pt.24
http://science4.2ch.net/test/read.cgi/sci/1159791010/
668ご冗談でしょう?名無しさん:2006/12/06(水) 11:34:51 ID:???
>>667
誘導ありがとうございます
669ご冗談でしょう?名無しさん:2006/12/06(水) 11:38:07 ID:szn+GCwh
電界強度を電界Eベクトル=(Ex,Ey,Ez)から求める方法を教えて下さい。
670ご冗談でしょう?名無しさん:2006/12/06(水) 11:51:17 ID:???
>>669

ベクトルの大きさが求められない人ですか?
671ご冗談でしょう?名無しさん:2006/12/06(水) 12:00:12 ID:szn+GCwh
>670
電界強度=SQRT(Ex^2+Ey^2+Ez^2) [V/m] でいいですか?
672ご冗談でしょう?名無しさん:2006/12/06(水) 12:17:13 ID:d7yNX1KS
semiログのグラフ用紙の使い方教えてください
673ご冗談でしょう?名無しさん:2006/12/06(水) 12:51:21 ID:AsKWzT7x
[m/t],[m*F]等でなく、
[m^t]のように、単位で指数というのは、あるのですか?
674ご冗談でしょう?名無しさん:2006/12/06(水) 12:59:01 ID:???
ない
675ご冗談でしょう?名無しさん:2006/12/06(水) 13:03:51 ID:???
意味がよく分からんが、面積は違うのか?
676ご冗談でしょう?名無しさん:2006/12/06(水) 13:04:42 ID:???
ああ単位を持つ量が指数にのるってことか。ならない。
677ご冗談でしょう?名無しさん:2006/12/06(水) 13:54:36 ID:AsKWzT7x
事象:2秒で4m動いた。

4/2[m/s]
この事象を上式で表現し、速度と名付けた。

4^2[m^t]
俺はこの事象を上式のように定義したよ。名前は付けてないけど。
この定義は、まったく無意味なの?
678ご冗談でしょう?名無しさん:2006/12/06(水) 14:05:36 ID:???
まったく無意味
679ご冗談でしょう?名無しさん:2006/12/06(水) 15:09:26 ID:/2d7IFky
「水深70mの水底に生じた気泡が水面に浮上したとき、直径が6cmになっ
た。気泡内の空気の変化を理想気体の等温変化と仮定すると、水底で生
じたときのこの気泡の直径はいくらか。ただし、気泡は球状であり、水
の密度は1000kg/m3、大気圧を100kPa、重力加速度を10m/s2とする。」
って問題が出てしまったんだけど、さっぱりわからないのでだれかさわ
りだけでも教えて下さい。
680ご冗談でしょう?名無しさん:2006/12/06(水) 15:21:20 ID:XO/fDnIf
独学で物理をやってる高1です。
電位 電位差の概念がよく分かりません。
教えて下さい
681ご冗談でしょう?名無しさん:2006/12/06(水) 15:28:44 ID:???
力学で言う高さみたいなもんだと思えばいいんじゃね?
682ご冗談でしょう?名無しさん:2006/12/06(水) 16:01:22 ID:???
>>677
他のものとの関連性がないなら無意味
683ご冗談でしょう?名無しさん:2006/12/06(水) 16:31:38 ID:szn+GCwh
>677
無意味だとは思わないが、その定義で表した量を何に使うのか?
684ご冗談でしょう?名無しさん:2006/12/06(水) 16:37:24 ID:???
>>677
そういったあらわしかたをすることによって、
何らかの現象を今よりも簡潔に説明で切るならば、
意味が有る。
685ご冗談でしょう?名無しさん:2006/12/06(水) 17:13:36 ID:uxN4B0bI
電気・電子板から誘導されてきました。

β線って放射線の一種ですよね?
ということは人体に電気を流していると被爆する可能性があるということなのでしょうか?

電気・電子板で質問したところβ崩壊に起因しない電子はβ線とは呼ばれないそうなのですが
どう違うというのでしょうか?
どうかよろしくお願い致します。
686ご冗談でしょう?名無しさん:2006/12/06(水) 17:30:44 ID:???
エネルギーが違う。>>685

687ご冗談でしょう?名無しさん:2006/12/06(水) 17:42:58 ID:???
>>685
>β崩壊に起因しない電子はβ線とは呼ばれないそうなのですが
>どう違うというのでしょうか?
放射線は基本的に発生機構で分類されている。
ベータ崩壊はエネルギースペクトルが広がるが、
電子を加速して放出した電子線はエネルギースペクトルがある値に収束するという特徴がある。

>人体に電気を流していると被爆する可能性があるということなのでしょうか?
放射線による被爆はそれに当たった時に電離して行くから。
電流の場合、電離するほど強烈なエネルギーを流したら多分消滅するんじゃね?
688ご冗談でしょう?名無しさん:2006/12/06(水) 17:49:07 ID:V3sMNtHv
私は>>677 = >>673 です。

回答してくれた>>682-684ら、ありがとう。
>>677の定義は、意味が無いと思います。
やはり、>>673のような単位が指数になる事は、
ないのですね。
689ご冗談でしょう?名無しさん:2006/12/06(水) 17:52:11 ID:zGgMVfNk
β線を人体に当てるのはできるけど
電流として流すのは無理
690ご冗談でしょう?名無しさん:2006/12/06(水) 19:13:29 ID:aPCNJqOv
>>688
ほかにも対数log(x)や三角関数sin(x)などもも無次元じゃないとだめ。
マクローリン展開すると1+x+x^2+x^3+・・・の形になるでしょ、
このときxは無次元じゃないとだめだべ
691ご冗談でしょう?名無しさん:2006/12/06(水) 19:40:55 ID:5hRbKWMr
>>690 logは「定数だけの移動」で単位の呪縛から逃げられる場合が
あるよ。又、熱力学的温度なんて言うのもその例だなあ。本質的に正の数だけで表され、
その比が意味を持つような量は単位何てどうでも良い。古典統計力学になると、
温度はボルツマン定数を決めちゃったら、動かしようだ無くなるが、状態和の
定数因子はどうでも良いので相変わらずlogに対する、独立変数の無次元性は
どうでも良くなる。で、量子論までくるともう無視出来なくなるねえ。
692ご冗談でしょう?名無しさん:2006/12/06(水) 19:52:06 ID:???
>放射線は基本的に発生機構で分類されている。

これって宇宙の果てでのβ崩壊から出たβ線はどんなに赤方偏移してもβ線って事?
693ご冗談でしょう?名無しさん:2006/12/06(水) 20:17:01 ID:gdwl/SqG
 MRI装置を動かす事になりました。磁束密度は1.5テスラです。酸素(鉄の)ボンベなんかガントリーの近くに置くとすっ飛んで
行くそうです。例えば1KGの鉄球を1.5テスラの磁束密度内に入れた場合はどの位の加速度を生じますか?
磁束密度1テスラ=Wb ・ m -2 と 磁性体の鉄のKGと 力ニュートンの簡単な式があったら教えて下さい。
お願い致します。
694ご冗談でしょう?名無しさん:2006/12/06(水) 20:27:28 ID:haDd9bkP
電球の電圧と電流を測っていたのですが、電流計の最大表示を100mAから1000mA
にするといっきにいままでより電圧があがったのですが、どうしてでしょうか?
695690:2006/12/06(水) 20:49:08 ID:aPCNJqOv
>>691
あ、そうなんですか。ありがとうございます。
logの中身の次元については少し授業で聞いてはいましたが
まだよく分かってませんでした。
696693:2006/12/06(水) 21:12:58 ID:gdwl/SqG
 鉄球は純粋な鉄とします。そして磁束はかなり大きな物で、鉄球に作用する磁束密度は
一定だと仮定します。お願いですから回答を宜しくお願いいたします。
鉄球で無くとも、磁束密度に対して垂直に向かい合う正立方体の鉄でもいいです。
 勿論前提条件があっての回答でいいので、磁束密度と吸着力の簡単な式を教えて下さい。
宜しくお願いいたします。m(_ _)m
697ご冗談でしょう?名無しさん:2006/12/06(水) 21:13:28 ID:???
649 ご冗談でしょう?名無しさん 2006/12/06(水) 01:07:15 ID:WOV73vTr
高校物理からの質問です

光波の分野で薄膜(シャボン玉の膜)による光の干渉実験が教科書で紹介されています
いきなり質問ですが
なぜ”薄”膜でないといけないのですか??


650 ご冗談でしょう?名無しさん sage 2006/12/06(水) 01:09:23 ID:???
別に薄い必要はないが
膜の厚さの幅が少なくないといけない


651 ご冗談でしょう?名無しさん sage 2006/12/06(水) 01:09:26 ID:???
薄膜じゃない条件で考えてごらん。


652 ご冗談でしょう?名無しさん sage 2006/12/06(水) 01:09:46 ID:???
厚い膜より薄い膜のほうが厚みが安定してるから


どこで誘導されてるんだ?w
698ご冗談でしょう?名無しさん:2006/12/06(水) 21:14:08 ID:???
誤爆したorz
699ご冗談でしょう?名無しさん:2006/12/06(水) 21:15:26 ID:???
>>696

空間的に均一な磁場中に磁性体を置いても力は受けない。
(磁化の向きの関係で回転させるような力は受けるけど)
だから磁場勾配がわからんとどうにもならん。
700ご冗談でしょう?名無しさん:2006/12/06(水) 21:26:58 ID:zGgMVfNk
法律では空気を電離・励起する能力があると放射線になる。
701693:2006/12/06(水) 21:30:05 ID:???
>>699
 どうも有り難う御座います。
磁場の勾配がなければ、力は働きませんよね。
調べてきます、済みませんでした。
702693:2006/12/06(水) 21:42:12 ID:gdwl/SqG
 磁束密度1.5テスラを均等に発生する平面の物質があり、そこに純粋な平面の鉄がくっついた
場合のはがす力はどうなるのでしょうか?簡単な条件下の場合でいいので教えて下さい。
703ご冗談でしょう?名無しさん:2006/12/06(水) 21:44:47 ID:???
>>702
電磁気と磁性体の教科書読め
704693:2006/12/06(水) 21:49:03 ID:???
 何故私がこんな事を聞くかと言えば、磁束に対して電流を流した場合の電線に働く力しか習ってないと思うからです。
705ご冗談でしょう?名無しさん:2006/12/06(水) 21:52:08 ID:???
この流れで思いついたんだけど
茶筒みたいな円筒の底に磁石をつけておく

鉄に仕事をさせながら磁石にくっつける

円筒の側面を冷やして超伝導にして磁束を通さなくする

磁場の勾配が無くなったところで鉄を剥がして元の位置に戻す

側面を温めて超伝導じゃなくする

鉄に仕事をさせながら磁石にくっつける

これって永久機関じゃね?
706693:2006/12/06(水) 21:52:59 ID:???
>>703

 スマソ、磁場と磁性体の本を買って読みます。
707ご冗談でしょう?名無しさん:2006/12/06(水) 21:54:43 ID:???
>>705
仕事させたり暖めたりしまくりじゃんw
708ご冗談でしょう?名無しさん:2006/12/06(水) 21:57:25 ID:???
冷やすときに得られるエネルギーと温めるのに必要なエネルギーが等しいから
鉄が磁石にくっつくときに取り出した仕事がゼロから湧き出てるように見えるって話だろ
709693:2006/12/06(水) 22:15:39 ID:???
 私は、診療放射線技師で、もー40才になります。
それで初めてMRI装置を使用する事になるのです。
その装置は最高で磁束密度は1.5テスラを叩きだします。
 ガントリー内の患者に向かって行く磁性体をMRI室に入れたら大変な事になり、アメリカでは間違って鉄製の酸素ボンベを
入れて患者と衝突して患者が死んだ事例があります。
MRI装置に剥がれない様な質量の磁性体をくっつけてしまったら、画像に影響するので超伝導に近い液体ヘリウム−269度を
気化させて剥がすしかないのです。スンマソ、つまらない私事を言いまして。
710ご冗談でしょう?名無しさん:2006/12/06(水) 22:30:18 ID:???
>>702

完全に無限平面だと磁場勾配が生じないから力は要らんわな。

強磁場中では(強磁性の)鉄はほぼ飽和磁化。
磁化に磁場をかければエネルギー。
無限遠(ゼロ磁場)からある磁場の所まで持ってくるときのトータルエネルギーは
このエネルギーになる。局所的な力は磁場勾配による。
鉄の飽和磁化は適当なデータブックでも見ればよし。鉄の種類によってだいぶ
違うけど。
711ご冗談でしょう?名無しさん:2006/12/06(水) 22:35:23 ID:???
>>710

本当に勉強になります。有り難う御座いました。
712ご冗談でしょう?名無しさん:2006/12/06(水) 22:37:52 ID:???
平面が広かろうと狭かろうと鉄と空気の透磁率が違う以上磁場の勾配はあるだろ
713ご冗談でしょう?名無しさん:2006/12/06(水) 22:41:50 ID:???
>>700
いちおう、法律上も電離放射線という言葉でそれは区別してるんじゃないの?
714ご冗談でしょう?名無しさん:2006/12/06(水) 22:58:18 ID:zGgMVfNk
>>713
そうだっけ?
定義してそれについて法律をつらつら書いてあるからな
区別という意味になるか
715ご冗談でしょう?名無しさん:2006/12/06(水) 23:07:26 ID:uFvPbOBM
すみません、実験でβ線のエネルギーを出したのですが
ここから、β線が光速に対してどのくらいの速度なのかを
求めようとしています。

電子の質量が分かっているので 1/2m(v^2)=Eから出そうとしたのですが
これでは、光の速度を超えてしまいます。
しかし、光のE=hνを使うと、波長が分からないのでどうしようもありません。

どうすればいいのでしょうか。
どなたか教えていただければ幸いです。
716ご冗談でしょう?名無しさん:2006/12/06(水) 23:09:57 ID:???
ド・ブロイさんに聞け
717ご冗談でしょう?名無しさん:2006/12/06(水) 23:10:12 ID:???
相対性理論の本を読め
718ご冗談でしょう?名無しさん:2006/12/06(水) 23:17:39 ID:uFvPbOBM
>>716
ド・ブロイ波ですね。
なんとかやってみます。
ありがとうございました。
719ご冗談でしょう?名無しさん:2006/12/06(水) 23:24:30 ID:haDd9bkP
電球の電圧と電流を測ろうとして、交流電圧を0から100Vにする実験なのですが
最初の電圧がテスターで測るとどうしても0にならず、0、5Vほどになってしまうのですが、
どうしてでしょうか?
720ご冗談でしょう?名無しさん:2006/12/06(水) 23:27:04 ID:???
条件が全くわからんので何とも言えないw
ちなみにショートさせると何ボルトになるの
721ご冗談でしょう?名無しさん:2006/12/07(木) 00:06:46 ID:???
>>712

多分鉄板も無限で、そうなると磁場勾配は無い気がする。
722ご冗談でしょう?名無しさん:2006/12/07(木) 00:10:47 ID:???
無限なら、勾配は無いよ。
723ご冗談でしょう?名無しさん:2006/12/07(木) 01:13:26 ID:KvnEevW6
ペガサス座の51番星のスペクトル線が周期4.23日、速度振幅約60m/sで変動している。
真空中での光の速さを3*10^8m/sとする。波長600.0nmの吸収線の変動の振幅は波長にしてどれだけか
有効数字2桁で答えよ


60m/sだったらβ=v/cが2*10^-7だから小さすぎてドップラー効果起きない→変動しないと思うんですけど、
どうでしょう
星自体でなくてスペクトル線が60m/sというところもよくわかりません
724ご冗談でしょう?名無しさん:2006/12/07(木) 01:50:39 ID:AHBicha5
ポテンシャルψの-gradψが力として現れるのですよね。
では、たとえば、ポテンシャル内を運動している質点は、必ずしも最大勾配をとらないと思うのですが
これはどのように解釈すれば良いのでしょう。
725ご冗談でしょう?名無しさん:2006/12/07(木) 01:53:47 ID:???
>>724
言いたいことが分からん
726ご冗談でしょう?名無しさん:2006/12/07(木) 02:17:43 ID:AHBicha5
>>725
あるポテンシャルのなかで静止している(たとえば重力ポテンシャルψにおいて)
質量を持った質点の受ける力は、F↑=-gradψですよね?
では、質点が加速度運動をしている場合は、上式は成立しないのではないかと思うのですが。
どうなのでしょう。
727ご冗談でしょう?名無しさん:2006/12/07(木) 02:21:45 ID:AHBicha5
>>726
要するに、質点が加速度運動をしている場合、力はポテンシャルの最大傾斜にならないのでは
ないのではないのかということです。
728ご冗談でしょう?名無しさん:2006/12/07(木) 02:38:29 ID:???
加速度運動って何だと思ってる?
729ご冗談でしょう?名無しさん:2006/12/07(木) 02:40:22 ID:???
>>723
速度振幅の単位が間違ってるぞw

51-Pegの視線速度振幅は約60km/sだ
730ご冗談でしょう?名無しさん:2006/12/07(木) 02:42:04 ID:AHBicha5
>>728
円運動、楕円運動などなど、とにかく速度ベクトルが変化する運動でいいんだよね。
特に円・楕円運動の場合が気になる・・・。
731ご冗談でしょう?名無しさん:2006/12/07(木) 02:42:46 ID:???
力がかかってる運動のことだよね。
732ご冗談でしょう?名無しさん:2006/12/07(木) 02:49:19 ID:AHBicha5
そうだね。運動方程式から言うならそれが一番適切な表現だね。
733ご冗談でしょう?名無しさん:2006/12/07(木) 03:01:09 ID:???
ma=−gradφでいいじゃん。何が疑問なんだ?
734ご冗談でしょう?名無しさん:2006/12/07(木) 03:05:58 ID:AHBicha5
a=Const.ならなにも異論はない。
aが変数でもその式は成り立つのか、その点が引っ掛かる。
735ご冗談でしょう?名無しさん:2006/12/07(木) 03:09:41 ID:AHBicha5
たとえば、
ma(t)=-gradφ
こうなるとどうにも素直に納得できない。
736685:2006/12/07(木) 03:10:28 ID:5Bb30SqF
皆さん回答ありがとうございます。
つまり様々なエネルギーの電子を含む電流であればβ線と
変わりないということなのでしょうか?

ちょっと質問はずれるかも知れませんが
電子回路のことは余り詳しくないのですが
電子回路では電流値、電圧値は幅を持つことなんてないですよね?
でもβ線のように電子線の場合だと同時にいろいろな
エネルギーの電子が存在出来るということなのでしょうか?

それと
>>687さんの
>電流の場合、電離するほど強烈なエネルギーを流したら多分消滅するんじゃね?
というのはどういうことなのでしょうか?
どういう原理で消滅するということなのですか?
737ご冗談でしょう?名無しさん:2006/12/07(木) 03:13:15 ID:???
>>734
大学生?高校生?
微分、っていうのはわかるよね?
φは「場」っていって、空間のあらゆる点について定義されていて、例えばどこか一点を
選べば、何かの値が与えられるようなものなの。
加速度も速度の微分だから、瞬間瞬間で値が変化している。

つまり、運動方程式は、ある瞬間の、ある場所において、成り立っているの。
そしてまた、それが全ての時間と、全ての空間において成り立ってるの。
だから、何も問題はない。
運動方程式に制限があったら、何の役にも立たないだろ。
少なくとも古典力学では最強の法則なんだから。
738ご冗談でしょう?名無しさん:2006/12/07(木) 03:15:31 ID:???
>>735
もっと丁寧に書けば
m(d^2/dt^2)x(t)=-gradφ(x)だよ。
739ご冗談でしょう?名無しさん:2006/12/07(木) 03:29:37 ID:AHBicha5
う〜ん、なるほど。
ただ、直感的理解がまだどうもいまいち・・・。
もう少し考えてみます。
レスどうも。
740ご冗談でしょう?名無しさん:2006/12/07(木) 03:31:32 ID:???
速度と加速度と力の定義はわかってるんだろうかw
741ご冗談でしょう?名無しさん:2006/12/07(木) 03:32:00 ID:???
>>739
もういちど聞くけど、大学生?
ここらへんは考えただけじゃ無理だよ。数学をしっかりやらないと。
微分積分、ベクトル解析の初歩だけでいいから。物理の根本に関わる部分だし。
直感的理解っていうのは、数学を理解して初めて得られるんだ。
742ご冗談でしょう?名無しさん:2006/12/07(木) 03:33:00 ID:???
>>727を解釈すると

円運動は中心力によって起こされていると理解できていないんだな
743ご冗談でしょう?名無しさん:2006/12/07(木) 03:37:41 ID:???
それなら、こう考えてみたらどうだろう。
おわんの中にビー玉を入れて回転させる。
実際は摩擦があるから中心に落ちてくけど、摩擦がなければ回り続ける。

おわんの形がポテンシャル。力はポテンシャルの最大傾斜方向に働く。
つまり、おわんの底に向かってビー玉は重力を受ける。
そして、(理論的には)回転し続ける。
744ご冗談でしょう?名無しさん:2006/12/07(木) 03:37:57 ID:AHBicha5
速度は位置ベクトルの時間微分、加速度は速度ベクトルの時間微分、力は運動量の時間微分(と結びつく)。
だと思ってるんですけど?運動方程式からすればそうですよね?
745ご冗談でしょう?名無しさん:2006/12/07(木) 03:39:16 ID:???
>>744
ということは

速度の向き≠加速度の向き≠力の向き

というのもわかるよな?
746ご冗談でしょう?名無しさん:2006/12/07(木) 03:40:18 ID:???
微妙におかしいなw
速度の向き≠加速度の向き=力の向き
747ご冗談でしょう?名無しさん:2006/12/07(木) 03:41:11 ID:???
>>745 速度の向き≠加速度の向き=力の向きでしょ?
748ご冗談でしょう?名無しさん:2006/12/07(木) 03:42:04 ID:AHBicha5
>>741
一応大学生です。

>>742
それは理解してます。

>>743
う〜ん、なるほど。
要するに、おわんのなかでビー玉がどんな運動をしようと
力はポテンシャルの最大傾斜方向に働くと。
749ご冗談でしょう?名無しさん:2006/12/07(木) 03:43:52 ID:AHBicha5
>>746
>>747
それは理解しています。
750ご冗談でしょう?名無しさん:2006/12/07(木) 03:55:57 ID:???
>おわんのなかでビー玉がどんな運動をしようと
>力はポテンシャルの最大傾斜方向に働くと。

そう。それが大事。ポテンシャルというのは、位置のみの関数なんだよ。
つまり、空間の性質って感じかな。モノがあろうがなかろうが、空間にポテンシャルは存在してるんだよ。

人の動きによって重力が変わったら大変でしょ?
重力は生物に関係なく、地球の周りに存在してる。
751ご冗談でしょう?名無しさん:2006/12/07(木) 03:58:56 ID:AHBicha5
ああ〜、なるほど!よくわかりました。すっきりしました。
イメージもはっきりつかめたし。
どうもありがとう。ひとつ勉強になりました。
752ご冗談でしょう?名無しさん:2006/12/07(木) 10:00:33 ID:UILBXEwI
相転移が一次か二次かを知ると何がわかるんでしょうか?
753ご冗談でしょう?名無しさん:2006/12/07(木) 10:16:49 ID:3JZ2/4aY
システムは、自己と非自己を識別し、非自己である外敵だけを
通常は攻撃します。しかし、実際には自己を攻撃するT細胞やB細胞も
骨髄でつくられているのです。T細胞はつくられた後、胸腺という
器官で成熟しますが、このとき自己を攻撃する細胞だけが死んで
しまいます。なんとT細胞の90%以上が胸腺で死んでしまうと
いいますから、防衛部隊に参加するのは「狭き門」なのです。
本来は消去されるべき、自己と反応する免疫細胞が残存し、
自己を攻撃するのが自己免疫疾患です。
754736:2006/12/07(木) 10:19:44 ID:5Bb30SqF
どうかよろしくお願い致します。
755ご冗談でしょう?名無しさん:2006/12/07(木) 11:04:24 ID:45O5zqH3
つまらない質問をさせていただきます。


エコキュートという給湯器は、理論上、電熱式のものに比べて3倍の
熱エネルギーを得る事ができるそうです。
一方、発電所の熱効率は50%程度のようです。

これなら、発電所のシステムにエコキュートを使えば、どんどん発電
できるのではないでしょうか。地球はどんどん寒冷化するかもしれませんが。


解釈間違ってます?
756ご冗談でしょう?名無しさん:2006/12/07(木) 11:15:24 ID:???
発電所のシステムにエコキュートを使うって、
具体的にどういう具合に?
757ご冗談でしょう?名無しさん:2006/12/07(木) 11:30:59 ID:???
>>755
エコキュートは
電気を熱に変換するんだよね

発電できない・・・

電熱式のものも電気エネルギーを100%エネルギーに変換できるわけじゃない
エコキュートも電気エネルギーを100%エネルギーに変換できるわけじゃない

電気を送電線で流すだけでエネルギーは少しは抵抗があるから
発熱などしてロスするんだけど、それはおk?

エネルギー効率で検索してね☆
758ご冗談でしょう?名無しさん:2006/12/07(木) 11:46:00 ID:45O5zqH3
>>757
電熱式のものって電気エネルギーを100%熱に変換できるんじゃないの?


エコキュートは…
電気を使って、エアコンと反対の事をする。
その熱でお湯を沸かす。

普通に電熱線に電気通すだけなら使った分の熱を得られる。
エコキュートは使った分の電気の3倍の熱量を得られる。
一方で、冷たい空気も排出するから、エネルギー保存則的には問題ない。

で、エコキュートで得た熱で発電所のお湯を沸かしてタービン回す。
って考えてたんだけど。
759ご冗談でしょう?名無しさん:2006/12/07(木) 12:00:02 ID:???
>>758
電気を送電線で流すだけでエネルギーは少しは抵抗があるから
発熱などしてロスするんだけど、それはおk?
760ご冗談でしょう?名無しさん:2006/12/07(木) 12:03:11 ID:???
>>759

それは従来の発電所でも、エコキュート式発電所でも一緒ではないでしょうか?
761ご冗談でしょう?名無しさん:2006/12/07(木) 12:52:39 ID:???
>>760
もしかして、世紀の大発見じゃない?
実験してみて。

>一方で、冷たい空気も排出するから、エネルギー保存則的には問題ない。
それをつかえば、発電しながらクーラーになるね☆




ちなみに、高校は出たの?
762ご冗談でしょう?名無しさん:2006/12/07(木) 16:00:54 ID:???
ヒートポンプの理想効率*熱機関の理想効率=1
763ご冗談でしょう?名無しさん:2006/12/07(木) 16:47:25 ID:???
トータルの熱量は使った電気分のような。

何かがおかしいのはわかってんだけど、ここは違わなくない気がするんですが。
764736:2006/12/07(木) 17:26:41 ID:5Bb30SqF
どうかよろしくお願い致します。
765723:2006/12/07(木) 20:09:38 ID:KvnEevW6
>>729
問題には確かに60m/sとかかれています

http://chunjiao.astro.ncu.edu.tw/~daisuke/ja/Research/Astronomy/Practice/Binary/c20020813/
ここの例:51 Pegasiというところをみても、60m/sのようです
766ご冗談でしょう?名無しさん:2006/12/07(木) 20:19:04 ID:???
>>752
モデルを作る時の一つの指針になる。
2次転移の時にはオーダーパラメータ(例えば磁化)が相転移点で連続で、
かつ一方の相(常磁性相)では磁化=0である。
すると、強磁性相でも相転移点に十分近ければ、磁化は十分小さいと考えられる。
従って、自由エネルギーを磁化の変数のテーラー展開することが出来る(ランダウ展開)。

もう一つの利点は、相転移点付近での系の振る舞いをおおよそ知ることが出来ること。
1次転移では、高温相を転移温度以下に急冷しても、しばらくは高温相が残る。
これは丁度水の過冷却と一緒。
2次転移では、過冷却のようなことがなく転移点以下になるなり即低温相が現れる。
767ご冗談でしょう?名無しさん:2006/12/07(木) 20:44:54 ID:???
768ご冗談でしょう?名無しさん:2006/12/07(木) 21:22:45 ID:tgdGI7ki
>>736
>> つまり様々なエネルギーの電子を含む電流であればβ線と
> 変わりないということなのでしょうか?
ベータ線は起源が違うってあれほどry
 
> 電子回路では電流値、電圧値は幅を持つことなんてないですよね?
電流も電圧も平均値だから幅はあるだろ

> でもβ線のように電子線の場合だと同時にいろいろな
> エネルギーの電子が存在出来るということなのでしょうか?
n→e^- +p+ν νがバラバラだからeもバラバラだ

> どういう原理で消滅するということなのですか?
もう専門書を買ったほうが早いよ 
769ご冗談でしょう?名無しさん:2006/12/07(木) 21:43:03 ID:Fn5m0eMA
どこでもドアを地上と海底に繋いでドアを開けても
エネルギー保存則によってドアの表面はガラス板に遮られているかのように海水は停止し
海水が地上に流れ出てくることはない。
と本に書いてあったのですが
ではドアを床と天井に設置して開いた床のドアに水を落としても
ガラス板に遮られているかのように水は流れ落ちないのでしょうか?
770ご冗談でしょう?名無しさん:2006/12/07(木) 21:46:23 ID:tgdGI7ki
>>769
流れ落ちる
771ご冗談でしょう?名無しさん:2006/12/07(木) 21:49:02 ID:5Bb30SqF
>>768
回答ありがとうございます。
専門書って何を買えば良いのでしょうか?
いくつか挙げてもらえるとありがたいです。
772ご冗談でしょう?名無しさん:2006/12/07(木) 21:49:31 ID:???
>>769

そもそもどこでもドアが現在知られている物理法則を満たすのかどうかわからんしなあ。
その辺をある程度前提として決めておいてもらわんと議論にならん。
773ご冗談でしょう?名無しさん:2006/12/07(木) 22:04:33 ID:tgdGI7ki
>>771
初級放射線 通商産業研究者
774ご冗談でしょう?名無しさん:2006/12/07(木) 22:08:10 ID:???
ふと思ったのですが、例えば東京-大阪間に
電線なり光ファイバーなりを使って、外界より全く影響を受けない
井戸型ポテンシャルを形成したとします。
そして東京でこの井戸型ポテンシャルに対してエネルギーを与え
第1励起状態にしたとします。
すると大阪でエネルギー準位が上がったことを計測するのは
いつなのでしょうか?
量子テレポーテーションのように一瞬で伝わるのでしょうか?
それともやはり光速で伝わるのでしょうか?
恐らく後者が正しいと思うのですがこのとき波動関数はどのような形で
伝搬していくのでしょうか?
775ご冗談でしょう?名無しさん:2006/12/07(木) 22:15:21 ID:???
>>770
永久機関が出来てしまうぞ!?
776ご冗談でしょう?名無しさん:2006/12/07(木) 22:16:44 ID:tgdGI7ki
>>775
だから、どこでもドアは完成不可能
背理法
777ご冗談でしょう?名無しさん:2006/12/07(木) 22:21:29 ID:???
>>776
じゃあワームホールもそうだね
778ご冗談でしょう?名無しさん:2006/12/07(木) 22:26:39 ID:???
>>765
小さすぎるということはない。
参考:
ttp://www.astroarts.co.jp/news/2004/09/02exoplanets/index-j.shtml
ttp://www.astroarts.co.jp/news/1998/01/980123NAO1/index-j.html

計算結果を現実に照らして妥当性を考えるのは大切だが、
科学で「大きい」「小さい」という感覚は、
客観的な基準があるときにのみ持つべきもの。
779ご冗談でしょう?名無しさん:2006/12/07(木) 22:42:55 ID:???
>>769
エネルギー保存則は局所局所で成り立つものなのでふつうに海水は流れる。
ただし、どこでもドアの時空歪曲を形成している場が
反作用を受けて大域的保存則は適当に満たされるだろう

仕組みによらず、関連する部分すべてを込みで見ればね
780ご冗談でしょう?名無しさん:2006/12/07(木) 23:50:57 ID:cAyE+Rki
固体と液体で吸収スペクトルを測定したところ、固体の方が液体よりも短波長側にシフトした。
結合エネルギーとか関係あるんですかね?
781ご冗談でしょう?名無しさん:2006/12/07(木) 23:55:43 ID:tgdGI7ki
>>780
物性の人間ではないんですが
面白そうなので
もう少し詳しくその実験教えてください
782ご冗談でしょう?名無しさん:2006/12/08(金) 00:07:07 ID:6AmyH3eP
>>781
説明が下手ですみません。
有機溶媒に目的物質を溶かしています。
まずは液体の状態で測定し、次に液体窒素で冷やしながら(固体になる)測定しました。
ピークがシャープになったり、増えたりするのは予想してたんですが、
固体の方がブルーシフトしました。冷やすだけで、ピークってシフトするんですかね
783ご冗談でしょう?名無しさん:2006/12/08(金) 00:09:55 ID:0CdmD+JP
>>782
何をあてたんですか?
784ご冗談でしょう?名無しさん:2006/12/08(金) 00:11:56 ID:0CdmD+JP
>>782
化合物や相の結合状態でケミカルシフトは
よくありますね
785ご冗談でしょう?名無しさん:2006/12/08(金) 00:15:20 ID:6AmyH3eP
>>783
普通に吸収スペクトルをとってます。

>>784
溶媒が固体になったことが要因かな〜と考えているのですが
いまいちよくわからないです。分子運動も関係しているのだろうか
786ご冗談でしょう?名無しさん:2006/12/08(金) 00:25:14 ID:0CdmD+JP
なんかイメージ的には固体に光当てたほうが
液体より散乱されにくい感じがします。

分子運動だとわたしも思いました
説明どうもありがとうございます。
回答にならなくて申し訳ないです。
787ご冗談でしょう?名無しさん:2006/12/08(金) 00:38:28 ID:???
>>782

温度によって電荷分布が変わって吸収が変わるやつもあるし、高温ではクエンチ
されていたパスからの発光が低温では見えてシフトすることもある。
さらには固体になると分子のあちこちにひずみが出来て(分子間のパッキングとの
兼ね合いによる)、それにより吸収波長が変わることも良くある。
分子間相互作用で軌道が変わって吸収が変わるなんてことも良くある。

だから、液体と固体ではかなり吸収が変わる(短波長シフトも、長波長シフトも
両方含む)なんてことはざら。
(シフトに見えて異なるピークの成長だったりもある)
溶液だって、濃度を変えていくとdimer、trimerなどを作って分子間相互作用等
により吸収が変わるとかあるんで、濃度ごとにかなり違う吸収になったりもする。
788782:2006/12/08(金) 00:49:28 ID:6AmyH3eP
>>786
いえいえ。興味をもっていただけて嬉しかったです。

>>787
よくあることだったとは知りませんでした。ありがとうございます!
今回の場合は
「溶媒が動くか、動かないか」がポイント、というヒントをもらいました。
分子運動が落ち着いた分のエネルギーだけシフトしたということでしょうか?
789ご冗談でしょう?名無しさん:2006/12/08(金) 00:49:56 ID:???
>>774
励起のハミルトニアン書けば
波動関数の伝搬はグリーン関数から計算できる

伝達速度の定義にもよるが
おそらく期待している答えは一瞬でも光速でもなく
励起するハミルトニアンのスペクトルに対応した群速度
790ご冗談でしょう?名無しさん:2006/12/08(金) 01:46:46 ID:NfK6Pk/J
ケプラーの第三法則って惑星にしか使えない?
791ご冗談でしょう?名無しさん:2006/12/08(金) 01:57:26 ID:???
>>790
同じふるまいをするモデルは考えられるけど
とりあえずは惑星の事を言ってるんだろ。
792ご冗談でしょう?名無しさん:2006/12/08(金) 02:03:23 ID:JC4bd64H
トレーラーと軽自動車が正面衝突して、トレーラーヘッドの運転手は無傷、
軽自動車運転手は死亡っていう事故があったのね。
重い方に乗ってるから安全ってよく言われるんだけど、
漏れの説が
 単純に重いからじゃなくて、重量にあわせて運転席のボディが堅牢に
 作られるから怪我が小さくて済む
なんだけど
 ボディが硬いか(丈夫か)どうかは関係なくて、物理的に言えば
 重い方に乗っている方が衝突のとき速度が変化しないから安全だと

じゃあ試しに軽自動車にトレーラーの荷台を引っ張らせて走り、
同じ軽自動車に正面衝突させたら、どうなる?両方死ぬだろうって
反論したんだけど、物理学者の意見を聞きたいの。

 
793ご冗談でしょう?名無しさん:2006/12/08(金) 02:14:31 ID:???
>>723
速度振幅がそのオーダーでも、現在の技術ならスペクトル解析で検出可能
794ご冗談でしょう?名無しさん:2006/12/08(金) 02:56:57 ID:???
>>792
正面衝突で死んでるときは大体座席部分がつぶれてるからあってるんじゃね?

あとはあれだ、トラックの運転席が堅牢なわけじゃなくて
運転席下部に色々積んでるから堅い+運転席が高いから直接衝突しない
ってのが大きいかと
795ご冗談でしょう?名無しさん:2006/12/08(金) 04:10:12 ID:M7+2q60c
いま、分光計を用いた光の波長の測定法を調べているんですが、
このサイトをみればわかるよ、みたいなサイトを知っている方がいれば、
教えてください。検索してもいまいち分かりませんでした。
796ご冗談でしょう?名無しさん:2006/12/08(金) 04:27:55 ID:HinN6B/X
東京の夜空はどうして赤いのですか?
797ご冗談でしょう?名無しさん:2006/12/08(金) 06:48:53 ID:4jjMdB52
「猫を電子レンジで乾かしていたら爆発した」という有名な作り話がありますが、
これは何かの定理(〜の理論?)を応用した話だと聞きました。
Wikipediaでもその定理の名前を確認したのですが失念してしまいました。
定理(理論)の名前を知っている方がおられたら教えてください。
798ご冗談でしょう?名無しさん:2006/12/08(金) 07:46:27 ID:???
>>796
インパス
799ご冗談でしょう?名無しさん:2006/12/08(金) 09:53:43 ID:tF8h2W/V
光よりも速いものって存在するんですか?
800ご冗談でしょう?名無しさん:2006/12/08(金) 10:17:08 ID:O6E0Z43u
http://plaza.rakuten.co.jp/miyawaki/diary/200405040000/で触れられている
タイムマシンの研究をしてる博士の名前はわかりませんか?
以前、ディスカバリーチャンネルで紹介されていました。
ただし、マレット博士ではありません。
801ご冗談でしょう?名無しさん:2006/12/08(金) 10:20:23 ID:???
>>797
シュレーディンガーの猫
802ご冗談でしょう?名無しさん:2006/12/08(金) 11:22:27 ID:???
>>773
回答ありがとうございます。
それって資格の本ですよね?
他にも良い本があれば教えてもらいたいのですが。
803ご冗談でしょう?名無しさん:2006/12/08(金) 13:33:04 ID:???
超ひも理論に関する書籍は多く発行されていますが
論文に関しては一度も見たことがありません。
見てみたいのですがどうすれば良いのでしょうか?
ScienceやNatureなどに登録されているのでしょうか?

804ご冗談でしょう?名無しさん:2006/12/08(金) 13:34:08 ID:???
モノクロメーターについてお聞きしたいのですが
これは回折格子とどう違うのでしょうか?
原理は全く同じだと思うのですがなぜ名前が違うのでしょうか?
805ご冗談でしょう?名無しさん:2006/12/08(金) 13:36:21 ID:???
>>804
回折格子はモノクロメーターの構成部品の一つ
806ご冗談でしょう?名無しさん:2006/12/08(金) 13:37:54 ID:SKXI7cFJ
ttp://www.igoods.jp/s/salon-more/1007/

このタキオン加工ってどういうものなのでしょうか?
光速以上の速度で動く物質を加工するって一体どういうことなのでしょうか?
しかもマッサージ効果まであるそうなのですがどういう原理で
こういう作用があるのでしょうか?
807ご冗談でしょう?名無しさん:2006/12/08(金) 13:38:55 ID:???
>>805
そうなのですか?
もしかして回折格子とフィルターを合わせて
モノクロメーターというのでしょうか?
808ご冗談でしょう?名無しさん:2006/12/08(金) 13:39:30 ID:???
>>803
arXiv.orgのレビュー講義録を紹介しあうスレッド
http://science4.2ch.net/test/read.cgi/sci/1162306343/

この辺参考に
809ご冗談でしょう?名無しさん:2006/12/08(金) 13:41:33 ID:???
>>807
一般的なモノクロメーターは
スリット、レンズ、回折格子/プリズム等分散性素子、レンズ、スリット
で出来ている
810ご冗談でしょう?名無しさん:2006/12/08(金) 13:44:25 ID:???
>>806
宇宙空間自体に存在する虚的エネルギーとして研究・実用化されました。
タキオンエネルギーとは、生命を活性化させ、生態系の調和を取り戻していく働きがあります
811ご冗談でしょう?名無しさん:2006/12/08(金) 15:07:45 ID:???
一次元調和振動子のエネルギーは
E=p^2/(2*m)+1/2*k*x^2@
で与えられる。不確定性関係Δp*Δx≧h´/2A (h´はディラックのhです。)
を用いて、一次元調和振動子の最低エネルギーを求めよ

という問で
Aを変形し
Δx≧h´/(2*Δp)を
@に代入すると

ΔE≧Δp^2/(2*m)+1/2*k*{h´/(2*Δp)}^2
となるから
最低エネルギーは E=p^2/(2*m)

と解いたのですが、全くわかりません
誰かご享受お願いします。
812ご冗談でしょう?名無しさん:2006/12/08(金) 15:20:13 ID:???
Δp^2/(2*m)+1/2*k*{h´/(2*Δp)}^2の最低値はいくらかわからないってこと?
高校数学の問題だけど。
813ご冗談でしょう?名無しさん:2006/12/08(金) 17:00:39 ID:???
微分して終わりですか?

814ご冗談でしょう?名無しさん:2006/12/08(金) 17:47:41 ID:IRtwpElb
逆格子、逆空間とかk空間と呼ばれるものについて、漠然とした質問なのですが
式を見て定義はわかりました。空間的な関係もなんとなく理解できています。
面心立方格子の逆格子を求めると、体心立方格子になるとか、
体心立方の逆格子が、面心立方になるというもの理解できた気がします。

それで、逆格子空間の単位格子はわかるのですが、逆格子空間で単位格子の
隣にある単位格子、そのまた隣にある単位格子の物理的意味って何なのでしょう。

フーリエ変換したものだと考えたら、なんというか同じ強度の格子は
繰り返さない気がするし・・・。
815ご冗談でしょう?名無しさん:2006/12/08(金) 19:02:01 ID:???
>>814
>フーリエ変換したものだと考えたら、なんというか同じ強度の格子は
>繰り返さない気がするし・・・。

これどういう意味?
816806:2006/12/08(金) 21:17:20 ID:???
>>810
タキオンを捕らえる技術ってもう確立されていたのですか?
817ご冗談でしょう?名無しさん:2006/12/08(金) 21:40:23 ID:???
釣りならよそでやってくれ
818ご冗談でしょう?名無しさん:2006/12/08(金) 21:50:37 ID:???
星占いに匹敵するくらいの効果を誇り、
相間の主張と同じくらい確立された話だよ。
819ご冗談でしょう?名無しさん:2006/12/08(金) 22:45:54 ID:???
>>800
どうかよろしくお願い致します。
私も気になって仕方がありません。
820ご冗談でしょう?名無しさん:2006/12/08(金) 23:03:24 ID:???
他人のふりして催促するのは逆効果だぞ
821ご冗談でしょう?名無しさん:2006/12/09(土) 08:42:09 ID:Z0gyx0iD
以下の説明にわからない点があるので教えて下さい。

縦軸が時間、横軸が空間のグラフを思い浮かべてください。
物体の等速直線運動は、このグラフ上では傾いた直線として表わされます。これを世界線と言います。
速度が大きくなるほど、世界線の傾きの角度は大きくなります。
分かりやすくするため、光速を45度ということに決めます。
物体が静止している場合、世界線は垂直(時計で言うなら0時と6時を結ぶライン)になり、
その同時刻線は水平(時計で言うなら3時と9時を結ぶライン)になります。
物体が光速に近づいたとします。仮に角度が垂直から30度傾き、1時と7時を結ぶラインになったとします。
すると同時刻線は2時と8時を結ぶラインになります。
822ご冗談でしょう?名無しさん:2006/12/09(土) 08:43:48 ID:Z0gyx0iD
>物体の等速直線運動は、このグラフ上では傾いた直線として表わされます。
>これを世界線と言います。
>速度が大きくなるほど、世界線の傾きの角度は大きくなります。

まずはこの世界線とは何なのか?というと
時間が経過するごとにどれだけの距離を進むのかを表わしたグラフ上の直線のことですよね?

>物体が静止している場合、世界線は垂直(時計で言うなら0時と6時を結ぶライン)

静止した物体の世界線は0m地点なら時間軸の直線に重なって時間が経過するだけだということですよね?
しかし!

>その同時刻線は水平(時計で言うなら3時と9時を結ぶライン)になります。

この同時刻線とは何なのでしょうか?
なぜ静止して物体の同時刻線は水平になるのですか?
水平の同時刻線はグラフ上のどの部分に引かれるのですか?
横軸の空間の直線に重なるのでしょうか?

>物体が光速に近づいたとします。
>仮に角度が垂直から30度傾き、1時と7時を結ぶラインになったとします。
>すると同時刻線は2時と8時を結ぶラインになります。

この場合もなぜ同時刻線は2時と8時を結ぶライン(40度)になるのですか?
ちなみに光速の世界線と同時刻線は両方とも45度なのでしょうか?
823ご冗談でしょう?名無しさん:2006/12/09(土) 08:46:46 ID:???
その説明の前に同時刻ってことがどういうことか書いてあるんじゃね?
それは読んだ?
824ご冗談でしょう?名無しさん:2006/12/09(土) 09:13:16 ID:???
>>823
掲示板に書き込まれていた説明はあれだけです。
説明はありません。
825ご冗談でしょう?名無しさん:2006/12/09(土) 09:15:59 ID:???
>>824
じゃあ特殊相対論の入門書でも読んだらいいよ
826ご冗談でしょう?名無しさん:2006/12/09(土) 11:19:01 ID:aPbusgzA
超伝導の実験をやってる2年生です。
月曜までにレポート書かなくちゃいけないんですが、
土日なんでここで質問させてください。

SQUIDっていう機械で超伝導をみて、
「Long Detrended Voltage - Position」のグラフで考察するということらしいんです。

それで、positionっていうのは試料の位置だと思うんですが、
「Long Detrended Voltage」っていうのが何なのか分からんのです。

「detrended」っていうのを辞書で調べても載ってないし
ググっても英語のぺージしか出てこなくて、
翻訳してもdetrendedだけ訳されないで出てきてしまいます。

長文になりました。よろしくお願いします。
827800:2006/12/09(土) 11:40:55 ID:???
どうかどうかよろしくお願い致します。
828ご冗談でしょう?名無しさん:2006/12/09(土) 11:45:03 ID:???
>>827
Dr. Nakamatsu
829ご冗談でしょう?名無しさん:2006/12/09(土) 12:21:48 ID:???
物理に限ったことではないのですが
論文を書いた後にどこかの学会が雑誌に発表しますよね?
これって掛け持ちしても良いものなのでしょうか?
ScienceやNatureはものすごく審査が厳しいと聞きます。
ということは出しても審査が通らなければライバルに追い越されるということは
ないのでしょうか?
また、PhysicalReviewに出すか、ScienceやNatureは
どうやって決めるのでしょうか?物理関係の論文はなぜかScienceやNatureに
あまり出されていないように思うのですが、これはなぜなのでしょうか?
830ご冗談でしょう?名無しさん:2006/12/09(土) 12:34:17 ID:???
>>829
審査にも通らない内容なんて・・・
しかも論文は日付を入れてあるから
雑誌に発表しなくても、発表してあればおkなんじゃね?
831814:2006/12/09(土) 13:08:14 ID:QbB+UJvE
>>815
>>フーリエ変換したものだと考えたら、なんというか同じ強度の格子は
>>繰り返さない気がするし・・・。

>これどういう意味?

ちょっと書き方へんだった
たとえばですよ。距離空間で5おきに原子があるのをフーリエ変換すると
2π/5に点があらわれるでしょう。2π/5の整数倍の位置には点が
あらわれなくて、2π/5の一点だと思うのですが、逆格子空間見ると
またまたたくさん周期的に並んでない?

あれ、なんでなのっていう話。
832ご冗談でしょう?名無しさん:2006/12/09(土) 13:30:28 ID:???
>これって掛け持ちしても良いものなのでしょうか?

掛け持ちの意味が良くわかりませんが,2重投稿とかなら基本的に禁止されています.
でもやる人はやってますが.
・・・競争激しい中国のかたがたは良くやってくれる(挙句,二誌からの査読がこっちに
同時に回ってきたり.当然内容は一緒)ので困ったものです.

>ということは出しても審査が通らなければライバルに追い越されるということは
>ないのでしょうか?

あります.
そのため揉めそうな時とか,競合するグループに査読が行って引き伸ばされそうな
ときとか,微妙で議論の余地があるけど事実だったら画期的・だから先にやったのは
俺だと主張するために現段階で載せておきたい,等という場合にはあえてジャーナルの
ランクを落として,そのかわり素早くアクセプトされそうなところに出すなどと
言う処置を取ることもあります.
833832:2006/12/09(土) 13:30:58 ID:???
続き.

>また、PhysicalReviewに出すか、ScienceやNatureはどうやって決める
>のでしょうか?

Nature,Scienceは速報的に,その後十分実験を重ねてまとめた長い論文を
PR*に,とか.

>物理関係の論文はなぜかScienceやNatureにあまり出されていないように
>思うのですが、これはなぜなのでしょうか?

元々これら二誌は一般向け(幅広い分野向け)を意識している雑誌なので,
ある程度専門外の分野にもアピールできるものでないと切られます.
物理だとわりと細かい話が多くなってくるので,そういう意味では載せにくく
なっています.もちろん物理でも一目でわかる画期的成果,とかなら載り
ますが.
後はページ数の制限ですか.長々と解析したり検証しないと駄目な分野などでは
大変載せにくくなります.まあ詳細を別に,って手もありますが.
834ご冗談でしょう?名無しさん:2006/12/09(土) 13:39:32 ID:???
拡散波動方程式って何なんでしょうか?
昔何か読んだ気もするのですが
普通の波動方程式との違いがわかりません。
835ご冗談でしょう?名無しさん:2006/12/09(土) 13:40:59 ID:???
>>831
>2π/5の整数倍の位置には点が あらわれなくて

点が現われるの意味が分からんけど
とりあえずフーリエ変換が2π/5の整数倍だったら
5毎の周期は満たすじゃん。
836ご冗談でしょう?名無しさん:2006/12/09(土) 13:46:01 ID:???
ロジャー・ペンローズによれば亜光速で動く物体はローレンス変換により
前後に縮むのではなく回転して見えると本に書いてあるのですが
これっておかしくないですか?
物体の向こう側から来る光は物体の背面に当たりその光も
こっちに届くために物体の背面も見える、というような
図が書かれていますが、物体に当たった光はのうち
一部はその物体に吸収されもう一部は反射して
その一部の反射された光を見ることで物体の色・形を
認識することが出来るのだと思うのですが、反射されということは
物体の向こう側に行くために観測者には光は届かないと思うのですが。
逆に物体に当たらない光はこちらに届きますが、物体に当たっていないために
物体の色・形情報を含んでいないと思うのですが。
それともあまり詳しくないので知りませんが、プラズモン波やエバネッセント光が
こちらに届くという意味なのでしょうか?
837ご冗談でしょう?名無しさん:2006/12/09(土) 13:47:40 ID:MT6JdK/Z
>>828
Dr. Nakamatsu で検索してみましたが全然関係ないように
思うのですが。
本当のことを教えてください。
838ご冗談でしょう?名無しさん:2006/12/09(土) 14:05:06 ID:???
>>836
>それともあまり詳しくないので知りませんが、プラズモン波やエバネッセント光が
 違う。もっと単純なこと。
 向こう側から出た光を、物体がかわすように動くから、光が観測者に届く、というだけ。
 光行差現象に似たものだよ。
839ご冗談でしょう?名無しさん :2006/12/09(土) 14:28:34 ID:MxV4VItQ
電磁石の下に、逆巻きで筒系の電力を流すと、理論的に重力が発生しませんか?
解答お願いします。
840ご冗談でしょう?名無しさん:2006/12/09(土) 14:32:21 ID:???
重力ってなに?
841ご冗談でしょう?名無しさん :2006/12/09(土) 14:41:23 ID:MxV4VItQ
空式エンジンと空式エンジンをぶつけたとき、どんなエネルギーが発生しますか?
842814:2006/12/09(土) 14:50:10 ID:QbB+UJvE
>>835
なんつうか

実空間 ・ ・ ・ ・ ・ ・ ・ ・ ・ ・ ・ ・ ・ ・ ・ ・ ・
↓フーリエ変換
逆空間 ・(1点のみ)
っていうイメージなのですが、参考書には

実空間・ ・ ・ ・ ・ ・ ・ ・ ・ ・ ・ ・ ・ ・ ・ ・ ・

逆空間・  ・  ・  ・  ・  ・  ・  ・  ・  ・
みたいに載ってるので、それがわからないのよ
843ご冗談でしょう?名無しさん:2006/12/09(土) 15:00:11 ID:???
>>842
そのフーリエ変換の計算過程を書いてみて。
844ご冗談でしょう?名無しさん:2006/12/09(土) 15:03:35 ID:???
>>838
やっぱりどうしても分かりません。
物体が光をかわしてそのかわされた光を
観測者が受け取ったとしても単に物体向こう側が見えるだけだと
思うのですが。どういうことなのでしょうか?
845ご冗談でしょう?名無しさん:2006/12/09(土) 15:07:24 ID:???
中性子よりも陽子の方がわずかに質量が大きいそうなのですが
なぜなのでしょうか?
アップクオークよりもダウンクオークの方が重いからなのでしょうか?
それとも結合力の問題なのでしょうか?
846ご冗談でしょう?名無しさん :2006/12/09(土) 15:12:11 ID:MxV4VItQ
バカばっか
847ご冗談でしょう?名無しさん:2006/12/09(土) 15:14:10 ID:???
>>842

格子振動ではブリルアンゾーン外は意味を持たないから無視。
電子物性では、逆格子ベクトル分の運動量は自由に格子との間でやりとり
できるので第一ブリルアンゾーン内に押し込んで還元ゾーン方式で書くことが
多い。
フェルミ面形状などを考えるときは、考えやすいように還元ゾーンを単位と
して、結晶格子が並んでいるのと同じようにいくつも並べて書くことがある。
(反復表示)
フェルミ面のネスティングだとか、ドハースファンアルフェンとか考えるとき
に便利。
X線なんかだと、n次の回折は1/nの格子間隔を持つ仮想的な結晶格子からの1次の
反射とも考えられるから、この場合はある結晶格子はその1/nの格子間隔のすべての
仮想格子を含む、として、逆格子空間に全部書き出してやると繰り返し構造になる。
848ご冗談でしょう?名無しさん :2006/12/09(土) 15:26:51 ID:MxV4VItQ
バカばっか
849ご冗談でしょう?名無しさん:2006/12/09(土) 15:28:50 ID:???
>>845
>観測者が受け取ったとしても単に物体向こう側が見えるだけだと
 そのことを「回転して見える」といっているだけなんだって。
850ご冗談でしょう?名無しさん :2006/12/09(土) 15:30:03 ID:MxV4VItQ
バカばっか
851ご冗談でしょう?名無しさん:2006/12/09(土) 15:33:07 ID:???
>>849
物体が光をかわして、そのかわされた光を
観測者が受け取ったとしても単に物体の向こう側の景色が見えるだけ

という意味です。紛らわしくすいません。
どういうことなのでしょうか?
852ご冗談でしょう?名無しさん:2006/12/09(土) 15:38:19 ID:Mr8QCWF/
>>847
それでわかった。その注釈なく、勝手に繰り返し構造書いてあるのって
ちょっと不親切だと思う。

X線の回折条件でn次回折が実空間では面間隔n分の1の面からの回折
だから、逆空間上で繰り返し構造というのも納得した。
このとき逆格子を図で書くのに同じ濃さで点を書いていいものかと少し
悩むが
853ご冗談でしょう?名無しさん:2006/12/09(土) 15:39:54 ID:Mr8QCWF/
847の前半部分はブロッホの定理とか関係するわけですな。
つか842の疑問はおかしくないよね。843が何か聞いてるが・・・
854ご冗談でしょう?名無しさん :2006/12/09(土) 15:41:28 ID:MxV4VItQ
バカばっか
855ご冗談でしょう?名無しさん:2006/12/09(土) 15:50:20 ID:???
>>851
移動する物体自体が発光しているという前提にすれば余計な事を考えなくてよい
856ご冗談でしょう?名無しさん :2006/12/09(土) 15:50:43 ID:MxV4VItQ
バカばっか
857ご冗談でしょう?名無しさん:2006/12/09(土) 15:52:40 ID:2GYXtgf4
久々に登場のミッキー柳井???じゃなくコレが初無修正となる杏野
みつちゃん。知性溢れるこのお顔はリポーターにバッチリで、色ん
なレポートを身体を張って果敢に突入!面接でお触り攻めを受けた
り、ファンの前でバイブ攻めを受けたり、体験マッサージでミッキ
ー柳井にアクロバティック生中出しを受けたり、高級料亭でイラマ
チオを受けたり、拘束されてのバイブ責め、ラストは3Pで連続ぶ
っかけを受けたりでもう大変な事になっちゃってます!みつちゃん
ファンはもちろん、ミッキーファンにも絶対に手に入れて頂きたい
一本。
http://www.rormb.com/btlive
858ご冗談でしょう?名無しさん :2006/12/09(土) 15:55:40 ID:MxV4VItQ
バカばっか
859ご冗談でしょう?名無しさん:2006/12/09(土) 16:10:42 ID:???
空式エンジンと空式エンジンをぶつけたとき、どんなエネルギーが発生しますか?
860ご冗談でしょう?名無しさん :2006/12/09(土) 16:15:21 ID:MxV4VItQ
空間エネルギー
861ご冗談でしょう?名無しさん:2006/12/09(土) 16:46:52 ID:???
バカには聞いてません
862ご冗談でしょう?名無しさん :2006/12/09(土) 16:52:49 ID:MxV4VItQ
バカばっか
863ご冗談でしょう?名無しさん:2006/12/09(土) 17:25:14 ID:???
エルゴード問題って一体どういうことなのでしょうか?
調べてみても一向によく分かりません。
どなたか分かりやすく説明していただけないでしょうか?
864ご冗談でしょう?名無しさん :2006/12/09(土) 17:30:54 ID:MxV4VItQ
バカばっか
865ご冗談でしょう?名無しさん:2006/12/09(土) 17:32:06 ID:???
>>863
「物理学とは何だろうか」 (上 ・下) 岩波新書 黄版 朝永振一郎
たしかこれとかに
866ご冗談でしょう?名無しさん :2006/12/09(土) 17:36:07 ID:MxV4VItQ
バカばっか
867ご冗談でしょう?名無しさん:2006/12/09(土) 18:11:06 ID:???
結局、熱ってなに?
868ご冗談でしょう?名無しさん:2006/12/09(土) 18:13:02 ID:???
>>867

エネルギーの分配の仕方。
いろいろな粒子のいろいろな自由度に、でたらめにエネルギーが分配されているほど
温度が高い。
869ご冗談でしょう?名無しさん:2006/12/09(土) 18:31:09 ID:???
いや、温度じゃなくて熱って何?って疑問なんですが。
870ご冗談でしょう?名無しさん:2006/12/09(土) 18:33:53 ID:???
青のベンザでおさまるもの
871ご冗談でしょう?名無しさん:2006/12/09(土) 18:40:22 ID:???
2つ以上の系で内部エネルギーをやり取りするとき
仕事に含めることが出来ない部分
872ご冗談でしょう?名無しさん :2006/12/09(土) 18:42:26 ID:MxV4VItQ
バカばっか
873ご冗談でしょう?名無しさん:2006/12/09(土) 19:48:12 ID:???
微視的な自由度にランダムに配分されてしまったエネルギー
874ご冗談でしょう?名無しさん :2006/12/09(土) 19:52:23 ID:MxV4VItQ
バカばっか
875ご冗談でしょう?名無しさん:2006/12/09(土) 19:52:48 ID:???
>>860
はスクリプトのバグかなんかですか?
876ご冗談でしょう?名無しさん:2006/12/09(土) 19:55:56 ID:???
空式エンジンと空式エンジンをぶつけたとき、どんなエネルギーが発生しますか?
877ご冗談でしょう?名無しさん :2006/12/09(土) 19:56:57 ID:MxV4VItQ
バカばっか
878ご冗談でしょう?名無しさん:2006/12/09(土) 19:58:59 ID:???
パカぱっか
879ご冗談でしょう?名無しさん:2006/12/09(土) 20:00:07 ID:???
バグとれたみたい
880ご冗談でしょう?名無しさん:2006/12/09(土) 20:04:41 ID:???
空式エンジンって何ですか?
検索したけどでてきません。
881ご冗談でしょう?名無しさん :2006/12/09(土) 20:30:42 ID:MxV4VItQ
バカばっか
882ご冗談でしょう?名無しさん:2006/12/09(土) 20:39:14 ID:???
>>851
 物体の裏側からでた光も、物体がするりとかわして見えるようになるんだよ。
 物体が光に近い速度で動いているということを忘れちゃダメだな。

883ご冗談でしょう?名無しさん:2006/12/09(土) 21:17:23 ID:???
しょーもない質問ですが誰か教えて下さい

鏡に写る物体は左右逆になるのに
どうして上下逆にならないんですか?
884ご冗談でしょう?名無しさん:2006/12/09(土) 21:21:08 ID:???
よーく考えてごらん
左右は逆になっていない事がわかるから
885ご冗談でしょう?名無しさん:2006/12/09(土) 21:22:17 ID:???
あしゅら男爵みたいな化粧して鏡見てみ
上下逆になってるから
886ご冗談でしょう?名無しさん:2006/12/09(土) 21:29:24 ID:75KJKA/6
質問です。
質量m1、m2がお互い引力をおよぼしながら、
角速度ωで半径rで円運動をしている。

重心まわりの全運動量を求める問題です。

答えはm1*m2*ω*r~2/(m1+m2)なんですが、

解き方がわかりません。


お願いします。
887883:2006/12/09(土) 21:31:08 ID:???
>>884

うーん、でも文字や数字を鏡に写すと
左右逆になった(みたいな)字になるじゃないですか
やっぱ左右逆になってじゃないの?
888ご冗談でしょう?名無しさん:2006/12/09(土) 21:33:48 ID:???
>>887
もっとよーく考えてごらん
立体的なもので考えたほうがわかりやすいかも
889883:2006/12/09(土) 21:41:50 ID:???
>>888

よーわからん…
鏡に字を写すと左右逆に見えるし
上下は逆になってないように見えるよ
立体的にどう考えれば
左右逆じゃないと分かるのやら
890ご冗談でしょう?名無しさん:2006/12/09(土) 21:42:01 ID:???
>>882
やっぱり納得がいきません。
エバネッセント光を観測するということではないのですよね?
仮に物体に当たった光が進路を変えずに観測者の方向に
向かってくるのだとしても、光速ならまだしも、亜光速であれば
ちょっと物体が動いたあとにそのずれた点にまたぶつかって
観測者には届くとは思えないのですが。
891800:2006/12/09(土) 21:43:52 ID:???
も、もうこのことが気になってしまって他のことに
何も手が付きません。
どなたか教えてくださいよ。
どうかどうかよろしくお願い致します。
892ご冗談でしょう?名無しさん:2006/12/09(土) 21:49:49 ID:???
>>891
Dr. Hamamatsu
893ご冗談でしょう?名無しさん:2006/12/09(土) 22:10:39 ID:???
>>889
ちゃんと左右は逆になってる。心配するな。
こんなのは方向の定義の問題だ。
894ご冗談でしょう?名無しさん:2006/12/09(土) 22:35:48 ID:???
なってねーよボケ
895ご冗談でしょう?名無しさん:2006/12/09(土) 22:36:59 ID:???
定義だ、で逃げるのは理解してない証拠
896ご冗談でしょう?名無しさん:2006/12/09(土) 22:38:36 ID:???
>>889
じゃあさ
鏡を見ながらひげをそるとき、どの方向に動かすのが一番違和感ある?
897ご冗談でしょう?名無しさん:2006/12/09(土) 22:39:16 ID:???
右手ってお箸持つ手ですよね。
お箸もってますよ
898ご冗談でしょう?名無しさん:2006/12/09(土) 22:39:48 ID:???
>>896
前後!
899ご冗談でしょう?名無しさん:2006/12/09(土) 22:50:11 ID:???
バカばっかの人がおとなしいところをみると
sageながら答えてるんだろうなwww
900883:2006/12/09(土) 22:59:58 ID:???
>>896
>>898

自分も前後かな?

でもその違和感と
鏡に写る物体の上下左右の差異とどうつながりが…
バカな自分にも分かるように教えて欲しいです
901ご冗談でしょう?名無しさん:2006/12/09(土) 23:01:47 ID:???
>>900
じゃあ、これが俺からの最後のヒント
君は、なんで左右が反転したと判断したの?
902883:2006/12/09(土) 23:22:03 ID:???
>>887みたいな現象が起きるからだよ

これが最後だなんてツレないなぁ〜(´・ω・`)
903ご冗談でしょう?名無しさん:2006/12/09(土) 23:26:08 ID:???
>>890
 ちゃんと図を描いて考えろ。「思えない」では話にならん。
904ご冗談でしょう?名無しさん:2006/12/09(土) 23:40:10 ID:???
>>894-895
左右の定義も考えずに反転しないとか言ってたのか。アホだな。
頼むから前後が反転するとか言い出さないでくれよ。
905ご冗談でしょう?名無しさん:2006/12/09(土) 23:43:23 ID:???
>>904
あのさ
左右は反転してもしなくても良いよ
定義次第だ

だがな、それは物理の話題じゃねぇ
906ご冗談でしょう?名無しさん:2006/12/09(土) 23:44:57 ID:???
>>902
あー
>>887みたいな現象が起こるとき、何で反転したのが"左右"だと思うわけ?
そこが最大のポイントなんだが
907904:2006/12/09(土) 23:46:09 ID:???
>>905
俺はあんたと同意見だよ。
こんなのは方向の定義の問題で物理的な議論ではない。

そして大勢の人間は「鏡で反転する」左右の定義を使ってるってことだ。
908ご冗談でしょう?名無しさん:2006/12/09(土) 23:54:19 ID:???
少なくとも幾何光学で考えるかぎり、平面鏡で反転しているのは鏡に垂直な軸なんだがな
909883:2006/12/10(日) 00:05:43 ID:???
うーん、
例えたら、たとえば
「あ」という文字が書いてあるカードを自分が持っている
そしてその隣に別の人Aが同じ「あ」と書いてあるカードをもっていて
お互いが同じ方向をむいてるとして
こんなカンジ
自分 ▼ ▼A
カード□ □カード

そいでAが自分の目の前にやってきてカードを見せたとき

こんなカンジ

 ▼
 □
 □
 ▲

左右反転することなく「あ」と読めるのに
同じシチュエーション(であるかの)ような
鏡に写した状態だと「あ」という文字が
左右逆(になったみたい)に見えるから…かな?
910ご冗談でしょう?名無しさん:2006/12/10(日) 00:09:05 ID:???
>>909
その「あ」という文字が透明なシートに書かれていたら?
911ご冗談でしょう?名無しさん:2006/12/10(日) 00:13:54 ID:???
>>908
それと左右の反転は全く別の話だよ。
「鏡に垂直な軸が反転してるから左右は反転しないはずだ」
という議論は完全に的を外している。
912ご冗談でしょう?名無しさん:2006/12/10(日) 00:17:31 ID:???
物理的には「鏡に垂直な軸だけが反転している」で終わっている。
それを左右が反転だと認識するのは心理学や社会学の領域。
913ご冗談でしょう?名無しさん:2006/12/10(日) 00:20:01 ID:???
なぜ鏡は上下対称でなく左右対称に写し出されるの?
ttp://makimo.to/2ch/science4_sci/1069/1069080866.html

このスレなくなったと思ったら1000行って大往生してたんだな
914ご冗談でしょう?名無しさん:2006/12/10(日) 00:25:00 ID:QyO50hPu
なぜ鏡は上下対称でなく左右対称に写し出されるの?

視線が鏡で反射しているとすれば
至極納得するがこれではだめなのか?
915ご冗談でしょう?名無しさん:2006/12/10(日) 00:27:05 ID:???
>>914
いいよ
916ご冗談でしょう?名無しさん:2006/12/10(日) 00:28:41 ID:???
反転、と言う比較に
異なる軸使って「左右は反転してない」って馬鹿?
917883:2006/12/10(日) 00:32:00 ID:???
>>910

透明でも「自分からみれば」
カードの「あ」は普通に読めるんじゃないの?
918ご冗談でしょう?名無しさん:2006/12/10(日) 00:35:15 ID:???
鏡の問題はスレ立てればいいよ
919ご冗談でしょう?名無しさん:2006/12/10(日) 00:57:15 ID:???
ようするに、紙(鏡)に判子を押すのと一緒じゃないのか
この場合も左右だけ反転する
920ご冗談でしょう?名無しさん:2006/12/10(日) 01:02:19 ID:???
たてた

なぜ鏡は左右だけ逆になるの?専用
http://science4.2ch.net/test/read.cgi/sci/1165680109/
921ご冗談でしょう?名無しさん:2006/12/10(日) 01:04:50 ID:???
>>919
それだって、現象として反転してるのは面に垂直な軸なんだよな
それをなんで前後でもなく上下でもなく左右が反転したと捕らえるのかが問題なわけ

もうちょっというと、誰かに"鏡文字を書いて"といわれると、ふつうは左右反転の文字を書くけど
上下反転で文字を書いても"鏡文字"には違いないよねという話
922ご冗談でしょう?名無しさん:2006/12/10(日) 01:11:07 ID:???
はんこを押す

押したはんこを手前に倒す方向に90度回転

持ち上げてさらに同じ方向に90度回転

見比べる

どう見ても上下対称です
923ご冗談でしょう?名無しさん:2006/12/10(日) 01:50:48 ID:???
鏡はスレが立ったので質問に割り込ませてください

物質は殆ど真空で占められているのに物性を持っています。しかしイオン化
してしまえば失われてしまいます。どうやら電子と原子核の間にあるだだっ
広い空間がこれを産み出しているようなのですが、高校物理までしか習って
いないので分かりません。この空間には何が詰まっているのですか
924ご冗談でしょう?名無しさん:2006/12/10(日) 01:52:50 ID:???
ただの真空。
925ご冗談でしょう?名無しさん:2006/12/10(日) 02:02:24 ID:???
真空には電子が詰まっているのだよ
926ご冗談でしょう?名無しさん:2006/12/10(日) 02:15:33 ID:???
そっちはよく知らんが、高エネルギーの原子核と電子が分離して出来たプラズマ
弾性体の内部は安定したチャンネルの電磁波が満ちているから、非常に固いと聞
いた事はある。大規模のな原子だと考えてくれ。ならば当然ミクロの世界でも起
こっているのだろう

蛇足:ミステリーサークルもコイツの仕業らしい。ライトセーバーも
927ご冗談でしょう?名無しさん:2006/12/10(日) 05:43:36 ID:3XWMGXac
気球は袋の中の空気を温め軽くして浮かせますよね
もし袋を密閉状態にしてこれを温めたら、浮くんでしょうか?
928ご冗談でしょう?名無しさん:2006/12/10(日) 06:16:22 ID:???
>>927
袋が膨らめば浮く
929ご冗談でしょう?名無しさん:2006/12/10(日) 06:17:40 ID:???
>>926
>蛇足:ミステリーサークルもコイツの仕業らしい
ほんとに蛇足だな。今どきこんな説唱えるのは大ウソ槻キョージュくらいかと思ってたが
930ご冗談でしょう?名無しさん:2006/12/10(日) 06:20:17 ID:3XWMGXac
>>928
つまり既にパンパンの密閉された袋を温めても浮かないってことですね
ありがとうございます
931ご冗談でしょう?名無しさん:2006/12/10(日) 08:37:26 ID:???
ダイヤモンドの盾とダイヤモンドの矛で戦ったらどっちが勝つんですか?
932ご冗談でしょう?名無しさん:2006/12/10(日) 08:39:36 ID:???
金持ちの方
933ご冗談でしょう?名無しさん:2006/12/10(日) 13:18:03 ID:???
ダイヤモンドは堪性がないんだから、盾にも矛にも使えん
まあ、矛のほうがとがってる分不利そうだが
934ご冗談でしょう?名無しさん:2006/12/10(日) 13:33:08 ID:???
おもちゃのこまって一秒間に何回転すると思いますか。20回くらいですかね?
935ご冗談でしょう?名無しさん:2006/12/10(日) 13:39:36 ID:???
>>934
マークつけてインバーターでない蛍光灯の下で回すと測れるんじゃない?
http://www.math.kobe-u.ac.jp/~kodama/tips-flicker.html
936ご冗談でしょう?名無しさん:2006/12/10(日) 13:50:25 ID:???
>>934
すいません、だいたいの値がわかればいいのでみなさんの直感でどれくらいかを知りたいのです
937ご冗談でしょう?名無しさん:2006/12/10(日) 14:11:43 ID:???
>>936
桁はそれくらいに1票
938ご冗談でしょう?名無しさん:2006/12/10(日) 14:23:38 ID:???
マイケルソン干渉計において片方の反射面がθだけ傾いているとき、
観測される干渉パターンはどのような形になるか?

という問題なのですが、やっぱり光路差を求めてその分だけ光路差が増えるので〜と
したほうがいいんでしょうか?
939ご冗談でしょう?名無しさん:2006/12/10(日) 14:41:53 ID:RDC5Zq44
電池の起電力を電位差計で測定する時と、電圧計で測定するときとではどんな違いがありますか?
940ご冗談でしょう?名無しさん:2006/12/10(日) 15:46:37 ID:???
マイケルソンモーレーの実験ってどこで
行われたのでしたっけ?
赤道上でしたっけ?
941ご冗談でしょう?名無しさん:2006/12/10(日) 15:54:13 ID:???
物理を専攻しているわけではないのですが
テンソルとかスピノールという言葉について素人ながらも
知識をつけておきたいのですが
何かお勧めの参考書はないでしょうか?
942ご冗談でしょう?名無しさん:2006/12/10(日) 16:45:26 ID:???
二重の極みの原理は本当にありえますか???
943ご冗談でしょう?名無しさん:2006/12/10(日) 16:53:43 ID:???
>>940 アメリカのどこかじゃなかったかな?赤道上である必然性はぜんぜんないよ。
944ご冗談でしょう?名無しさん:2006/12/10(日) 16:55:44 ID:???
>>942
るろうに剣心の二重の極みって物理的にどうよ?
http://science4.2ch.net/test/read.cgi/sci/1048523785/
945ご冗談でしょう?名無しさん:2006/12/10(日) 16:56:05 ID:???
>>941
Sakuraiの当該の章とか
946ご冗談でしょう?名無しさん:2006/12/10(日) 16:56:44 ID:???
>>938
したほうがいいね
947ご冗談でしょう?名無しさん:2006/12/10(日) 17:05:24 ID:???
>>943
自転の最も速い場所の方が測定しやすいのではないのですか?
948ご冗談でしょう?名無しさん:2006/12/10(日) 17:05:58 ID:???
>>945
何ですかそれは
もっと詳しくお願い致します。
949ご冗談でしょう?名無しさん:2006/12/10(日) 17:14:41 ID:???
>>948
J J Sakurai 現代の量子力学 下
になんかあったとおもう
950ご冗談でしょう?名無しさん:2006/12/10(日) 19:24:01 ID:???
>>926
「硬い」って言ってもどうせ音速がでかいとかそういう意味だろ?
951ご冗談でしょう?名無しさん:2006/12/10(日) 20:48:22 ID:gOkhMf60
二次元や4次元の世界は宇宙に存在できるんでしょうか?
例えばアニメみたいな
952ご冗談でしょう?名無しさん:2006/12/10(日) 21:32:33 ID:???
光について質問です。
ヤングの干渉実験で光は波動性があることが確認されましたが、光は粒子性もあるといいます。
そこで粒子の性質を用いても光の干渉が起きるということなのですがその理由がわかりません。
なぜ粒子である光子が干渉を起こすのでしょうか?
953ご冗談でしょう?名無しさん:2006/12/10(日) 21:43:19 ID:???
またβかよ
954ご冗談でしょう?名無しさん:2006/12/10(日) 21:44:57 ID:???
マジレスだが理由はない

これについては何故と考えるよりそういうものだと思って
これまでの実験事実とその解釈を学んだほうがはるかに有益
955ご冗談でしょう?名無しさん:2006/12/10(日) 21:46:31 ID:???
>>947
自転(赤道で460m/s)よりも公転(30km/s)が効くから、あまり関係ない
956952:2006/12/10(日) 21:54:13 ID:???
>>954
そうですか。。。ありがとうございました。
957ご冗談でしょう?名無しさん:2006/12/10(日) 22:12:21 ID:jaQolO9l
私は>>677です。(参考: >>682-684,>>688,>>690)

事象:
銀行へ元金を預ける。
残高は複利で増える。
数式で表せば下式となる。
預金年数満了時残高 = 元金*(金利)^(預金年数)

変数名[単位]:
預金年数満了時残高[yen]
元金[yen]
金利[?]
預金年数[t]

質問:
金利[?]の単位は、どのように書くべきですか?
958ご冗談でしょう?名無しさん:2006/12/10(日) 22:16:03 ID:QyO50hPu
>>957
 %
959ご冗談でしょう?名無しさん:2006/12/10(日) 22:36:03 ID:???
>>958
100倍してないじゃん。

>>957
…利率には元金の分1を足しとかないと残高がどんどん減ってくよ。
利率は単位無し。それに年数の単位はtじゃなくて年だよ。
960ご冗談でしょう?名無しさん:2006/12/10(日) 22:48:19 ID:???
>>952
光は粒子でも波動でもなく、その両方を合わせ持つものってことだよ。
今までの常識に照らし合わせるんじゃなく、
そういう新しい概念だと受け入れる必要がある。
961957:2006/12/10(日) 22:56:40 ID:5pIRLsYz
預金年数満了時残高 = 元金*(1+金利)^(預金年数)

変数名[単位]:
預金年数満了時残高[yen]
元金[yen]
金利[?]
預金年数[nen]

金利の単位を、金利[1^(1/nen)]と書くのは、間違いですか?
962ご冗談でしょう?名無しさん:2006/12/10(日) 22:57:23 ID:???
間違い
963ご冗談でしょう?名無しさん:2006/12/10(日) 23:02:54 ID:???
どっちかっつーと預金年数のとこを無次元にするように
964957:2006/12/10(日) 23:13:59 ID:5pIRLsYz
>>963 そのように、します。しかし、納得できません。
965ご冗談でしょう?名無しさん:2006/12/10(日) 23:23:33 ID:???
アンタが納得するしないと正しいかどうかは関係ないだろうが。
966952:2006/12/11(月) 01:08:26 ID:???
>>960
粒子性の面から干渉が起きる理由というのを
知りたいということなんです。。
967ご冗談でしょう?名無しさん:2006/12/11(月) 01:18:36 ID:???
あくまで

波動性の性質が干渉を引き起こす。
粒子性は関係ない。
968ご冗談でしょう?名無しさん:2006/12/11(月) 01:21:05 ID:???
>>966
だから粒子性を持ってるってことじゃない。
波動粒子性というものを持ってるってこと。
>>967
その説明は余計な誤解を生むだけなのでは。
969ご冗談でしょう?名無しさん:2006/12/11(月) 02:56:12 ID:???
>>947 955の説明の理由ですが、マイケルソン−モーリーの実験は「宇宙空間に対し、光の進行方向
により干渉縞ができるかどうか」という実験だったから。
あなたの「地球上のどこそこ」というのは「フーコーの振り子」の実験と混同してない?
970ご冗談でしょう?名無しさん:2006/12/11(月) 12:59:50 ID:???
>>961

年も無次元だね
複利適応回数だから

左辺と右辺で次元は合うでしょ
971ご冗談でしょう?名無しさん:2006/12/11(月) 13:05:40 ID:???
>>966
現象に対し、十分説明足る理論がある。

光だけじゃなく電子も二面性を持つ。古典的イメージなら「波束」
が近いかも。
正確にイメージしたいなら量子力学をやるしかない。
972961:2006/12/11(月) 14:30:25 ID:uasm5ZMo
>>963,>>970
預金年数は、その名前から時間という単位を想像させるものの、
そうでなく1回・2回の回数であり、無次元である。

教えてくれた人ありがとう。本当にすごいと思いました。
感心しました。
973ご冗談でしょう?名無しさん:2006/12/11(月) 15:15:51 ID:DABhv48y
初心者向けの相対論の入門書が何冊も出ていますが
個人的にオススメの本があれば教えて下さい。
974ご冗談でしょう?名無しさん:2006/12/11(月) 15:59:36 ID:???
音速を超えると衝撃波が生じますが
水中ではどうなるのでしょうか?
水中での音速が規準となるのでしょうか?
となるとかなりの速度を出さないと衝撃波は発生しないということになりますよね?
975ご冗談でしょう?名無しさん:2006/12/11(月) 16:11:42 ID:???
>>903
もちろん図には書いて考えております。
どういうことなのでしょうか?
どう考えても分かりません。
976ご冗談でしょう?名無しさん:2006/12/11(月) 17:21:00 ID:???
>>975
光の速度のうち物体の運動方向への成分もcだと思ってないか?
977ご冗談でしょう?名無しさん:2006/12/11(月) 17:24:29 ID:Pua563n1
δ(x)を∫[-∞,∞]f(x)δ(x)dx=f(0)を満たす関数と定義したとき
δ(x^2-a^2)=(δ(x-a)-δ(x+a))/2a (a>0)を示すにはどうすればよいか教えてください。
978ご冗談でしょう?名無しさん:2006/12/11(月) 17:27:07 ID:???
>>976
仰る意味がよく理解出来ません。
物体の運動方向云々のことではなく
観測者は物体によって跳ね返った光を見ることで物体を認識するのに
直進してくる光を認識してもそれは物体と相互作用していない光ではないではないのですか?
979ご冗談でしょう?名無しさん:2006/12/11(月) 17:42:56 ID:???
>>978
だから、物体の運動方向に対する光の速度成分が物体の移動速度より小さければ、
物体は光の進路を邪魔することがないから観測者まで到達できるでしょ。

>>890に物体が亜光速なら光がぶつかって観測者に届かないのでは、と書いて
あったので、運動方向成分までcだと思い込んでるんじゃないかと邪推したまで
980ご冗談でしょう?名無しさん:2006/12/11(月) 18:02:39 ID:kjz0Cqlu
>>973
入門書って啓蒙書レベル?だったら
内山龍雄『相対性理論入門』
なんていいんじゃない?
981ご冗談でしょう?名無しさん:2006/12/11(月) 18:14:55 ID:???
>>979
何度何度も繰り返し言っているように
物体は光の進路を邪魔することがないから観測者まで到達できるのは
もちろん分かります。しかし
物体によって進路を邪魔されていない光=物体に当たっていない光=物体の後ろから来た光そのもの=物体の色の情報を含まない光
つまり、物体の背景(物体の背面ではないですよ)もしくは物体の後ろにあるものが見えるだけで
物体の背面は見えないのではないのですか?
982ご冗談でしょう?名無しさん:2006/12/11(月) 18:18:15 ID:???
>>981
物体の後ろから来た光の話などしていない。
わかりにくけりゃ物体自身が発光してるとでも思え。

物体の背面から出た光が物体に邪魔されずに観測者に届けば
物体の背面が見える。それだけ
983ご冗談でしょう?名無しさん:2006/12/11(月) 18:22:18 ID:???
>>981>>982が「背面」としている面が食い違っている気がしてきた
984ご冗談でしょう?名無しさん:2006/12/11(月) 18:25:04 ID:???
読み直したら全く同じ指摘を既に受けてるじゃねーか
>>855とか>>882とか

で、亜光速なら背面からの光は物体に邪魔されるから届かないのでは
という>>890の疑問も既に解決されたはず。
何が疑問として残ってるのかわからない
985ご冗談でしょう?名無しさん:2006/12/11(月) 18:26:27 ID:???
>>983
物体が静止していれば観測者から見えないはずの向こう側の面、じゃないのか?
986ご冗談でしょう?名無しさん:2006/12/11(月) 18:34:09 ID:???
 B
A□C→
 D
   ・観測者

正方形の物体で考えて上のような位置関係だとすると、A面やB面が背面
物体が充分速ければA面から出た光は観測者に到達できて見える。
987ご冗談でしょう?名無しさん:2006/12/11(月) 18:46:33 ID:???
もしかしてA面から出た光は図の左側にしか飛ばないとでも思っているのかな?
988ご冗談でしょう?名無しさん:2006/12/11(月) 19:07:42 ID:???

  B1B2B3B4 B5B6B7B8 B9
A1□ □ □ □ □ □ □ □ □ C1→
A2□ □ □ □ □ □ □ □ □ C2→
A3□ □ □ □ □ □ □ □ □ C3→
A4□ □ □ □ □ □ □ □ □ C4→
A5□ □ □ □ □ □ □ □ □ C5→
A6□ □ □ □ □ □ □ □ □ C6→
A7□ □ □ □ □ □ □ □ □ C7→
A8□ □ □ □ □ □ □ □ □ C8→
A9□ □ □ □ □ □ □ □ □ C9→
  D1D2 D3D4 D5D6 D7D8 D9


            ・観測者


すいません、ちょっとだけ勘違いしていたのは光は直進するものだけではなく
散乱する光を観測者は見るということなのですね。
しかし、例えばB6から360度全方向に光を発すると考えたとして
B6から観測者に向かって放たれた光は
物体がいくら光速で動いていたとしてもB6自体或いはB5とB6の間に
衝突すると思うのですが。
それとも物体の□と□を間をすり抜けてくるというわけではないですよね?
989ご冗談でしょう?名無しさん:2006/12/11(月) 19:27:17 ID:???
>>988
>物体がいくら光速で動いていたとしてもB6自体或いはB5とB6の間に
>衝突すると思うのですが。
うん。だからB面は見えない。キミのいう「背面」はB面だけを指してるの?

こっちは>>985に書いたとおりだからA面も背面。で、こっちは
物体が充分速ければ見える、という話をしていた
990ご冗談でしょう?名無しさん:2006/12/11(月) 19:41:21 ID:??? BE:227210764-2BP(12)
次スレ
■ちょっとした疑問や質問はここに書いてね70■
http://science4.2ch.net/test/read.cgi/sci/1165833649/
991ご冗談でしょう?名無しさん:2006/12/11(月) 20:22:51 ID:???
1)ドーナツ1個が代謝される時に放出されるエネルギーは425kcalである。この値をKJに換算せよ。

2)ドーナツ1個を代謝して得られるエネルギーを使って、高さ1.8mの棚に1kgの本を持ち上げると何冊
  持ち上げられるか。

3)天然ガスを家庭で直接燃やして暖房とする時(効率85%)、必要なメタンの質量はいくつか

4)総合的変換効率0.375の出力500MWの石炭式火力発電所において1年間に生成する電気エネルギーと
 消費する石炭量を求めよ。石炭の発熱量を30KJ/gとせよ


992ご冗談でしょう?名無しさん:2006/12/11(月) 21:03:00 ID:???
テンプレに割り込みこのスレにもマルチする罪深き丸投げ野郎だな
993974:2006/12/11(月) 21:39:43 ID:???
どうかどうかよろしくお願い致します。

994ご冗談でしょう?名無しさん:2006/12/11(月) 21:42:05 ID:???
>827 名前: 800 sage 投稿日:2006/12/09(土) 11:40:55 ID:???
>どうかどうかよろしくお願い致します。

同じやつか?
995ご冗談でしょう?名無しさん:2006/12/11(月) 21:49:39 ID:???
>>993
水中なのに例えば空気の音速が関係あると思うか?
996ご冗談でしょう?名無しさん:2006/12/11(月) 22:01:06 ID:???
>>991
お前化学板でもマルチしてんじゃねーかw
997ご冗談でしょう?名無しさん:2006/12/11(月) 22:14:29 ID:???
そういやあ超音速魚雷ってのもあったな
998ご冗談でしょう?名無しさん:2006/12/12(火) 07:28:42 ID:CB9gbfoJ
ume
999ご冗談でしょう?名無しさん:2006/12/12(火) 07:29:50 ID:CB9gbfoJ
ume
1000ご冗談でしょう?名無しさん:2006/12/12(火) 07:30:40 ID:CB9gbfoJ
ume
10011001
このスレッドは1000を超えました。
もう書けないので、新しいスレッドを立ててくださいです。。。